Download as pdf or txt
Download as pdf or txt
You are on page 1of 135

C H A P T E R 8

Integration Techniques
and Improper Integrals

Section 8.1 Basic Integration Rules ......................................................................719

Section 8.2 Integration by Parts.............................................................................730

Section 8.3 Trigonometric Integrals ......................................................................751

Section 8.4 Trigonometric Substitution ................................................................764

Section 8.5 Partial Fractions ..................................................................................786

Section 8.6 Numerical Integration.........................................................................800

Section 8.7 Integration by Tables and Other Integration Techniques .................808

Section 8.8 Improper Integrals ..............................................................................820

Review Exercises ........................................................................................................833

Problem Solving .........................................................................................................846

© 2018 Cengage Learning. All Rights Reserved. May not be scanned, copied or duplicated, or posted to a publicly accessible website, in whole or in part.
C H A P T E R 8
Integration Techniques and Improper Integrals
Section 8.1 Basic Integration Rules
1. Use long division to rewrite the function as the sum of a polynomial and a proper rational function.

2. (a) Write the integral as a sum:


2 + x 2 x
 x2 + 9
dx =  x2 + 9
dx +  x2 + 9
dx

(b) Use a trigonometric identity:

 cot  (csc x − 1) dx
2 2
x dx =

d  1 2x
x 2 + 1 + C  = 2 ( x 2 + 1) ( 2 x) =
−1 2
3. (a) 2
dx    2 x2 + 1
d  2 1 x
x + 1 + C  = ( x 2 + 1) ( 2 x) =
−1 2
(b)
dx   2 2
x +1
d 1  1 1  x
x 2 + 1 + C  =  ( x 2 + 1) ( 2 x) =
−1 2
(c)
dx  2  2  
2 2 x2 + 1
d  2x
(d) ln ( x 2 + 1) + C  = 2
dx  x +1
x
 x2 + 1
dx matches (b).

d  1  2x  x
4. (a) ln x2 + 1 + C =  2  = 2
dx   2  x + 1 x +1
 
 = (
x 2 + 1) ( 2) − ( 2 x)( 2)( x 2 + 1)( 2 x) 2(1 − 3 x 2 )
2
d  2x
(b) + C =
dx  ( x 2 + 1)2 
( x2 + 1)
4
( x2 + 1)
3
 
d 1
(c) [arctan x + C ] =
dx 1 + x2
d  2x
(d) ln ( x 2 + 1) + C  = 2
dx  x +1
1
 x2 +1
dx matches (c).

1
 (5x − 3)
4
5. dx 7.  dx
u = 5 x − 3, du = 5 dx, n = 4
(
x1− 2 x )
1
u u =1− 2 x , du = − dx
n
Use du.
x

2t + 1 du
6.  t2 dt Use  u
.
+t − 4
u = t 2 + t − 4, du = ( 2t + 1) dt

du
Use  u
.

© 2018 Cengage Learning. All Rights Reserved. May not be scanned, copied or duplicated, or posted to a publicly accessible website, in whole or in part. 719
720 Chapter 8 Integration Techniques and Improper Integrals

2 16. Let u = t + 6, du = dt.


8.  (2t − 1) + 4
2
dt
5 −3
 (t dt = 5(t + 6) dt
+ 6)
3
u = 2t − 1, du = 2 dt , a = 2
−2
du (t + 6)
Use  u2 + a2 . = 5⋅
−2
+C

−5
3 = + C
9.  1−t 2
dt 2(t + 6)
2

u = t , du = dt , a = 1 17. Let u = z − 10, du = dz.


du 7 7
Use  a2 − u 2
.
 ( z − 10)7 dz = 7  ( z − 10)
−7
dz = − +C
6( z − 10)
6

−2 x
10.  x2 − 4
dx 18. Let u = t 4 + 1, du = 4t 3 dt.
1
(t 4 + 1) (4t 3 ) dt
12
t 4
3
1 t 4 + 1 dt =
u = x 2 − 4, du = 2 x dx, n = −
2
1 (t + 1)
32
4

 u du.
n
Use = ⋅ +C
4 (3 2)
1 4
 t sin t (t + 1) + C
2 32
11. dt =
6
u = t 2 , du = 2t dt
 1 
 z + (1 − z )  dz
−6
Use  sin u du. 19.  z
2
+  dz =
(1 − z )  6
2


 sec 5 x tan 5 x dx
−5
12.
=
z3
+
(1 − z ) +C
u = 5 x, du = 5 dx 3 5
z3 1
= + +C
Use  sec u tan u du. 3 5(1 − z )
5

z3 1
 (cos x)e
sin x
13. dx = − +C
5( z − 1)
5
3
u = sin x, du = cos x dx
 
e 2
u
Use du. =  4 x dx −  2( 2 x + 3)
−2
20.  4 x − (2 x + 3)2  dx dx
 
1
x
−1
14.
2
dx
= 2x2 −
(2 x + 3)
+C
x − 4
−1
u = x, du = dx, a = 2 1
= 2x2 + +C
du 2x + 3
Use u u 2 − a2
.
21. Let u = −t 3 + 9t + 1,
15. Let u = x − 5, du = dx. du = ( −3t 2 + 9) dt = −3(t 2 − 3) dt.

 14( x − 5) dx = 14 ( x − 5) dx = 2( x − 5) + C
6 6 7
1 −3(t − 3)
2
t2 − 3
 −t 3 + 9t + 1 dt = −
3  −t 3 + 9t + 1
dt

1
= − ln −t 3 + 9t + 1 + C
3

© 2018 Cengage Learning. All Rights Reserved. May not be scanned, copied or duplicated, or posted to a publicly accessible website, in whole or in part.
Section 8.1 Basic Integration Rules 721

22. Let u = 3x 2 + 6 x, du = (6 x + 6) dx = 6( x + 1) dx. 3x  12 


24.  x + 4 dx =   3 −  dx
x + 4
x +1 1
(3 x 2 + 6 x) 6( x + 1) dx
−1 2
 2
3x + 6 x
dx =
6  = 3 x − 12 ln x + 4 + C

1 (3 x + 6 x)
12
2
x + 2 x +1+1
= ⋅ +C 25.  x +1
dx =  x +1
dx
6 (1 2)
 1 
=
1
3x2 + 6 x + C
=  1 +

 dx
x + 1
3
= x + ln x + 1 + C
x2 1
23.  x −1
dx =  ( x + 1) dx +  x −1
dx

1 2
= x + x + ln x − 1 + C
2

 1 1  1 1
26.   9 z − 5 − 9 z + 5  dz =
9
ln 9 z − 5 − ln 9 z + 5 + C
9
1 9z − 5
= ln +C
9 9z + 5

 (5 + 4 x )  (25 + 40 x + 16 x 4 ) dx
2
csc 2 3t 1 1
(− csc2 3t ) dt
2 2
27. dx = 32.  cot 3t
dt = −
3  cot 3t
40 3 16 5
= 25 x + x + x +C 1
3 5 = − ln cot 3t + C
3
x
=
15
(48 x 4 + 200 x 2 + 375) + C
33. Let u = 1 + e x , du = e x dx.
2
 2  4 2  2  e x 
28.  x 3 +  dx =
x   9 x + 12 +  dx
x  e− x +1
dx = 2  − x
e +
  dx
1  e x 
9 2 ex
= x + 12 x + 4 ln x + C = 2 dx = 2 ln (1 + e x ) + C
2 1 + ex

29. Let u = 2π x 2 , du = 4π x dx. 4 e− x


1
34.  3 − ex dx = 4 
3e − 1
dx
−x

 x(cos 2π x ) dx 4π 
(cos 2π x 2 )(4π x) dx
2
= 4
= − ln 3e − x − 1 + C
1 3
= sin 2π x 2 + C

ln x 2 1
35.  dx = 2 (ln x) dx
30. Let u = π x, du = π dx. x x
(ln x)
2
1 = 2 + C = (ln x) + C
2
 csc π x cot π x dx (csc π x)(cot π x) π
π
= dx 2
1
= − csc π x + C −sin x
π 36. Let u = ln (cos x), du = dx = − tan x dx.
cos x
31. Let u = cos x, du = −sin x dx.
 ( tan x)(ln cos x) dx = −  (ln cos x)( − tan x) dx
sin x −1 2
dx = −  (cos x) ( −sin x) dx
2
 cos x
−ln (cos x)
=  + C
2
= −2 cos x + C
1 + cos α
37.  sin α
dα =  csc α dα +  cot α dα
= −ln csc α + cot α + ln sin α + C

© 2018 Cengage Learning. All Rights Reserved. May not be scanned, copied or duplicated, or posted to a publicly accessible website, in whole or in part.
722 Chapter 8 Integration Techniques and Improper Integrals

1 1 cos θ + 1 cos θ + 1  2 2 sin ( 2 t )


38. = ⋅
= 41. Let u = cos , du = dt.
cos θ − 1 cos θ − 1 cos θ + 1 cos 2 θ − 1 t t2
cos θ + 1
= = −csc θ ⋅ cot θ − csc 2 θ tan ( 2 t ) 1 1  2 sin ( 2 t ) 
−sin 2 θ  t 2
dt =  
2 cos( 2 t )  t2
 dt

1
 cos θ − 1 dθ =  (−csc θ cot θ − csc θ ) dθ
2
1  2
= ln cos  + C
2 t
= csc θ + cot θ + C
1 cos θ 1
= + +C 42. Let u = − = −t − 3 , du = 3t − 4 dt.
sin θ sin θ t3
1 + cos θ 3
= +C e −1 t 1
sin θ  t4
3
dt =  e −1 t 3t − 4 dt
3
( )
39. Let u = 4t + 1, du = 4 dt. 1 3
= e −1 t + C
3
−1 1 4
 dt = −
4
dt
6 6
1 − ( 4t + 1) 1 − ( 4t + 1)
2 2
43.  dz =  (3dz )
z 9 z 2 − 25 (3 z )
2
1 3z − 52
= − arcsin ( 4t + 1) + C
4 6  3z 
= arcsec   + C
5  5 
40. Let u = 2 x, du = 2dx, a = 5.
1 1 1
 25 + 4 x 2 dx (2) dx
2  52 + ( 2 x ) 2
=

1 2x
= arctan +C
10 5

1 2
44.  ( x − 1) dx =  dx = arcsec 2( x − 1) + C
4x2 − 8x + 3 2
2( x − 1) 2( x − 1) − 1

4 1 1 1
45.  4x2 + 4 x + 65
dx =  x + (1 2) 2 dx 46.  x2 − 4x + 9
dx =  x2− 4x + 4 + 5
dx
  + 16
1
1  x + (1 2)  =  dx
( )
2
= arctan   +C ( x − 2) + 5
2
4  4 
1  2 x + 1 1  x − 2
= arctan  +C = arctan  +C
4  8  5  5 
5  5 
= arctan  ( x − 2)  + C
5  5 

ds t  1
47. = ,  0, − 
dt 1−t 4
 2
s
(a) (b) u = t 2 , du = 2t dt
1

t 1 2t
 dt =
2
dt
1 − t4 1 − (t 2 )
2
0.8
t
−1 1
1
= arcsin t 2 + C
2 − 1.2 1.2

−1
 1 1 1 1
 0, − : − = arcsin 0 + C  C = −
 2 2 2 2 − 0.8

1 2 1
s = arcsin t −
2 2

© 2018 Cengage Learning. All Rights Reserved. May not be scanned, copied or duplicated, or posted to a publicly accessible website, in whole or in part.
Section 8.1 Basic Integration Rules 723

dy 1  1 dy
= ( 4 − e 2 x ) = 16 − 8e 2 x + e 4 x
2
48. (a) = ,  2,  52.
dx 4x − x 2  2 dx
 (16 − 8e + e 4 x ) dx
2x
y y =
2
1 4x
= 16 x − 4e 2 x + e +C
1 4
x
4 dr 10et
53. =
−1
dt 1 − e 2t
−2
10et
r =  dt
1 − (et )
2
1
(b) y =  4 x − x2
dx
= 10 arcsin (et ) + C
1
=  4 − ( x 2 − 4 x + 4)
dx

(1 + et )
2
dr 1 + 2e t + e 2 t
1  x − 2 54. = = = e − 3 t + 2e − 2 t + e − t
=  dx = arcsin 
 2 
+C dt e3t e 3t
4 − ( x − 2)
2

(e + 2e − 2t + e − t ) dt
− 3t
r =
 1 1 1 1
 2, : = arcsin (0) + C  C = = − e − 3t − e − 2 t − e − t + C
 2 2 2 3
 x − 2 1
y = arcsin   + dy sec 2 x
 2  2 55. =
dx 4 + tan 2 x
2
Let u = tan x, du = sec 2 x dx.

sec 2 x 1  tan x 
 4 + tan 2 x dx
0 4
y = = arctan   +C
2  2 
−2
1
9 56. y′ =
49. x 4 x2 − 9
Let u = 2 x, du = 2dx, a = 3.
1 1
−5 3
y = x dx =  (2) dx
−1
4x − 9 2
( 2 x ) ( 2 x)2 − 32
y = 4e 0.8 x 1 2x
= arcsec +C
3 3
50. (0, 1) 5
1 1
2 3 (2 − 3t ) 2 3 (3t − 2) dt
4 4
57. dt =
1
−2 2 1 (3t − 2)5 
=  
−2  3 5  2 3

y = 5 − 4e − x 1 1
= (1 − 0) =
15 15
dy
= (e x + 5) = e 2 x + 10e x + 25
2
51. 0 5 0
dx −11
58. −1 (t + 2)11 dt = −1 5(t + 2) dt
 (e + 10e x + 25) dx
2x
y =
0 0
1 2x  (t + 2)−10   −1 
= e + 10e x + 25 x + C = 5  =  
2  ( −10)  −1  2(t + 2)10  −1
1 1 1 1023
= −  10 −  = ≈ 0.5
2 2 1 2048

© 2018 Cengage Learning. All Rights Reserved. May not be scanned, copied or duplicated, or posted to a publicly accessible website, in whole or in part.
724 Chapter 8 Integration Techniques and Improper Integrals

59. Let u = 2 x, du = 2 dx. 1 ex 1 ex


64. −3 e2 x dx = −3 dx
(e x + 1)
2
π 4
1 π 4 cos
+ 2e x + 1
0 cos 2 x dx = 
2 0
2 x( 2) dx

−3 (e + 1) e dx
1 −2
x x
π 4 =
=  12 sin 2 x = 1
2
0
1
= − (e x + 1) 
−1

60. Let u = sin t , du = cos t dt.   −3


1
π π  −1 
 0 sin t cos t dt =  13 sin 3 t  = 0
2
=  x 
0
 e + 1 − 3
−1 1
61. Let u = − x 2 , du = −2 x dx. = + −3 ≈ 0.684
e +1 e +1
1 1 1
− x2
dx = − 12  e − x ( −2 x) dx = − 12 e− x 
2 2
 0 xe 0  0 65. Let u = x 2 + 36, du = 2 x dx.
= 1
(1 − e−1 ) ≈ 0.316 2x
 0 (x ) (2 x) dx
8 8 −1 2
0
2 2
dx = + 36
2
x + 36
−1
62. Let u = 1 − ln x, du = dx. 12 8
x = 2  x 2 + 36
( )  = 8
  0
e1 − ln x e  −1 
1 dx = −  (1 − ln x)  dx
x 1
 x 3 2 x 2 + 3x − 2 3 2
e
66. 1 x
dx = 1  2 x + 3 −  dx
x
 1 2 1
= − (1 − ln x)  = 3
 2 1 2 =  x 2 + 3 x − 2 ln x 
1

= (9 + 9 − 2 ln 3) − (1 + 3 − 0)
ln ( x + 1)
3
3 3 1
63. 2 dx = 3 ln ( x + 1) dx = 14 − 2 ln 3
x +1 2 x +1
3
 ln ( x + 1) 2  5 2t 5 2t − 4 5 4
= 3    67. 3 2
dt = 3 2
dt + 3 (t − 2)2 dt
 2  t − 4t + 4 t − 4t + 4
 2
5
 4 
3 2 3
= ln ( x + 1)  = ln (t 2 − 4t + 4) −
2  2  t − 2  3
3  4
= (ln 4) − (ln 3)  ≈ 1.072
2 2
=  ln 9 −  − (ln 1 − 4)
2   3
8
= ln 9 + ≈ 4.864
3

4 x3 4 x 3 − 12 x
2 ( x − 3)
4 4 4 −2
68. 2 = 2 dx + 2
(12 x) dx
x − 6x2 + 9
4
x − 6x2 + 9
4

4
 6 
= ln ( x 4 − 6 x 2 + 9) − 2
 x − 3  2
 6
=  ln 169 −  − (ln 1 − 6)
 13 
72
= ln 169 +
13
72
= 2 ln 13 + ≈ 10.688
13

© 2018 Cengage Learning. All Rights Reserved. May not be scanned, copied or duplicated, or posted to a publicly accessible website, in whole or in part.
Section 8.1 Basic Integration Rules 725

69. Let u = 3x, du = 3 dx. 3x + 2


5
74. A = 0
x2 + 9
dx
2 3 1 1 2 3 3
0 dx =  dx 5
=  2
3x 5 2
4 + (3 x ) 0
2 2
4 + 9x 3 0 dx + dx
0 x + 9 x2 + 9
2 3 5
1  3 x  3 2  x 
=  arctan   =  ln x 2 + 9 + arctan  
6  2  0 2 3  3  0
π 3 2  5 3
= ≈ 0.175 = ln (34) + arctan   − ln 9
18 2 3  3 2
7 3  34  2 5
7 1   x  7 = ln   + arctan  
70. 0 100 − x 2
dx = arcsin   = arcsin  
  10 0  10 
2 9 3  3
≈ 2.6806
0
0  −1 1 − x  75. y 2 = x 2 (1 − x 2 )
− 4 3
1− x
71. dx =  3 
 ln 3 −4
y = ± x 2 (1 − x 2 )
1
= −
ln 3
(3 − 35 ) 1
A = 4  x 1 − x 2 dx
0
240
= −2 (1 − x 2 )
1 12
= ≈ 218.457 (−2 x) dx
ln 3 0

4 32 1
1 1 1 = − (1 − x 2 ) 
x2 + 2 x
( x + 1) dx = x2 + 2 x
( 2 x + 2) dx   0
72. 0 7 2 0 7 3
4 4
 1 
1 = − (0 − 1) =
= 
2
⋅ 7x + 2x  3 3
 2 ln 7 0
π 2 π 2
1 171 76. A = 0 sin 2 x dx = − 12 [cos 2 x]0 = − 12 ( −1 − 1) = 1
= 73 − 1 = ≈ 87.877
2 ln 7  ln 7
1 1  x + 2
3 2
(−4 x + 6)
32 77.  x2 dx = arctan   +C
73. A = 0 dx + 4 x + 13 3  3 
3 2 The antiderivatives are vertical translations of each other.
= − 14 
0
(6 − 4 x)
32
(−4) dx 1

32
= − 14  52 (6 − 4 x) 
52 C=0
 0 −7 5

= 1
− 10 (0 − 6 ) 52
C = − 0.2

18 −1
= 5
6 ≈ 8.8182

x − 2 1 4  x + 2 6
78.  2
dx = ln ( x 2 + 4 x + 13) − arctan   +C
x + 4 x + 13 2 3  3 
The antiderivatives are vertical translations of each other. − 10 10

−6

1  −2  6
79.  1 + sin θ dθ = tan θ − sec θ + C  or
1 + tan θ 2

( )  C=2

π 7π

2 2
The antiderivatives are vertical translations of each other. C=0

−6

© 2018 Cengage Learning. All Rights Reserved. May not be scanned, copied or duplicated, or posted to a publicly accessible website, in whole or in part.
726 Chapter 8 Integration Techniques and Improper Integrals

3 5
 e x + e− x  1 3x
80.    dx = e + 9e x − 9e − x − e −3 x  + C
 2  24 
−5 5

The antiderivatives are vertical translations of each other.


−5

1a
81. No. When u = x 2 , it does not follow that x = u 1 a  1
 0 ( x − ax ) dx
1a
2
84. =  x 2 − x3  =
because x is negative on [−1, 0). 2 3 0 6a 2
1 2 1
sin x cos x sin x(1 + sin x ) + cos 2 x Let = , 12a 2 = 3, a = .
82. + = 6a 2 3 2
cos x 1 + sin x cos x(1 + sin x) y

sin x + sin 2 x + cos 2 x ( a1 , a1)


= 2
cos x(1 + sin x) y=x

sin x + 1
= 1

cos x(1 + sin x ) y = ax 2

1 x
= = sec x 1 2
cos x
So,
85. (a) They are equivalent because
 sin x cos x 
 sec x dx =   cos x + 1 + sin x  dx e x + C1 = e x ⋅ eC1 = Ce x , C = eC1 .
(b) They differ by a constant.
= −ln cos x + ln 1 + sin x + C
sec 2 x + C1 = ( tan 2 x + 1) + C1 = tan 2 x + C
1 + sin x
= ln +C
cos x 5
86.  f ( x) dx < 0 because there is more area below the
= ln sec x + tan x + C 0

x-axis than above.


83. sin x + cos x = a sin ( x + b)
2 4x
sin x + cos x = a sin x cos b + a cos x sin b 87. 0 x2 + 1
dx ≈ 3
sin x + cos x = ( a cos b) sin x + ( a sin b) cos x
Matches (a).
Equate coefficients of like terms to obtain the following. y

1 = a cos b and 1 = a sin b 3

So, a = 1 cos b. Now, substitute for a in 1 = a sin b. 2

 1 
1 =   sin b
1

 cos b 
x
π 1 2 3
1 = tan b  b =
4
π 1 2 4
Because b = ,a = = 2. So, 88. 0 x2 + 1
dx ≈ 4
4 cos(π 4)
Matches (d).
 π
sin x + cos x = 2 sin  x + . y

 4
dx dx
 sin x + cos x =  2 sin ( x + (π 4))
3

2
1  π
=
2  csc x +  dx
 4
1

x
1  π  π 1 2 3 4
=− ln csc x +  + cot  x +  + C
2  4   4

© 2018 Cengage Learning. All Rights Reserved. May not be scanned, copied or duplicated, or posted to a publicly accessible website, in whole or in part.
Section 8.1 Basic Integration Rules 727

89. (a) y = 2π x 2 , 0 ≤ x ≤ 2 1
(c) x = , 0 ≤ y ≤ 4
y 2
4 1
2π  y   dy
25

20 0
 2
15 y

10
4
5

x
−3 −2 −1 1 2 3
x=1
2

(b) y = 2 x, 0 ≤ x ≤ 2
y
y= 2x x
−2 2
3

1
x
−3 −2 −1 1 2 3
−1

−2
−4
−3

(c) y = x, 0 ≤ x ≤ 2 91. (a) Shell Method:


y
Let u = − x 2 , du = −2 x dx.
3
1 2
2 V = 2π  xe− x dx
0
1 2
y=x = −π  e − x ( −2 x) dx
0
x
−2 2 1
= −π e − x 
2
−1
 0
= π (1 − e −1 ) ≈ 1.986
90. (a) x = π y, 0 ≤ y ≤ 4
y

1
y = x, 0 ≤ x ≤ 4π
π 1

1
8 2

4 x
1 1
2 2

−2
π 2π 3π 4π (b) Shell Method:
−4 b 2
V = 2π  xe − x dx
0
(b) x = y, 0 ≤ y ≤ 4 b
= −π e − x 
2

y = x , 2
0 ≤ x ≤ 2  0

( ) = 43
y 2
= π 1 − e−b
4
2 3π − 4
e−b =

y = x2  3π 
b = ln   ≈ 0.743
 3π − 4 

x
−2 −1 1 2

© 2018 Cengage Learning. All Rights Reserved. May not be scanned, copied or duplicated, or posted to a publicly accessible website, in whole or in part.
728 Chapter 8 Integration Techniques and Improper Integrals

92. y
94. y = ln (cos x), 0 ≤ x ≤ π 3
y = sin(x 2)
−sin x
y′ = = − tan x
cos x
(π (2
,
2
2

0.5
[1 + ( y′)2 = 1 + tan 2 x = sec 2 x
π 3 π 3
1 + ( y′) dx =
2
0.5 1
x s = 0 0 sec x dx
y = cos(x 2) π 3
= ln sec x + tan x  0
Shell Method:
= ln 2 + ( )
3 − ln (1) = ln 2 + ( )
3 ≈ 1.317
( )
π 2
V = 2π  x cos( x 2 ) − sin ( x 2 ) dx
0

π 95. y = 2 x
= π sin ( x 2 ) + cos( x 2 ) 2 1
0
y′ =
 2  x
2
= π  +  − (0 + 1) 1 x +1
 2 2   1 + ( y′) = 1 +
2
=
x x
= π ( 2 −1 ) 9 x +1
S = 2π  2 x dx
0 x
93. y = f ( x) = ln(sin x) 9
= 2π  2 x + 1 dx
cos x 0
f ′( x) = 9
sin x  2 3 2 8π
π 2 cos 2 x π 2 sin 2 x + cos 2 x
= 4π  ( x + 1)  =
  3  0 3
10 10 − 1 ≈ 256.545 ( )
s= π 4 1+
sin 2 x
dx = π 4 sin 2 x
dx y

π 2 1 π 2 12
=  π 4 sin x dx =  π 4 csc x dx 9

π 2
= −ln csc x + cot x π 4
6

( 2 + 1)
= −ln(1) + ln x
3 6 9 12
= ln( 2 + 1) ≈ 0.8814

2
2 1 1  1
96. A = 0 dx =  ln ( 2 x + 1) = ln 5 ≈ 0.805 y
2x + 1  2 0 2

1 2  1 
A 0  2 x + 1 
x = x  dx 1

1 2 1 2x + 1 − 1
A 0 2
= ⋅ dx −1 1 2
x

2x + 1
−1
(0.743, 0.25)
1 2 1 
2 A 0 
= 1 −  dx
2x + 1
2
1 1 
=  x − 2 ln ( 2 x + 1)
2A
 0
1  1  4 − ln 5
= 2 − ln 5 = ≈ 0.743
2 A  2  2 ln 5
1 2 1 1 1 2 1
y =
A 0 ⋅
2 2x + 1
dx =
2A 0 2x + 1
dx
2
1 1  ln 5
=
2A  2 ln ( 2 x + 1) = 4 ln 5 = 0.25
 0

© 2018 Cengage Learning. All Rights Reserved. May not be scanned, copied or duplicated, or posted to a publicly accessible website, in whole or in part.
Section 8.1 Basic Integration Rules 729

1 b 99. y = tan (π x)
97. Average value =  f ( x) dx
b − a a
y′ = π sec 2 (π x)
1 3 1
= 
3 − ( −3) 3 1 + x 2

dx 1 + ( y′) = 1 + π 2 sec 4 (π x)
2

1 14
1 + π 2 sec 4 (π x ) dx ≈ 1.0320
0
3
= arctan ( x) −3 s =
6
1
= arctan (3) − arctan ( −3) 100. y = x2 3
6
1 2
= arctan (3) ≈ 0.4163 y′ =
3 3 x1 3
4
1 + ( y′) = 1 +
2
1 b 9x2 3
f ( x) dx
b − aa
98. Average value =
8 4
1 π n
sin ( nx) dx
s = 1 1+
9x2 3
dx ≈ 7.6337
(π n) − 0  0
=

π n
n  −1 
= cos( nx)
π  n 0
1 2
= − cos(π ) − cos(0) =
π π

sin 3 x 1
 cos  (1 − sin x) cos x dx + C = sin x(cos 2 x + 2) + C
3 2
101. (a) x dx = = sin x −
3 3

 (1 − sin x)  (1 − 2 sin x + sin 4 x) cos x dx


2
 cos
5 2 2
(b) x dx = cos x dx =
2 sin 5 x 1
= sin x − sin 3 x + +C = sin x(3 cos 4 x + 4 cos 2 x + 8) + C
3 5 15

 (1 − sin x) cos x dx
3
 cos
7 2
(c) x dx =

 (1 − 3 sin x + 3 sin x − sin x) cos x dx


2 4 6
=
3 1
= sin x − sin 3 x + sin 5 x − sin 7 x + C
5 7
1
= sin x(5 cos6 x + 6 cos 4 x + 8 cos 2 x + 16) + C
35

 (1 − sin x)
7
 cos
15 2
(d) x dx = cos x dx

You would expand (1 − sin 2 x) .


7

 tan  (sec x − 1) tan x dx  (sec x − 1) tan 3 x dx


3 2
102. (a) x dx = (b)  tan 5 x dx = 2

 sec x tan x dx −  tan x dx


2
= tan 4 x
 tan
3
= − x dx
4
tan 2 x
= −  tan x dx
2
tan 2 x
 tan x dx = 2 + ln cos x + C
3

 tan  (sec x − 1) tan 2 k −1 x dx


2k +1 2
(c) x dx =
tan 2 k x
 tan
2 k −1
= − x dx
2k
(d) You would use these formulas recursively.

© 2018 Cengage Learning. All Rights Reserved. May not be scanned, copied or duplicated, or posted to a publicly accessible website, in whole or in part.
730 Chapter 8 Integration Techniques and Improper Integrals

103. Let f ( x ) =
1
2
x ( x 2 + 1 + ln x + x2 + 1 ) + C.
1 1 2 1  1 2 
 x ( x + 1) ( 2 x) + 1 + ( x + 1) ( 2 x)  
−1 2 −1 2
f ′( x) = x2 + 1 +
2 2 x + x2 + 1  2 
1 x2 1  x 
=  + x2 + 1 + 1 + 
2  x +12
x + 2
x + 1 x + 1  
2

1  x + ( x + 1)  x2 + 1 + x 
2 2
1
=  +  
2
 x2 + 1 x + x + 1 
2
x 2 + 1  

1  2( x + 1) 
2
1  2 x2 + 1 1 
=  +  =   = x2 + 1
2  x2 + 1 x2 + 1  2  x2 + 1 
 

So,  x 2 + 1 dx =
1
2(x x 2 + 1 + ln x + x2 + 1 ) + C.
Let g ( x) =
1
2
(
x x 2 + 1 + arcsinh ( x) . )
1 1 2 1 
 x ( x + 1) ( 2 x) +
−1 2
g ′( x) = x2 + 1 + 
2 2 2
x + 1
1 x2 1 
=  + x2 + 1 + 
2 x +12 2
x + 1

=
1 x +

2
( x2+ 1) + 1 

2 2
x +1 
 
1  2( x + 1) 
2
=   = x2 + 1
2  x2 + 1 

So,  x 2 + 1 dx =
1
2
(x x 2 + 1 + arcsinh ( x) + C . )
4 ln (9 − x)
104. Let I = 2 ln (9 − x) + ln ( x + 3)
dx.

I is defined and continuous on [2, 4]. Note the symmetry: as x goes from 2 to 4, 9 − x goes from 7 to 5 and x + 3 goes from
5 to 7. So, let y = 6 − x, dy = −dx.

2 ln (3 + y ) 4 ln (3 + y )
I = 4 (−dy ) = 2 dy
ln (3 + y ) + ln (9 − y ) ln (3 + y ) + ln (9 − y )

Adding:
4 ln (9 − x) 4 ln (3 + x) 4
2I = 2 ln (9 − x) + ln ( x + 3)
dx + 2 ln (3 + x) + ln (9 − x)
dx =  2 dx = 2  I =1

You can easily check this result numerically.

Section 8.2 Integration by Parts


1. Integration by parts is based on the formula for the 2. You should try letting u be a portion of the integrand
derivative of a product. whose derivative is a function simpler than u. You
should try letting dv be a function whose integral is easy
to calculate.

© 2018 Cengage Learning. All Rights Reserved. May not be scanned, copied or duplicated, or posted to a publicly accessible website, in whole or in part.
Section 8.2 Integration by Parts 731

3. Let u be that single term and dv = dx. x4


11. dv = x3 dx  v = x
3
dx =
4
4. The tabular method is useful for integrals of the form
1
u = ln x  du = dx
x sin ax dx,  x n cos ax dx, and x
n n
e ax dx. x

x  v du
3
ln x dx = uv −
 xe dx
9x
5.
x4  x4  1
u = x, dv = e9 x dx = (ln x) −   4  x dx
4
x4 1 3
x e
2 2x
6. dx
4
= ln x − x dx
4
u = x 2 , dv = e 2 x dx
x4 1 4
= ln x − x +C
4 16
 (ln x)
2
7. dx 1 4
= x ( 4 ln x − 1) + C
u = (ln x) , dv = dx
2 16

12. dv = e x 2 dx  e
x2
v = dx = 2e x 2
8.  ln 5 x dx
u = ln 5 x, dv = dx u = 7 − x  du = − dx

 (7 − x)e dx  v du
x 2
= uv −
 x sec x dx
2
9.
= (7 − x )( 2e x 2 ) +  2e
x 2
dx
u = x, dv = sec 2 x dx
= 2(7 − x)e x 2
+ 4e x 2
+C
10. x
2
cos x dx = (18 − 2 x )e x 2
+ C

u = x 2 , dv = cos x dx

1
13. dv = sin 4 x dx  v =  sin 4 x dx = −
4
cos 4 x

u = 2x + 1  du = 2 dx

 (2 x + 1) sin 4 x dx = uv −  v du
 1   1 
= ( 2 x + 1) − cos 4 x  −   − cos 4 x ( 2 dx)
 4   4 
1 1
= − ( 2 x + 1)cos 4 x + sin 4 x + C
4 8

1 1 4x
14. dv = cos 4 x dx  v =  cos 4 x dx =
4
sin 4 x 15. dv = e 4 x dx  v = e
4x
dx =
4
e dx

u = x  du = dx u = x  du = dx

 x cos 4 x dx = uv −  v du 1  1 
 xe dx = x e 4 x  −   e 4 x  dx
4x

1  1 4  4 
= x sin 4 x  −  sin 4 x dx
4  4 x 4x 1 4x
= e − e + C
x 1 4 16
= sin 4 x + cos 4 x + C
4 16 e4 x
= (4 x − 1) + C
16

© 2018 Cengage Learning. All Rights Reserved. May not be scanned, copied or duplicated, or posted to a publicly accessible website, in whole or in part.
732 Chapter 8 Integration Techniques and Improper Integrals

1
16. dv = e −2 x dx  v = e
−2 x
dx = − e − 2 x
2
u = 5 x  du = 5dx
5x
 e2 x  5 xe
−2 x
dx = dx

 1   1 
= (5 x) − e −2 x  −   − e −2 x 5dx
 2   2 
5 −2 x 5 −2 x
= − xe +  e dx
2 2
5 −2 x 5 −2 x
= − xe − e +C
2 4
5
= − e −2 x ( 2 x + 1) + C
4

17. Use integration by parts three times.


(1) dv = e x dx  e
x
v = dx = e x
u = x3  du = 3 x 2 dx

(2) dv = e x dx  e
x
v = dx = e x
u = x2  du = 2 x dx

(3) dv = e x dx  e
x
v = dx = e x
u = x  du = dx

xe dx = x e − 3 x 2e x dx = x3e x − 3x 2e x + 6 xe x dx
3 x 3 x

= x3e x − 3 x 2e x + 6 xe x − 6e x + C = e x ( x3 − 3x 2 + 6 x − 6) + C

e1 t  −1  1
18.  t2
dt = −  e1 t  2  dt = −e1 t + C
t 
21. Let u = ln x, du =
x
dx.

(ln x) (ln x)
2 3
2 1 
19. dv = t dt  v =  t dt =
t2  x
dx =  (ln x)  x  dx = 3 + C
2
1
u = ln(t + 1)  du = dt 1
t +1 22. dv = x − 3 dx  v = x
−3
dx = − x − 2
2
t2 1 t2
 t ln(t + 1) dt =
2
ln (t + 1) − 
2 t +1
dt u = ln x  du =
1
dx
x
t2 1  1 
= ln (t + 1) −   t − 1 +  dt ln x 1  1 −2  1
2 2  t + 1  x3
dx = − x − 2 ln x −
2   − 2 x  dx
x
t2 1 t 2 
= ln (t + 1) −  − t + ln (t + 1) + C 1 1
2 2 2  = − ln x +  x −3 dx
2x2 2
1 2
= 2(t − 1) ln t + 1 − t 2 + 2t  + C 1  1  x−2
4 = − 2 ln x +   +C
2x  2 −2
1 6 1 1
20. dv = x5 dx  x = − 2 ln x − +C
5
v = dx = x
6 2x 4x2
1
u = ln 3x  du = dx
x
x6 x6  1 
x 
5
ln 3 x dx = ln 3 x −   dx
6 6  x
x6 x6
= ln 3 x − +C
6 36

© 2018 Cengage Learning. All Rights Reserved. May not be scanned, copied or duplicated, or posted to a publicly accessible website, in whole or in part.
Section 8.2 Integration by Parts 733

1 −2
23. dv =
(2 x + 1)
2
dx  v =  (2 x + 1) dx

1
= −
2( 2 x + 1)
u = xe 2 x  du = ( 2 xe 2 x + e 2 x ) dx
= e 2 x ( 2 x + 1) dx

xe 2 x xe 2 x e2 x
 (2 x + 1)2 dx = − 2(2 x + 1) +  2
dx

− xe2 x e2 x e2 x
= + +C = +C
2( 2 x + 1) 4 4( 2 x + 1)

x 1
 (x + 1)
−2
24. dv = dx  v = 2
x dx = −
(x 2
+ 1)
2
2( x 2 + 1)

u = xe 2 x2
 du = 2 x3e x ( 2
+ 2 xe x
2
) dx = 2xe x2
( x2 + 1) dx

2 2 2 2 2
x3e x x 2e x 2 x 2e x ex ex
  xe dx = −
x
dx = − + + + C = +C
( x2 + 1)
2
2( x 2 + 1) 2( x 2 + 1) 2 2( x 2 + 1)

 ( x − 5) (x − 5)
12 2 32
25. dv = x − 5 dx  v = dx = 3
u = x  du = dx

x − 5 dx = x 23 ( x − 5)  23 ( x − 5)
32 32
x − dx

(x − 5) (x − 5)
2x 32 4 52
= 3
− 15
+C

= 2
15 (x − 5)
32
(5 x − 2( x − 5)) + C
(x − 5) (3 x + 10) + C
2 32
= 15

−1 2 −1 2 1
26. dv = (1 − 6 x)  (1 − 6 x) dx = − (1 − 6 x)
12
dx  v =
3
u = 2x  du = 2dx
2x −1 2
 1 − 6x
dx =  (1 − 6 x) 2 x dx

= uv −  v du
 1 1 2 1
= ( 2 x) − (1 − 6 x)  −  − (1 − 6 x) ( 2 dx)
12

 3  3
2x 2
= − (1 − 6 x) − (1 − 6 x) + C
12 32

3 27
 2x
(1 − 6 x) + C
2
= 1 − 6 x − −
 3 27 
− 6x − 2
= 1 − 6x   +C
 27 

© 2018 Cengage Learning. All Rights Reserved. May not be scanned, copied or duplicated, or posted to a publicly accessible website, in whole or in part.
734 Chapter 8 Integration Techniques and Improper Integrals

27. dv = csc 2 x dx   csc


2
v = x dx = − cot x
u = x  du = dx

 x csc x dx = uv −  v du
2

= x( − cot x) − (− cot x)dx


= − x cot x + ln sin x + C

28. u = t , du = dt , dv = csc t cot t dt , v = −csc t

 t csc t cot t dt = −t csc t +  csc t dt = −t csc t − ln csc t + cot t + C

29. Use integration by parts three times.


(1) u = x3 , du = 3x 2 dx, dv = sin x dx, v = −cos x

x sin dx = − x3 cos x + 3 x 2 cos x dx


3

(2) u = x 2 , du = 2 x dx, dv = cos x dx, v = sin x

x
3
( )
sin x dx = − x3 cos x + 3 x 2 sin x − 2 x sin x dx = − x3 cos x + 3 x 2 sin x − 6 x sin x dx

(3) u = x, du = dx, dv = sin x dx, v = −cos x

x
3
sin x dx = − x3 cos x + 3 x 2 sin x − 6 − x cos x + (  cos x dx)
3 2
= − x cos x + 3 x sin x + 6 x cos x − 6 sin x + C
= (6 x − x3 ) cos x + (3x 2 − 6) sin x + C

30. Use integration by parts twice.


(1) u = x 2 , du = 2 x dx, dv = cos x dx, v = sin x

x cos x dx = x 2 sin x − 2  x sin x dx


2

(2) u = x, du = dx, dv = sin x dx, v = −cos x

x
2
(
cos x dx = x 2 sin x − 2 − x cos x +  cos x dx) = x 2 sin x + 2 x cos x − 2 sin x + C

31. dv = dx  v =  dx = x
1
u = arctan x  du = dx
1 + x2
x
 arctan x dx = x arctan x − 1 + x2
dx

1
= x arctan x − ln (1 + x 2 ) + C
2

32. dv = dx  v =  dx = x
1
u = arccos x  du = − dx
1 − x2
 x 
4  arccos x dx = 4 x arccos x +  dx 
2
 1− x 

(
= 4 x arccos x − 1 − x2 + C)

© 2018 Cengage Learning. All Rights Reserved. May not be scanned, copied or duplicated, or posted to a publicly accessible website, in whole or in part.
Section 8.2 Integration by Parts 735

33. Use integration by parts twice.


(1) dv = e −3 x dx  e dx = − 13 e −3 x
−3 x
v =
u = sin 5 x  du = 5 cos 5 x dx

e
−3 x
(
sin 5 x dx = sin 5 x − 13 e −3 x − )  (− 13 e ) 5 cos x dx = − 13 e
−3 x −3 x
sin 5 x + 5
3 e
−3 x
cos 5 x dx

(2) dv = e −3 x dx  e dx = − 13 e −3 x
−3 x
v =
u = cos 5 x  du = −5 sin 5 x dx

e
−3 x
 (
sin 5 x dx = − 13 e −3 x sin 5 x + 53  − 13 e −3 x cos 5 x −  (− 13 e )(−5 sin 5 x)dx
−3 x

= − 13 e −3 x sin 5 x − 59 e −3 x cos 5 x − 25
e
−3 x
9
sin 5 x dx

(1 + 259 ) e −3 x
sin 5 x dx = − 13 e −3 x sin 5 x − 95 e −3 x cos 5 x

e
−3 x
sin 5 x dx = 9
34 (− 13 e −3 x
)
sin 5 x − 95 e −3 x cos 5 x + C = − 34
3 −3 x
e sin 5 x − 5 −3 x
34
e cos 5 x + C

34. Use integration by parts twice.


(1) dv = e 4 x dx  e 1 4x
4x
v = dx = 4
e
u = cos 2 x  du = − 2 sin 2 x dx

e 1 e4 x
 14 e (− 2 sin 2 x) dx
4x 4x
cos 2 x dx = 4
cos 2 x −

2
= 1 e 4 x cos 2 x + 1 e 4 x sin 2 x dx
4

(2) dv = e 4 x dx  e 1 4x
4x
v = dx = 4
e
u = sin 2 x  du = 2 cos 2 x dx

e 1 e4 x 1  1 e4 x
 14 e (2 cos 2 x) dx
4x 4x
cos 2 x dx = 4
cos 2 x + 2 4
sin 2 x −
1 4x
cos 2 x + 18 e 4 x sin 2 x − 1
e
4x
= 4
e 4
cos 2 x dx + C

(1 + 14 )e 4x
cos 2 x dx = 1 e4 x
4
cos 2 x + 18 e 4 x sin 2 x + C

e 1 4x 1 4x
4x
cos 2 x dx = 5
e cos 2 x + 10
e sin 2 x + C

35. dv = dx  v = x
1
u = ln x  du = dx
x
y′ = ln x
1
y =  ln x dx = x ln x −  x x  dx = x ln x − x + C = x( −1 + ln x) + C

36. dv = dx  v =  dx = x
x 1 1 2
u = arctan  du = 2 
dx = dx
2 1 + ( x 2)  2  4 + x2

x x 2x x
y =  arctan 2 dx = x arctan − 4 + dx = x arctan − ln ( 4 + x 2 ) + C
2 x2 2

© 2018 Cengage Learning. All Rights Reserved. May not be scanned, copied or duplicated, or posted to a publicly accessible website, in whole or in part.
736 Chapter 8 Integration Techniques and Improper Integrals

37. Use integration by parts twice.


1 −1 2 2
v =  (3 + 5t ) dt = (3 + 5t )
12
(1) dv = dt 
3 + 5t 5
u = t2  du = 2t dt

t2 2 2
dt = t 2 (3 + 5t ) −  (3 + 5t ) 2t dt
12 12
 3 + 5t 5 5
2 4
= t 2 (3 + 5t ) −  t (3 + 5t ) dt
12 12

5 5
2
(2) dv = (3 + 5t )  (3 + 5t ) (3 + 5t )
12 12 32
dt  v = dt =
15
u = t  du = dt
2
t 2 4 2 2 
dt = t 2 (3 + 5t ) t (3 + 5t ) −  15 (3 + 5t )
12 32 32
 3 + 5t 5
− 
5 15
dt 

2 8 8
= t 2 (3 + 5t ) t (3 + 5t ) + (3 + 5t )3 2 dt
12 32

75 

5 75
2 8 16
= t 2 (3 + 5t ) t (3 + 5t ) + (3 + 5t ) + C
12 32 52

5 75 1875

=
2
1875
(
3 + 5t 375t 2 − 100t (3 + 5t ) + 8(3 + 5t )
2
)+C
2
= 3 + 5t ( 25t 2 − 20t + 24) + C
625

38. Use integration by parts twice.

 ( x − 3) (x − 3)
12 2 32
(1) dv = x − 3 dx  v = dx = 3
u = x2  du = 2 x dx

(x − 3)  23 ( x − 3) 2 x dx
32 32
x 2 x2
2
x − 3 dx = 3

( )
2 x2 x − 3 3 2 − 4
( x − 3) x dx
32
3
= 3

(2) dv = ( x − 3) dx   ( x − 3) (x − 3)
32 32 2 52
v = dx = 5
u = x  du = dx
4 2
(x − 3) (x − 3)  52 ( x − 3) dx
32 52 52
x 2 2
2
x − 3 dx = x − 3 5
x −
3 
8 2  + C
(x − 3) (x − 3) (x − 3)
2 2 32 8 52 72
= x − x + 15  7
3 15 
= 2
35
(x − 3)
32
(5 x 2 + 12 x + 24) + C

y dy
39. (a) (b) = x y cos x, (0, 4)
dx
dy
8  y
=  x cos x dx
6

y  x cos x dx (u = x, du = dx, dv = cos x dx, v = sin x)


−1 2
dy =
2
2 y1 2 = x sin x −  sin x dx = x sin x + cos x + C 6
x
−4 −2 2 4
(0, 4): 2(4) 12
= 0+1+C  C = 3
2 y = x sin x + cos x + 3 −6 6

−2

© 2018 Cengage Learning. All Rights Reserved. May not be scanned, copied or duplicated, or posted to a publicly accessible website, in whole or in part.
Section 8.2 Integration by Parts 737

y
40. (a)
4

x
−6 4

−4

dy  18 
(b) = e − x 3 sin 2 x,  0, − 
dx  37 

e
−x 3
y = sin 2 x dx

Use integration by parts twice.


(1) u = sin 2 x, du = 2 cos 2 x
dv = e − x 3 dx, v = −3e − x 3

e  6e
−x 3
sin 2 x dx = −3e − x 3 sin 2 x + −x 3
cos 2 x dx

(2) u = cos 2 x, du = −2 sin 2 x


dv = e − x 3 dx, v = −3e − x 3

 e sin 2 x dx
−x 3
(
= −3e − x 3 sin 2 x + 6 −3e− x 3 cos 2 x −  6e
−x 3
sin 2 x dx + C )
37  e − x 3 sin 2 x dx = −3e − x 3 sin 2 x − 18e − x 3 cos 2 x + C

1
y = e
−x 3
sin 2 x dx =
37
(−3e−x 3 sin 2 x − 18e−x 3 cos 2 x) + C 4

 −18  −18 1
 0, : = [0 − 18] + C  C = 0 −8 4
 37  37 37
−1 − x 3
y =
37
(3e sin 2 x + 18e−x 3 cos 2 x) −4

dy x 43. u = x, du = dx, dv = e x 2 dx, v = 2e x 2


41. = e x 8 , y ( 0) = 2
dx y
 xe  2e
x 2
dx = 2 xe x 2 − x 2
dx
10

= 2 xe x 2 − 4e x 2 + C
So,
3 3
 0 xe dx = 2 xe x 2 − 4e x 2 
− 10 10 x 2
0
−2

= (6e3 2 − 4e3 2 ) − ( −4)


dy x
42. = sin x, y(0) = 4 = 4 + 2e3 2 ≈ 12.963
dx y
8 (0, 4)

−5 10

−2

© 2018 Cengage Learning. All Rights Reserved. May not be scanned, copied or duplicated, or posted to a publicly accessible website, in whole or in part.
738 Chapter 8 Integration Techniques and Improper Integrals

44. Use integration by parts twice. 1


2
(1) u = x , du = 2 x dx, dv = e −2 x
dx,
46. dv = sin 2 x dx  v =  sin 2 x dx = −
2
cos 2 x

u = x  du = dx
1
v = − e −2 x
2 −1 1
 x sin 2 x dx =
2
x cos 2 x +  cos 2 x dx
2
1  1 −2 x 
x e   − 2 e
2 −2 x
dx = − x 2e −2 x −  2 x dx −1 1
2  = x cos 2 x + sin 2 x + C
2 4
1
= − x 2 e −2 x +  xe
−2 x
dx 1
2 = (sin 2 x − 2 x cos 2 x ) + C
4
1
(2) u = x, du = dx, dv = e −2 x dx, v = − e −2 x So,
2
π
π 1  π
1  1 1  0 x sin 2 x dx =  (sin 2 x − 2 x cos 2 x) = − .
x e dx = − x 2e−2 x +  − xe −2 x −  − e −2 x dx 
2 −2 x
2 4 0 2
 2 2 
1 2 −2 x 1 −2 x 1 −2 x 1
= − x e − xe − e + C
2 2 4 47. u = arccos x, du = − dx, dv = dx, v = x
1 − x2
−2 x  1 2 1 1
= e − x − x −  x
 2 2 4
 arccos x dx = x arccos x +  1 − x2
dx
So,
2 = x arccos x − 1 − x2 + C
2   1 1 1 
0 x e
2 −2 x
dx = e −2 x  − x 2 − x −  So,
  2 2 4  0
12 12
 1  1 0 arccos x =  x arccos x − 1 − x2 
= e −4  −2 − 1 −  − −   0
 4  4
1 1 3
−13 1 = arccos  − +1
= + ≈ 0.190 2  2 4
4e 4 4
π 3
= − + 1 ≈ 0.658.
1 6 2
45. u = x, du = dx, dv = cos 2 x dx, v = sin 2 x
2
x2
1 1
 x cos 2 x dx = 2 x sin 2 x −  2 sin 2 x dx
48. dv = x dx  v =  x dx =
2
2x
1 1 u = arcsin x 2  du = dx
= x sin 2 x + cos 2 x + C 1 − x4
2 4
So, x2 x3
 x arcsin x 
2
dx = arcsin x 2 − dx
π 4 2 1 − x4
π 4 1 1 
0 x cos 2 x dx =  x sin 2 x + cos 2 x
2 4 0 =
x2 1
arcsin x 2 + ( 2)(1 − x 4 ) + C
12

2 4
π   1
=  (1) + 0  −  0 + 
8   4 =
1 2
2
(
x arcsin x 2 + 1 − x 4 + C )
π 1
= − ≈ 0.143 1 1 2 1
8 4 So,  0 x arcsin x
2
dx = x arcsin x 2 + 1 − x4 
2 0
1
= (π − 2).
4

© 2018 Cengage Learning. All Rights Reserved. May not be scanned, copied or duplicated, or posted to a publicly accessible website, in whole or in part.
Section 8.2 Integration by Parts 739

49. Use integration by parts twice.


(1) dv = e x dx  e dv = e x dx  e
x
v = dx = e x (2) v = x
dx = e x
u = sin x  du = cos x dx u = cos x  du = −sin x dx

 e sin x dx e e
x x x
= e sin x − cos x dx = e sin x − e x cos x −
x x
sin x dx

2  e x sin x dx = e x (sin x − cos x)


ex
e
x
sin x dx = (sin x − cos x) + C
2
1
1 ex  e 1 e(sin 1 − cos 1) + 1
 0 e sin x dx =  2 (sin x − cos x) = 2 (sin 1 − cos 1) + 2 =
x
So, ≈ 0.909.
0
2

2x
50. u = ln ( 4 + x 2 ), du = dx, dv = dx, v = x
4 + x2
2x2
 ln(4 + x ) dx = x ln ( 4 + x 2 ) − 4 +
2
dx
x2
 4 
= x ln ( 4 + x 2 ) − 2 1 −  dx
 4 + x2 
 4 x
= x ln ( 4 + x 2 ) − 2 x − arctan  + C
 2 2
x
= x ln ( 4 + x 2 ) − 2 x + 4 arctan + C
2
1
 x   1 
 0 ln(4 + x ) dx = x ln(4 + x ) − 2 x + 4 arctan 2  0 =  ln 5 − 2 + 4 arctan 2   ≈ 1.464.
1
2 2
So,

x2 1
51. dv = x dx, v = , u = arcsec x, du = dx
2 x x2 − 1
x2 x2 2 x2 1 2x x2 1
 x arcsec x dx =
2
arcsec x − x 2
x −1
dx =
2
arcsec x − 
4 2
x −1
dx =
2
arcsec x −
2
x2 − 1 + C

So,
4
4  x2 1   15   2π 3 15 3 2π
2 x arcsec x dx =  arcsec x −
2 2
x 2 − 1 =  8 arcsec 4 −
2 
 − 
2   3
−  = 8 arcsec 4 −
2  2
+
2

3
≈ 7.380.

1
52. u = x, du = dx, dv = sec 2 2 x dx, v = tan 2 x
2
1 1 1 1
 x sec  2 tan 2 x dx
2
2 x dx = x tan 2 x − = x tan 2 x + ln cos 2 x + C
2 2 4
So,
π 8
π 8 1 1  π 1  2 π 1
0 x sec2 2 x dx =  x tan 2 x + ln cos 2 x  = (1) + ln  = − ln( 2) ≈ 0.1097.
 2 4 0 16 4  2  16 8

53. x e
2 2x
dx = x 2 ( 12 e ) − (2 x)( 14 e ) + 2( 18 e ) + C
2x 2x 2x
Alternate signs u and its derivatives v′ and its antiderivatives
1 2 2x 1 2x 1 2x
= 2
xe − 2
xe + 4
e + C + x 2
e2 x
= 1 2x
4
e (2 x 2 − 2 x + 1) + C – 2x 1 2x
2
e

+ 2 1 2x
4
e

– 0 1 2x
8
e

© 2018 Cengage Learning. All Rights Reserved. May not be scanned, copied or duplicated, or posted to a publicly accessible website, in whole or in part.
740 Chapter 8 Integration Techniques and Improper Integrals

 x2 
 (1 − x)(e + 1) dx = (1 − x)( − e − x + x ) − ( −1) e − x +
−x
54.  + C
 2
 x2 
= ( − e − x + x + xe − x − x 2 ) +  e − x +  + C
 2
x2
= − + x + xe − x + C
2
Alternate signs u and its derivatives v′ and its antiderivatives
+ (1 − x) e− x + 1
– −1 − e− x + x
x2
+ 0 e− x +
2

 ( x + 2) sin x dx = ( x + 2) ( − cos x) − 2( x + 2)( − sin x) + 2(cos x) + C


2 2
55.

= − cos x ( x + 2) + 2 sin x ( x + 2) + 2 cos x + C


2

Alternate signs u and its derivatives v′ and its antiderivatives

( x + 2)
2
+ sin x
– 2( x + 2 ) − cos x
+ 2 − sin x
– 0 cos x

56. x
3
cos 2 x dx = x3 ( 12 sin 2 x) − 3x (− 14 cos 2 x) + 6 x(− 18 sin 2 x) − 6(161 cos 2 x) + C
2

1 3 3 2 3 3
= 2
x sin 2 x + 4
x cos 2 x − 4
x sin 2 x − 8
cos 2 x + C
= 1
8 (4 x 3
sin 2 x + 6 x cos 2 x − 6 x sin 2 x − 3 cos 2 x) + C
2

Alternate signs u and its derivatives v′ and its antiderivatives


3
+ x cos 2x
1
– 3x 2 2
sin 2x

+ 6x − 14 cos 2x

– 6 − 18 sin 2x
1
+ 0 16
cos 2x

1 1
 (6 + x) 4 x + 9 dx = (6 + x) ( 4 x + 9) − (4 x + 9) + C
32 52
57.
 
6 60
1
(4 x + 9) [60 + 10 x − 4 x − 9] + C
32
=
60
1
(4 x + 9) (51 + 6 x) + C
32
=
60
1
(4 x + 9) (17 + 2 x) + C
32
=
20

Alternate signs u and its derivatives v′ and its antiderivatives

(4 x + 9)
12
+ 6+ x

(4 x + 9)
1 32
– 1 6

(4 x + 9)
1 52
+ 0 60

© 2018 Cengage Learning. All Rights Reserved. May not be scanned, copied or duplicated, or posted to a publicly accessible website, in whole or in part.
Section 8.2 Integration by Parts 741

58.  x ( x − 2) dx
2 32
= 2 2
5
x (x − 2)
52
− 8
35
x (x − 2)
72
+ 16
315 (x − 2)
92
+C = 2
315 (x − 2)
52
(35x2 + 40 x + 32) + C

Alternate signs u and its derivatives v′ and its antiderivatives

(x − 2)
32
+ x2

(x − 2)
2 52
– 2x 5

(x − 2)
4 72
+ 2 35

(x − 2)
8 92
– 0 315

x x  u 2 = x  2u du = dx
3
59. Answers will vary. Sample answer: sin x dx. 63. u =

It takes three applications of integration by parts for the  sin x dx =  sin u(2u du) = 2 u sin u du
term x3 to become a constant. Integration by parts:
Other possible answers:  x3 cos x dx,  x3e x dx w = u, dw = du , dv = sin u du , v = −cos u

60. In order for the integration by parts technique to be


2  u sin u du = 2 −u cos u + (  cos u du )
efficient, you want dv to be the most complicated portion = 2( −u cos u + sin u ) + C
of the integrand and you want u to be the portion of the
integrand whose derivative is a function simpler than u.
= 2− ( x cos x + sin )
x + C

Suppose you let u = sin x and dv = x dx. Then


64. u = x 2 , du = 2 x dx
2
du = cos x dx and v = x 2. So
 2x
3
cos( x 2 ) dx = x
2
cos( x 2 )( 2 x) dx =  u cos u du
2 2
x x
 x sin x dx = uv −  v du = 2 sin x −  2 cos x dx, Integration by parts:
which is a more complicated integral than the original w = u , dw = du , dv = cos u du , v = sin u
one.
 u cos u du = u sin u −  sin u du
61. (a) No = u sin u + cos u + C
Substitution
= x 2 sin ( x 2 ) + cos( x 2 ) + C
(b) Yes
u = ln x, dv = x dx
65. u = x 2 , du = 2 x dx
(c) Yes 1 x2 4 1
5 x2
u = x 2 , dv = e−3 x dx
x e dx =
2
e x 2 x dx =  eu u 2 du
2
(d) No Integration by parts twice.
Substitution (1) w = u 2 , dw = 2udu, dv = eu du , v = eu
(e) Yes. Let u = x and 1 u 2 1
 e u du = u 2eu −  2ueu du 
1 2 2 
dv = dx.
x +1 1
= u 2eu −  ueu du
2
(Substitution also works. Let u = x + 1. )
(f ) No (2) w = u, dw = du, dv = eu du, v = eu
Substitution 1 u 2
2
1
e u du = u 2eu − ueu −  eu du
2
( )
62. (a) The slope of f at x = 2 is approximately 1.4
1 2 u
because f ′( 2) ≈ 1.4. = u e − ueu + eu + C
2
(b) f ′ < 0 on (0, 1)  f is decreasing on (0, 1). 1 4 x2 2 2
= x e − x 2e x + e x + C
f ′ > 0 on (1, ∞)  f is increasing on (1, ∞). 2
2
ex
=
2
( x 4 − 2 x 2 + 2) + C

© 2018 Cengage Learning. All Rights Reserved. May not be scanned, copied or duplicated, or posted to a publicly accessible website, in whole or in part.
742 Chapter 8 Integration Techniques and Improper Integrals

66. Let u = 2 x , u 2 = 2 x, 2u du = 2dx.

e
2x
dx =  eu (u du )

Now use integration by parts.


dv = eu du  v =  eu du = eu
w = u  dw = du

e e
2x
dx = ueu − u
du
u u
= ue − e + C
2x 2x
= 2x e −e +C

x
 (4 + x )
−1 2
67. (a) dv = dx  v = 2
x dx = 4 + x2
4 + x2
u = x2  du = 2 x dx
3
x
 4 + x2
dx = x 2 4 + x 2 − 2  x 4 + x 2 dx

2
(4 + x 2 ) + C = 13 4 + x 2 ( x 2 − 8) + C
32
= x2 4 + x2 −
3
1
(b) u = 4 + x 2  x 2 = u − 4 and 2 x dx = du  x dx = du
2
x3 x2 u − 4 1
 4+ x 2
dx = 
4 + x 2
x dx =    du
u 2
1 1 2 
=  (u1 2 − 4u −1 2 ) du =  u 3 2 − 8u1 2  + C
2 2 3 
1 12
= u (u − 12) + C
3
1 1
= 4 + x 2 ( 4 + x 2 ) − 12 + C = 4 + x 2 ( x 2 − 8) + C
3 3

 (4 − x) dx
12
68. (a) dv = 4 − x dx  v =

= − 23 ( 4 − x)
32

u = x  du = dx

4 − x dx = − 23 x( 4 − x)  (4 − x) dx
32 32
x + 2
3

= − 23 x( 4 − x) (4 − x)
32 4 52
− 15
+C

(4 − x) 5 x + 2( 4 − x) + C = − 15 ( 4 − x) (3 x + 8) + C
2 32 2 32
= − 15

(b) u = 4 − x  x = 4 − u and dx = −du

x 4 − x dx = −  ( 4 − u ) u du

= −  ( 4u1 2 − u 3 2 ) du
= − 83 u 3 2 + 52 u 5 2 + C
( )
2 u 3 2 20 − 3u + C
= − 15

( 4 − x) 20 − 3( 4 − x) + C
2 32
= − 15

( 4 − x) (3 x + 8) + C
2 32
= − 15

© 2018 Cengage Learning. All Rights Reserved. May not be scanned, copied or duplicated, or posted to a publicly accessible website, in whole or in part.
Section 8.2 Integration by Parts 743

69. n = 0:  ln x dx = x(ln x − 1) + C
x2
n = 1:  x ln x dx (2 ln x − 1) + C
=
4
x3
n = 2:  x 2 ln x dx = (3 ln x − 1) + C
9
x4
n = 3:  x3 ln x dx = (4 ln x − 1) + C
16
x5
n = 4:  x 4 ln x dx = (5 ln x − 1) + C
25
xn +1
2 (
In general,  x n ln x dx =  n + 1)ln x − 1 + C.
( n + 1)

 e dx = e + C
x x
70. n = 0:

n = 1:  xe x dx = xe x − e x + C = xe x −  e x dx

n = 2:  x 2e x dx = x 2e x − 2 xe x + 2e x + C = x 2e x − 2  xe x dx
n = 3:  x3e x dx = x3e x − 3x 2e x + 6 xe x − 6e x + C = x3e x − 3 x 2e x dx

n = 4:  x 4e x dx = x 4e x − 4 x3e x + 12 x 2e x − 24 xe x + 24e x + C = x 4e x − 4 x3e x dx


In general,  x n e x dx = x n e x − n  x n −1e x dx.

71. dv = sin x dx  v = −cos x x n +1


n −1
73. dv = x n dx  v =
u = x n
 du = nx dx n +1
1
x sin x dx = − x n cos x + n  x n −1 cos x dx
n
u = ln x  du = dx
x
n +1
72. dv = cos x dx  v = sin x x xn
 x ln x dx = n + 1 ln x −  n + 1 dx
n

u = x n
 du = nx n −1 dx
x n +1 x n +1
= ln x − +C
x cos x dx = x n sin x − n  x n −1 sin x dx
n
n +1 (n + 1)2
x n +1
= ( n + 1)ln x − 1 + C
(n + 1)
2

1 ax
74. dv = eax dx  v =
e
a
u = xn  du = nx n −1dx
x n e ax n
x −  x n −1 e ax dx
n
e ax dx =
a a
75. Use integration by parts twice.
1 ax 1 ax
(1) dv = e ax dx  e v = (2) dv = e ax dx  e v =
a a
u = sin bx  du = b cos bx dx u = cos bx  du = −b sin bx dx
eax sin bx b
e −  e ax cos bx dx
ax
sin bx dx =
a a
eax sin bx b  e ax cos bx b  e ax sin bx b b2
= −  +  eax sin bx dx  = − 2 eax cos bx − 2  e ax sin bx dx
a a a a  a a a
 b2  e ax ( a sin bx − b cos bx)
Therefore, 1 + 2   e ax sin bx dx =
 a  a2
e ax ( a sin bx − b cos bx)
 e sin bx dx =
ax
+ C.
a 2 + b2

© 2018 Cengage Learning. All Rights Reserved. May not be scanned, copied or duplicated, or posted to a publicly accessible website, in whole or in part.
744 Chapter 8 Integration Techniques and Improper Integrals

76. Use integration by parts twice.


 1  1
(1) dv = eax dx  v = eax (2) dv = e dx 
ax
v = e ax
a a
 
 u = cos bx  du = −b sin bx  u = sin bx  du = b cos bx
e ax cos bx b e ax cos bx b  e ax sin bx b ax 
e +  e ax sin bx dx = −  e cos bx dx 
ax
cos bx dx = + 
a a a a a a 
e ax cos bx be ax sin bx b 2 ax
= + − 2  e cos bx dx
a a2 a
 b2  e ax ( a cos bx + b sin bx)
Therefore, 1 + 2   e ax cos bx dx =
 a  a2
e ( a cos bx + b sin bx)
ax

 e cos bx dx =
ax
+ C.
a 2 + b2

77. n = 2 (Use formula in Exercise 67.)

x sin x dx = − x 2 cos x + 2  x cos x dx


2

= − x 2 cos x + 2  x sin x − sin x dx ( Use formula in Exercise 68; ( n = 1).)


 
= − x 2 cos x + 2 x sin x + 2 cos x + C

78. n = 2 (Use formula in Exercise 68.)

x
2
cos x dx = x 2 sin x − 2  x sin x dx, ( Use formula in Exercise 67.) ( n = 1)

(
= x 2 sin x − 2 − x cos x +  cos x dx) = x 2 sin x + 2 x cos x − 2 sin x + C

79. n = 5 (Use formula in Exercise 69.)


x6 x6
2(
x
5
ln x dx = −1 + 6 ln x) + C = (−1 + 6 ln x) + C
6 36

80. n = 3, a = 2 (Use formula in Exercise 70 three times.)

x 3e 2 x 3
x e
3 2x
dx = −  x 2e 2 x dx, (n = 3, a = 2)
2 2
x 3e 2 x 3  x 2e 2 x 
= −  −  xe
2x
dx, (n = 2, a = 2)
2 2 2 
x 3e 2 x 3 x 2e 2 x 3  xe 2 x 1 
= − +  −  e 2 x dx
2 4 2 2 2 
x 3e 2 x 3 x 2e 2 x 3xe 2 x 3e2 x
= − + − + C, (n = 1, a = 2)
2 4 4 8
e2 x
=
8
(4 x3 − 6 x2 + 6 x − 3) + C
81. a = − 3, b = 4 (Use formula in Exercise 71.)

e − 3 x ( − 3 sin 4 x − 4 cos 4 x )
e
−3 x
sin 4 x dx = + C
(− 3)2 + ( 4)
2

− e − 3 x (3 sin 4 x + 4 cos 4 x)
= + C
25

© 2018 Cengage Learning. All Rights Reserved. May not be scanned, copied or duplicated, or posted to a publicly accessible website, in whole or in part.
Section 8.2 Integration by Parts 745

82. a = 2, b = 3 (Use formula in Exercise 72.) 85. 1

e 2 x ( 2 cos 3 x + 3 sin 3 x)
e
2x
cos 3 x dx = +C
13
1 0 1.5
83. 0

1
−1 7
A = 0 e
−x
sin (π x) dx
1
 e − x ( −sin π x − π cos π x) 
−1 =  
 1 + π2 0
dv = e − x dx  v =  e − x dx = − e − x
1 π 
u = 2x  du = 2 dx
=  + π
1 + π2 e 

 2 xe
−x
dx = 2 x( −e − x ) −  − 2e
−x
dx =
π 1 
 + 1
1 + π2e 
= −2 xe − x − 2e − x + C
3 3
≈ 0.395 (See Exercise 71.)
 0 2 xe dx = −2 xe − x − 2e − x 
−x
A =
0
86. 80
= ( −6e −3 − 2e −3 ) − ( −2)
= 2 − 8e −3 ≈ 1.602

84. 100 0 4
0

x4
dv = x3 dx  x
3
v = dx =
4
1
0 3 u = ln x  du = dx
0 x
2 1 1 2
x4 x4  1 
 0 10 xe  0 xe
3x 3x
A = dx = dx
x 
3
ln x dx = ln x −  dx 
10 4 4 x 
1 3x x4 x3
dv = e3 x dx  v =  e3 x dx = e = ln x −  dx
3 4 4
u = x  du = dx x4 x4
= ln x − +C
1 1  x 3x 1  4 16
10   3e
xe3 x dx = e − 3x
dx
10  3  3
3  x4 x4 
=
x 3x
e −
1 3x
e +C A = 1 x3 ln x dx =  ln x −
4

16 1
30 90
2  81 81  1
x 1 3x  =  ln 3 −  +
A =  e3 x − e 4 16  16
 30 90  0
81
1 1 6 1 = ln 3 − 5 ≈ 17.247
=  e6 − e  + 4
 15 90  90
1
=
90
(5e6 + 1) ≈ 22.424

© 2018 Cengage Learning. All Rights Reserved. May not be scanned, copied or duplicated, or posted to a publicly accessible website, in whole or in part.
746 Chapter 8 Integration Techniques and Improper Integrals

87. (a) dv = dx  v = x
1
u = ln x  du = dx
x
e
ln x dx = [ x ln x − x]1 = 1 ( Use integration by parts once.)
e
A = 1
(b) R( x) = ln x, r ( x) = 0
e
(ln x)
2
V = π dx
1
e
= π  x(ln x) − 2 x ln x + 2 x ( Use integration by parts twice, see Exercise 3.)
2
 1
= π (e − 2) ≈ 2.257
y

(e, 1)
1

x
1 2 3

(c) p( x) = x, h( x) = ln x
e
e  x2 
V = 2π  x ln x dx = 2π  ( −1 + 2 ln x)
1
4 1

=
(e 2 + 1)π
≈ 13.177 (See Exercise 73.)
2
e

(d) x =
 1 x ln x dx =
e2 + 1
≈ 2.097
1 4
1 e
1 (ln x) dx e − 2
2

y = 2 = ≈ 0.359
1 2
 e2 + 1 e − 2 
(x, y) =  ,  ≈ ( 2.097, 0.359)
 4 2 

88. y = x sin x, 0 ≤ x ≤ π
π
(a) A = 0 x sin x dx

= − x cos x +  cos x dx ( Excercise 71)


π
= − x cos x + sin x]0
= − π ( −1) = π

© 2018 Cengage Learning. All Rights Reserved. May not be scanned, copied or duplicated, or posted to a publicly accessible website, in whole or in part.
Section 8.2 Integration by Parts 747

π π
 0 π ( x sin x)
2
(b) V = dx = π  x 2 sin 2 x dx
0

1 − cos 2 x 1 sin 2 x
Let u = x 2 , du = 2 x dx, dv = sin 2 x dx = dx, v = x − .
2 2 4
2 1 sin 2 x  1 sin 2 x 
 x sin x dx = x  2 x − 4  −   2 x − 4 (2 x dx)
2 2

1 x 2 sin 2 x  x sin 2 x 
= x3 − −   x2 −  dx
2 4  2 
1 3 x 2 sin 2 x x3 x sin 2 x
= x − − +  dx
2 4 3 2
1 1 1
= x 3 − x 2 sin 2 x + (sin 2 x − 2 x cos 2 x ) + C ( Integration by Parts)
6 4 8
π
π 1 1 1  1 1
V = π  x 2 sin 2 x dx = π  x 3 − x 2 sin 2 x + (sin 2 x − 2 x cos 2 x) = π 4 − π 2
0
6 4 8 0 6 4
π
2π x( x sin x )dx = 2π 2 cos x + 2 x sin x − x 2 cos x = 2π (π 2 − 4) = 2π 3 − 8π
π
(c) V = 0 0

π π
(d) m = 0 x sin ( x ) dx = [sin x − x cos x]0 = π
y

1 π
Mx = 
0 2
( x sin x)2 dx 3

2
1 1 1 
=  π 3 − π  (See part (a ).) 1
2 6 4 
x
1 3 1 π π
= π − π 2
12 8 −1

π
My =  0 x( x sin x) dx = π2 − 4 (See part (b).)

x =
My
=
π2 − 4
≈ 1.8684, y =
Mx
=
(1 12)π 3 − (1 8)π = 1 π 2 − 1 ≈ 0.6975
m π m π 2 8

89. In Example 6, you showed that the centroid of an equivalent region was (1, π 8). By symmetry, the centroid of this region is
(π 8, 1). You can also solve this problem directly.
1
1 π  π 
A =  0  2 − arcsin x  dx =  2 x − x arcsin x − 1 − x2  (Example 3)
0
π π 
=  − − 0  − ( −1) = 1
2 2 
My 1 π  π Mx 1 (π 2) + arcsin x  π 
x =
A
=  0 x 2 − arcsin x  dx = ,
 8
y =
A
= 0 2
 − arcsin x  dx = 1
 2 

© 2018 Cengage Learning. All Rights Reserved. May not be scanned, copied or duplicated, or posted to a publicly accessible website, in whole or in part.
748 Chapter 8 Integration Techniques and Improper Integrals

90. f ( x) = x 2 , g ( x ) = 2 x
f ( 2) = g ( 2) = 4, f ( 4) = g ( 4) = 16
4
 x3 1 x  64 16   8 4  56 12
 2 (x − 2 x ) dx =  −
4
2
m = 2  =  − − −  = − ≈ 1.3543
 3 ln 2 2  3 ln 2   3 ln 2  3 ln 2

1
 2 2(x + 2 x )( x 2 − 2 x ) dx
4
2
Mx = y

(4, 16)
16
1 4
=  ( x 4 − 22 x ) dx
2
f(x) = x
12
2 2
4 8
1  x5 22 x  g(x) = 2 x
=  −  4
2 5 2 ln 2  2 (2, 4)

x
1  1024 128   32 8  1 2 3 4
=  −  − − 
2  5 ln 2   5 ln 2 
496 60
= − ≈ 12.6383
5 ln 2
4 56 12
 2 x x − 2 x  dx = −
2
My = + ≈ 4.1855
(ln 2)
2
ln 2

 My Mx 
(x, y) =  ,  ≈ (3.0905, 9.3318)
 m m 

1 π
e −4t (cos 2t + 5 sin 2t ) dt
π 0
91. Average value =
π
1  −4t  −4 cos 2t + 2 sin 2t  −4t  −4 sin 2t − 2 cos 2t 
= e  + 5e  (From Exercises 71 and 72)
π   20   20

 0
7
=
10π
(1 − e−4π ) ≈ 0.223
2 2
92. (a) Average =  1 (1.6t ln t + 1) dt = 0.8t 2 ln t − 0.4t 2 + t  = 3.2(ln 2) − 0.2 ≈ 2.018
1

4 4
(b) Average =  3 (1.6t ln t + 1) dt = 0.8t 2 ln t − 0.4t 2 + t  = 12.8(ln 4) − 7.2(ln 3) − 1.8 ≈ 8.035
3

93. c(t ) = 100,000 + 4000t , r = 5% = 0.05, t1 = 10


10 10
 0 (100,000 + 4000t ) e dt = 4000  (25 + t )e−0.05t dt
−0.05t
P =
0

100 −0.05t
Let u = 25 + t , dv = e −0.05t dt , du = dt , v = − e .
5
  100 −0.05t 
10
100 10 −0.05t    100 −0.05t 
10
10,000 −0.05t  
10
P = 4000( 25 + t ) − e  +  e dt  = 4000( 25 + t ) − e  −  e   ≈ $931,265
  5  0 5 0    5  0  25  0 

© 2018 Cengage Learning. All Rights Reserved. May not be scanned, copied or duplicated, or posted to a publicly accessible website, in whole or in part.
Section 8.2 Integration by Parts 749

94. c(t ) = 1000 + 120t , r = 2% = 0.02, t1 = 30


30
0 (1000 + 120t )e dt
− 0.02 t
P =

Let u = 1000 + 120t , dv = e− 0.02t dt , du = 120 dt , v = − 50e− 0.02t


30
P = (1000 + 120t )( − 50e− 0.02t ) −
30
0 − 50e− 0.02t (120 dt )
0
30
= − 230,000e − 0.6 + 50,000 − 300,000e− 0.02t 
0

= − 230,000e − 0.6 + 50,000 − 300,000e− 0.6 + 300,000


= 350,000 − 530,000e − 0.6
≈ $59,130

π  x 1 
π
π π 2π −( 2π n), if n is even
95.  −π x sin nx dx = − cos nx + 2 sin nx = − cos π n − cos( −π n) = − cos π n = 
 n n  −π n n n ( 2π n), if n is odd

π
π  x2 2x 2 
 − π x cos nx dx =  n sin nx + n 2 cos nx − n3 sin nx
2
96.
−π
2π 2π
= 2 cos nπ + 2 cos( − nπ )
n n

= 2 cos nπ
n
( 4π n 2 ), if n is even

= 
−( 4π n 2 ), if n is odd

=
(−1)n 4π
n2

 nπ  2  nπ 
97. Let u = x, dv = sin  x  dx, du = dx, v = − cos x .
 2  nπ  2 
1
1  nπ   −2 x  nπ  2 1  nπ 
I1 =  0 x sin 2 x  dx =  nπ cos 2 x  + nπ  0 cos 2 x  dx
0
1
 nπ   2  
2
2  nπ
= − cos  +   sin  x 
nπ  2   nπ   2  0
2
2  nπ   2   nπ 
= − cos  +   sin  
nπ  2   nπ   2 
 nπ  2  nπ 
Let u = ( − x + 2), dv = sin  x  dx, du = − dx, v = − cos x .
 2  nπ  2 
2
2  nπ   −2( − x + 2)  nπ  2 2  nπ 
I2 = 1 (− x + 2) sin 
 2
x  dx = 
  nπ
cos x  −
 2 1 nπ  1
cos
 2
x  dx

2
 nπ   2   nπ 
2
2
= cos  −   sin  x 
nπ    
2 nπ  2 1
2
2  nπ   2   nπ 
= cos  +   sin  
nπ    
2 nπ  2 
 2  2  nπ   2 
2
 nπ  8h  nπ 
h( I1 + I 2 ) = bn = h   sin   +   sin   = sin  
 nπ   2   nπ  ( nπ )
2
 2   2 

© 2018 Cengage Learning. All Rights Reserved. May not be scanned, copied or duplicated, or posted to a publicly accessible website, in whole or in part.
750 Chapter 8 Integration Techniques and Improper Integrals

π π
98. (a) A = 0 x sin x dx = [sin x − x cos x]0 = π

2π 2π
(b) π x sin x dx = [sin x − x cos x]π = −2π − π = −3π

A = 3π
3π 3π
(c)  2π x sin x dx = [sin x − x cos x]2π = 3π + 2π = 5π

A = 5π
The area between y = x sin x and y = 0 on nπ , ( n + 1)π  is ( 2n + 1)π :
( n + 1)π (n + 1)π
 nπ x sin x dx = [sin x − x cos x]nπ = ±( n + 1)π ± nπ = ±( 2n + 1)π

A = ±( 2n + 1)π = ( 2n + 1)π

99. For any integrable function,  f ( x)dx = C +  f ( x)dx, but this cannot be used to imply that C = 0.

 π
2x 2x π 
100. Use the fact that sin x ≥ on the interval 0, . To see this, note that if g ( x ) = sin x − , then g (0) = g   = 0.
π  2  π 2
Furthermore, the sine function is concave downward on this interval.

(π 2)
r +1
π 2
π 2 π 2 r  x r +1 
Let f ( r ) = 0 x r sin x dx. Then you have f ( r ) < 0 x dx =   = .
 r + 1 0 r +1

(π 2) (π 2)
r+2 r +1
π 2
π 2 r π 2 r  2x   2 xr + 2  2
Furthermore, f ( r ) = 0 x sin x dx > 0 x  dx =   = = .
π  π r + 2  0 π r +2 r + 2

(π 2) (π 2)
r +1 r +1

So, < f (r ) <


r + 2 r +1

(π 2) (π 2)
r +1 r +1
r +1 r +1 r +1
2 2 2
r  < r  f (r ) < r  
π  r + 2 π  π  r +1
r +1
r 2 r
< r  f (r ) <
r + 2 π  r +1

  2 r +1 
Taking limits by the Squeeze Theorem, lim r   f ( r ) = 1.
r →∞  π 
 

 
( )
r +1
 f (r )   r 2 f (r )   2( r + 1)  2
= π
Now analyze the following limit: lim  
r → ∞  f ( r + 1) 
lim
r →∞  π r  = π
( )
r+2
   ( r + 1)r 2 π f ( r + 1)   
 
x r +1
Using integration by parts, let u = cos x, du = − sin x dx, dv = x r dx, v = .
r +1
π 2
π 2 r  xr +1  π 2 − xr +1 1
So, you have 0 x cos x dx =  cos x − 0 sin x dx = f ( r + 1).
r + 1 0 r +1 r +1
Now, consider the limit with c = −1 :
π 2 r
r −1  x sin x dx f ( r ) ( r + 1) 2
0
lim = lim =
π 2
r f ( r + 1) π
0
r →∞ r r →∞
x cos x dx

2
Therefore, c = −1 and L = .
π

© 2018 Cengage Learning. All Rights Reserved. May not be scanned, copied or duplicated, or posted to a publicly accessible website, in whole or in part.
Section 8.3 Trigonometric Integrals 751

Section 8.3 Trigonometric Integrals

 sin  sin  sin 3x sin 3x dx


8 3 2
1. The integral x dx requires more steps. The other 6. 3x dx =
integral can be found by u-substitution
 (1 − cos 3x) sin 3x dx
2
=
(u = sin x, du = cos x dx).
 sin 3x dx − cos 3x(sin 3x dx)
2
=
2. The power of the tangent is odd. So, save a secant- 1 cos3 3x
= − cos 3x + +C
tangent factor, and convert the remaining tangents to 3 9
secants.
 sin  (1 − cos x) cos x sin x dx
3
7. x cos 2 x dx = 2 2
3. Let u = cos x, du = −sin x dx.
 (cos x − cos x) sin x dx
2 4
=
 cos
5
x sin x dx = −  cos5 x ( −sin x) dx
= −  (cos 2 x − cos 4 x ) ( −sin x) dx
cos 6 x
= − +C cos3 x cos5 x
6 = − + + C
3 5
4. Let u = sin 2 x, du = 2 cos 2 x dx.
x 1 x
1 8. Let u = sin , du = cos dx.
 sin 2 x cos 2 x dx = 2  sin 2 x(2 cos 2 x) dx
7 7
3 3 3
3 x  x  2 x
1  sin 8 2 x 
=   +C
 cos 3 dx =   cos 1 − sin  dx
3  3
2 8 
 x  1 x
1 = 3 1 − sin 2  cos  dx
= sin 8 2 x + C  3  3 3
16
 x 1 x
= 3 sin − sin 3  + C
 cos  cos x(1 − sin x) sin x dx  3 3 3
3 4 2 4
5. x sin x dx =
x 3 x
 (sin x − sin x) cos x dx = 3 sin − sin +C
4 6
=
3 3
sin 5 x sin 7 x
= − +C
5 7

 sin 2θ cos 2θ dθ =  (1 − cos 2θ ) cos 2θ sin 2θ dθ


3 2
9.

 (cos 2θ ) − (cos 2θ )  sin 2θ dθ


12 52
=

= − 12  (cos 2θ ) − (cos 2θ )  ( −2 sin 2θ ) dθ


12 52
 
= − 12  23 (cos 2θ ) (cos 2θ )  +C
32 2 72

 7 
= − 13 (cos 2θ ) (cos 2θ )7 2
32 1
+ 7
+C

cos5 t
 cos t (1 − sin t ) (sin t ) dt
−1 22

2
10. dt =
sin t

 (1 − 2 sin t + sin 4 t ) (sin t )


2 −1 2
= cos t dt

 (sin t )  cos t dt
−1 2
− 2(sin t ) + (sin t )
32 72
=

4 2
(sin t ) + (sin t ) + C
52 92
= 2 sin t −
5 9

1 + cos 6 x 1 1  1
11.  cos
2
3 x dx =  dx =  x + sin 6 x  + C = (6 x + sin 6 x) + C
2 2 6  12

© 2018 Cengage Learning. All Rights Reserved. May not be scanned, copied or duplicated, or posted to a publicly accessible website, in whole or in part.
752 Chapter 8 Integration Techniques and Improper Integrals

 1 − cos 12θ  1 − cos 12θ 


 sin 6θ dθ =    dθ
4
12. 
2  2 
1
=
4
(1 − 2 cos 12θ + cos2 12θ ) dθ
1  1 + cos 24θ 
=  1 − 2 cos 12θ +  dθ
4  2 
1 3 1 
4  2
=  − 2 cos 12θ + cos 24θ  dθ
2 
1 3 1 1  3 1 1
=  θ − sin 12θ + sin 24θ  + C = θ − sin 12θ + sin 24θ + C
4 2 6 48  8 24 192

1 + cos 2 x 1 sin 2 x
13. dv = cos 2 x dx = dx  v = x +
2 2 4
u = 8 x  du = 8 dx

 8 x cos x dx = uv −  v du
2

1 sin 2 x  1 sin 2 x 
= 8 x x +  −   2 x + (8 dx)
 2 4  4 
= 4 x 2 + 2 x sin 2 x − 2 x 2 + cos 2 x + C
= 2 x 2 + 2 x sin 2 x + cos 2 x + C

14. Use integration by parts twice.


1 − cos 2 x x sin 2 x 1
dv = sin 2 x dx =  v = − = ( 2 x − sin 2 x )
2 2 4 4
u = x  du = 2 x dx
2

1
dv = sin 2 x dx  v = −cos 2 x
2
u = x  du = dx
1 2 1
x
2
sin 2 x dx =
4
x ( 2 x − sin 2 x) −
2
(2 x 2 − x sin 2 x) dx
1 1 1 3 1
= x3 − x 2 sin 2 x − x +  x sin 2 x dx
2 4 3 2
1 3 1 2 1 1 1 
= x − x sin 2 x +  − x cos 2 x +  cos 2 x dx 
6 4 2 2 2 
1 3 1 2 1 1
= x − x sin 2 x − x cos 2 x + sin 2 x + C
6 4 4 8
1
=
24
(4 x3 − 6 x 2 sin 2 x − 6 x cos 2 x + 3 sin 2 x) + C
π 2 2
15. 0 cos3 x dx = (n = 3, odd )
3

π 2  1  3  5  π  5
16. 0 cos 6 x dx =      = π (n = 6, even )
 2  4  6  2  32

π 2 1π  π
17. 0 sin 2 x dx =   = (n = 2, even )
2 2  4

π 2  2  4  6  8  128
18. 0 sin 9 x dx =      = (n = 9, odd )
 3  5  7  9  315

© 2018 Cengage Learning. All Rights Reserved. May not be scanned, copied or duplicated, or posted to a publicly accessible website, in whole or in part.
Section 8.3 Trigonometric Integrals 753

π 2  1  3  5  7  9  π  63
19. 0 sin10 x dx =        = π (n = 10, even )
 2  4  6  8  10  2  512

π 2  2  4  6  8  10  256
20. 0 cos11 x dx =       = (n = 11, odd)
 3  5  7  9  11  693

21.  sec 4 x dx = 1
4  sec 4 x (4 dx)
= 1 ln sec 4 x + tan 4 x + C
4

 sec  (1 + tan x) sec x dx


4 2 2
22. x dx =
1
= tan x + tan 3 x + C
3

1
23. dv = sec 2 π x dx  tan π x
v =
π
u = sec π x  du = π sec π x tan π x dx
1 1
 sec π x dx = sec π x tan π x −  sec π x tan π x dx = sec π x tan π x −  sec π x(sec π x − 1) dx
3 2 2
π π
1
2  sec3 π x dx =
π
(sec π x tan π x + ln sec π x + tan π x ) + C1
1
 sec
3
π x dx =

(sec π x tan π x + ln sec π x + tan π x )+C

 tan  (sec 3x − 1) tan 3x dx


6 2 4
24. 3 x dx =

 tan 3x sec 3x dx −  tan 3x dx


4 2 4
=

 tan 3x sec 3x dx −  tan 3x(sec 3x − 1) dx


4 2 2 2
=

 tan 3x sec 3x dx −  tan 3x sec 3x dx + (sec 3x + 1) dx


4 2 2 2 2
=
tan 5 3 x tan 3 3x tan 3 x
= − + + x +C
15 9 3
x  x  x
 tan   sec
5 2
25. dx = − 1 tan 3 dx
2 2  2
x x x
=  tan 3 sec2 dx −  tan 3 dx
2 2 2
x
tan 4
2 −  sec 2 x − 1 tan x dx
=
2   2  2
1 x x x
= tan 4 − tan 2 − 2 ln cos + C
2 2 2 2

πx πx 1 πx
 tan
3
26. sec 2 dx = tan 4 +C
2 2 2π 2

27. Let u = sec 2t , du = 2 sec 2t tan 2t dt.

 tan  (sec 2t − 1) sec3 2t ⋅ tan 2t dt


3
2t ⋅ sec3 2t dt = 2

sec5 2t sec3 2t
 (sec 2t − sec 2t )(sec 2t tan 2t ) dt
4 2
= = − +C
10 6

© 2018 Cengage Learning. All Rights Reserved. May not be scanned, copied or duplicated, or posted to a publicly accessible website, in whole or in part.
754 Chapter 8 Integration Techniques and Improper Integrals

 tan  tan x( tan x + 1) sec x dx 1


5
28. x sec 4 x dx = 5 2 2 2
 sin (πθ ) dθ  1 − cos( 2πθ ) dθ
4
35. r = = 
4
 ( tan x + tan x) sec x dx
7 5 2
=
1
1 − 2 cos( 2πθ ) + cos 2 ( 2πθ ) dθ
4 
=
1 1
= tan 8 x + tan 6 x + C
8 6 1  1 + cos( 4πθ ) 
=  1 − 2 cos( 2πθ ) +  dθ
4  2 
1
 sec sec5 4 x( 4 sec 4 x tan 4 x) dx
4
6
29. 4 x tan 4 x dx =
1 1 θ 1 
= θ − sin ( 2πθ ) + + sin ( 4πθ ) + C
sec6 4 x 4  π 2 8π 
= +C
24 1
= 12πθ − 8 sin ( 2πθ ) + sin ( 4πθ ) + C
32π 
x x x 1 x x
 sec tan dx = 2  sec  sec tan  dx
2
30. α α
2 2 2 2 2 2
 sin cos 2 dα
2
36. s =
x 2 2
+C= sec 2 or − cos α  1 + cos α  1 − cos 2 α
2 1
=   2

 2
 dα =

 4

2 x x x 1 2 x 2 x
 sec 2 tan 2 dx = 2 tan 2  2 sec 2  dx = tan 2 + C 1 1
sin 2 α dα =  (1 − cos 2α ) dα
4
=
8
31.  sec
5
x tan 3 x dx =  sec 4 x tan 2 x (sec x tan x) dx 1 sin 2α 
= θ −  +C
8 2 
=  sec 4 x(sec 2 x − 1)(sec x tan x) dx
1
= ( 2α − sin 2α ) + C
 (sec x − sec x)(sec x tan x) dx
6 4
= 16
sec 7 x sec5 x
= − +C 37. y =  tan 3 3 x sec 3 x dx
7 5
 (sec 3x − 1) sec 3x tan 3x dx
2
=
32.  tan 3 3 x dx =  (sec 3x − 1) tan 3x dx
2

= 13  sec 2 3 x(3 sec 3x tan 3x) dx − 13  3 sec 3x tan 3 x dx


1 1 −3 sin 3x
tan 3 x(3 sec 2 3x) dx + 
3
= dx = 1 sec3 3x − 13 sec 3 x + C
3 cos 3 x 9
1 1
= tan 2 3 x + ln cos 3 x + C
 tan x sec x dx
4
6 3 38. y =

 tan x( tan x + 1) sec x dx


12 2 2
=
tan 2 x (sec2 x − 1) dx
33.  sec x
dx =  sec x =  ( tan x + tan x) sec x dx
52 12 2

2 tan 7 2 x + 2 tan 3 2 x + C
=  (sec x − cos x) dx = 7 3

= ln sec x + tan x − sin x + C

tan 2 x sin 2 x
34.  sec5 x =  cos2 x ⋅ cos5 x dx

 sin x ⋅ cos x dx
2 3
=

 sin x(1 − sin x) cos x dx


2 2
=

 (sin x − sin x) cos x dx


2 4
=
sin 3 x sin 5 x
= − +C
3 5

© 2018 Cengage Learning. All Rights Reserved. May not be scanned, copied or duplicated, or posted to a publicly accessible website, in whole or in part.
Section 8.3 Trigonometric Integrals 755

y y
39. (a) 40. (a)
4 1

x x
4 −1 1

(0, − 14)
−4 −1

dy  1
dy (b) = sec 2 x tan 2 x,  0, − 
(b) = sin 2 x, (0, 0) dx  4
dx
y =  sec 2 x tan 2 x dx u = tan x, du = sec 2 x dx
1 − cos 2 x
y =  sin x dx = 
2
dx
2 tan 3 x
y= +C
1 sin 2 x 3
= x− +C
2 4  1 1 1 1
 0, −  : − = C  y = tan x −
3
1 sin 2 x  4 4 3 4
(0, 0): 0 = C , y = x −
2 4 1

− 1.5 1.5

−6 6

−1

−4
dy 3 sin x
41. = , y (0) = 2
dx y
8

−9 9

−4

dy
42. = 3 y tan 2 x, y (0) = 3
dx
8 (0, 3)

−1 1

−2

1
 cos 2 x cos 6 x dx cos((2 − 6) x) + cos(( 2 + 6) x) dx
2 
43. =

1
=  cos(−4 x) + cos 8 x dx
2
1
=  (cos 4 x + cos 8 x) dx
2
1  sin 4 x sin 8 x 
=  + +C
2 4 8 
sin 4 x sin 8 x
= + +C
8 16
1
= (2 sin 4 x + sin 8 x) + C
16

© 2018 Cengage Learning. All Rights Reserved. May not be scanned, copied or duplicated, or posted to a publicly accessible website, in whole or in part.
756 Chapter 8 Integration Techniques and Improper Integrals

1
44.  cos(5θ ) cos(3θ ) dθ =
2
cos(5 − 3)θ + cos(5 + 3)θ  dθ

1
=  (cos 2θ + cos 8θ ) dθ
2
1  sin 2θ sin 8θ 
=  + +C
2 2 8 
sin 2θ sin 8θ
= + +C
4 16

1
45.  sin 2t cos 9t dt =
2  sin (2 − 9)t  + sin (2 + 9)t  dt
1
=
2  sin(− 7t ) + sin(11t ) dt
1
(sin 7t + sin 11t ) dt
2
= −

1 1
= cos 7t − cos 11t + C
14 22

1
46.  sin 8 x cos 7 x dx =
2  sin (8 − 7) x + sin (8 + 7) x dx
1
=
2  (sin x + sin 15x) dx
1 1
= − cos x − cos 15 x + C
2 30

47.  sin θ sin 3θ dθ = 1


2  (cos 2θ − cos 4θ ) dθ

= 1 1
2 2 ( sin 2θ − 1
4 )
sin 4θ + C

= 1
8
( 2 sin 2θ − sin 4θ ) + C

1
 sin 5 x sin 4 x dx (cos x − cos 9 x) dx
2
48. =

1 sin 9 x 
=  sin x −  +C
2 9 
sin x sin 9 x
= − +C
2 18
1
= (9 sin x − sin 9x) + C
18

 cot  (csc 2 x − 1) cot 2 x dx


3 2
49. 2 x dx =
1 1 2 cos 2 x
cot 2 x( −2 csc 2 2 x) dx − 
2
= − dx
2 sin 2 x
1 1
= − cot 2 2 x − ln sin 2 x + C
4 2
1
=
4
(
ln csc 2 2 x − cot 2 2 x + C )

© 2018 Cengage Learning. All Rights Reserved. May not be scanned, copied or duplicated, or posted to a publicly accessible website, in whole or in part.
Section 8.3 Trigonometric Integrals 757

x x
 csc  csc 3x(1 + cot 3x) dx
4 2 2
51. 3 x dx =
 tan
5
50. sec 4 dx
4 4
 csc 3x dx +  cot 3x csc 3x dx
2 2 2
=
5 x x  x
=  tan  tan 2 + 1 sec 2 dx
4 4  4 1 1
= − cot 3 x − cot 3 3 x + C
 x x x 3 9
  tan
7
= + tan 5  sec 2 dx
4 4 4
x x
tan 8
2 tan 6
= 4 + 4 +C
2 3
1 x 2 x
= tan 8 + tan 6 + C
2 4 3 4

x x x x x x
 cot  cot
3
52. csc 4 dx = 2
csc3  csc cot  dx
2 2 2 2 2 2
 x  x x x
  csc
2
= − 1csc3  csc cot  dx
2  2 2 2
 x x  x x
  csc
5
= − csc3  csc cot  dx
2 2  2 2
1 x 1 x
= − csc6 + csc 4 + C
3 2 2 2

cot 2 t csc 2 t − 1 1 cos 2 x 1 − sin 2 x


53.  csc t
dt =  csc t
dt 55.  sec x tan x dx =  sin x
dx =  sin x
dx

=  (csc t − sin t ) dt =  (csc x − sin x) dx


= ln csc t − cot t + cos t + C = ln csc x − cot x + cos x + C

cot 3 t cos3 t (1 − sin 2 t )cos t dt 56. 


sin 2 x − cos 2 x
dx = 
1 − 2 cos 2 x
dx
54.  csc t dt =  sin 2 t dt =  sin 2 t cos x cos x
cos t =  (sec x − 2 cos x) dx
=  sin 2 t dt −  cos t dt
= ln sec x + tan x − 2 sin x + C
−1
= − sin t + C = −csc t − sin t + C
sin t

 ( tan t − sec 4 t ) dt =  ( tan t + sec t )( tan t − sec 2 t ) dt , ( tan 2 t − sec2 t = −1)


4 2 2 2
57.

= −  ( tan 2 t + sec 2 t ) dt = −  ( 2 sec2 t − 1) dt = −2 tan t + t + C

1 − sec t cos t − 1
58.  cos t − 1 dt =  (cos t − 1) cos t dt =  sec t dt = ln sec t + tan t + C

1 − cos 2 x π 4 π 4
59.
π
 − π sin
2
x dx = 2
0
π

2
dx 61. 0 6 tan 3 x dx = 6 
0
(sec2 x − 1) tan x dx
π π 4
 1  = 6 tan x sec 2 x − tan x dx
=  x − sin 2 x = π 0
 2 0 π 4
 tan 2 x 
= 6 + ln cos x 
π 3 π 3
 2
0 tan 2 x dx =  0 (sec x − 1) dx 0
2
60.
1  2  1 
π 3 π = 6  + ln   = 6 − ln 2
= [tan x − x]0 = 3 −  2 2  2 
3  
= 3(1 − ln 2)

© 2018 Cengage Learning. All Rights Reserved. May not be scanned, copied or duplicated, or posted to a publicly accessible website, in whole or in part.
758 Chapter 8 Integration Techniques and Improper Integrals

π 3 π 3 π 3 1 π3
0 0 sec1 2 x (sec x tan x) dx π 6 (sin 2 x + sin 10 x) dx
2 π 6
62. sec3 2 x tan x dx = 64. sin 6 x cos 4 x dx =
π 3 π 3
2   cos 2 x cos 10 x 
=  sec3 2 x = − −
3 0  4 20 π 6
2
= 2
3
( 2 −1 ) 1
=  +
1  1
 − − −
1
 =
3
 8 40   8 40  10

63. Let u = 1 + sin t , du = cos t dt. π 2 π 2


65.  − π 2 3 cos
3
x dx = 3
−π 2
(1 − sin 2 x) cos x dx
π 2 cos t π 2
0 1 + sin t
dt = ln 1 + sin t  0 = ln 2
 sin 3 x 
π 2
= 3sin x − 
 3  −π 2

 1  1 
= 31 −  −  −1 +  = 4
 3  3 

π π
66. 0 sin 5 x dx = 0 sin 4 x sin x dx
π
0 (1 − cos x)(1 − cos x) sin x dx
2 2
=
π
= 0 1 − 2 cos 2 x + cos 4 x sin x dx
π
 2 1 
= − cos x + cos3 x − cos5 x
 3 5 0
 2 1   2 1
= − ( −1) + ( −1) − ( −1) − −1 + − 
 3 5   3 5
4 2 16
= 2− + =
3 5 15

67. (a) Let u = tan 3x, du = 3 sec 2 3x dx.


1
 sec
4
3x tan 3 3x dx =  sec
2
3x tan 3 3x sec2 3x dx =
3
( tan 2 3x + 1) tan3 3x(3 sec2 3x) dx
1 tan 6 3x tan 4 3x
=
3  ( tan 5 3x + tan 3 3x)(3 sec2 3x) dx =
18
+
12
+ C1

Or let u = sec 3x, du = 3 sec 3x tan 3x dx.

 sec  sec
4
3 x tan 3 3 x dx = 3
3 x tan 2 3 x sec 3 x tan 3 x dx
1 sec6 3x sec 4 3 x
=  sec3 3 x(sec 2 3 x − 1)(3 sec 3 x tan 3 x) dx = − +C
3 18 12
(b) 0.05

− 0.5 0.5

− 0.05

(1 + tan 2 3x) − (1 + tan 2 3x) + C


3 2
sec6 3x sec 4 3 x
(c) − +C =
18 12 18 12
1 1 1 1 1 1 1
= tan 3x + tan 3x + tan 2 3x +
6 4
− tan 4 3x − tan 2 3x − +C
18 6 6 18 12 6 12
tan 6 3x tan 4 3x  1 1
= + + − +C
18 12  18 12 
tan 6 3x tan 4 3x
= + + C2
18 12

© 2018 Cengage Learning. All Rights Reserved. May not be scanned, copied or duplicated, or posted to a publicly accessible website, in whole or in part.
Section 8.3 Trigonometric Integrals 759

68. (a) Let u = tan x, du = sec 2 x dx. (b) 8

 sec 1
2
x tan x dx = 2
tan 2 x + C1

Or let u = sec x, du = sec x tan x dx. −4 4

 sec x(sec x tan x) dx 1 −2


= 2
sec 2 x + C

(c) 1
2
sec 2 x + C = 1
2 (tan 2 x + 1) + C
= 2 (2 )
1 tan 2 x + 1 + C

= 1 tan 2 x + C2
2

sin 2 x
69. (a)  sin x cos x dx = 2
+C

cos 2 x
(b) −  cos x ( −sin x) dx = − +C
2
(c) dv = cos x dx  v = sin x
u = sin x  du = cos x dx

 sin x cos x dx = sin 2 x −  sin x cos x dx


2  sin x cos x dx = sin x 2

sin 2 x
 sin x cos x dx =
2
+C

(Answers will vary.)


1 1
(d)  sin x cos x dx =  2 sin 2 x dx = −
4
cos 2 x + C

The answers all differ by a constant.

70. (a) f has a maximum at the points where f ′ changes from positive to negative: x = − π , π .
(b) f has a minimum at the points where f ′ changes from negative to positive: x = 0.

π 2 π 4
 0 (sin x − sin x) dx  − π 4 cos x − sin 2 x dx
3 2
71. A = 73. A =
π 2 π 2 π 4
= 0 sin x dx − 0 sin 3 x dx =  − π 4 cos 2 x dx
π 2 π 4
= [−cos x]0 − 2
( Wallis's Formula )  sin 2 x  1 1
3 =   = + =1
=1− 2 = 1  2  −π 4 2 2
3 3

π 4
 − π 2 cos x − sin x cos x dx
2
1
sin 2 (π x) dx 74. A =
72. A = 0
11 − cos( 2π x) π 4 1 + cos 2 x 
= 0 dx =  − π 2  2
− sin x cos x dx

2
1 π 4
1 sin 2π x  1 1 sin 2 x sin 2 x
=  x − = =  x + − 
2 4π  0 2 2 4 2  −π 2

π 1 1  π 1
=  + −  − − − 
 8 4 4   4 2
3π 1
= +
8 2

© 2018 Cengage Learning. All Rights Reserved. May not be scanned, copied or duplicated, or posted to a publicly accessible website, in whole or in part.
760 Chapter 8 Integration Techniques and Improper Integrals

75. Disks π 2  2 x  2  x 
76. V = π  cos  2  − sin  2  dx
R( x) = tan x, r ( x) = 0 0
    
π 4 π 2
V = 2π  tan 2 x dx = π cos x dx
0 0
π 2
= π [sin x]0
π 4
= 2π 
0
(sec2 x − 1) dx = π

π 4
= 2π [tan x − x]0
 π
= 2π 1 −  ≈ 1.348
 4
y

1
2

x
π π
8 4
− 12

−1

π
π π π π 1  π2
77. (a) V = π  sin 2 x dx = (1 − cos 2 x) dx
2 0
=  x − sin 2 x =
0 2 2 0 2
y
π π
(b) A =  0 sin x dx = [−cos x]0 =1+1 = 2
1
Let u = x, dv = sin x dx, du = dx, v = −cos x. ( π2 , π8 (
1
1 π 1
x sin x dx = [− x cos x]0 +
π π
 = 1 [− x cos x + sin x]π = π
A0  0 cos x dx
2
x =
2  2 0
2
π x
1 π 1 π 1 1  π π π
sin 2 x dx =  (1 − cos 2 x) dx =  x − sin 2 x =
2A0
y = 2

8 0 8 2 0 8
π π 
(x, y) =  , 
2 8

π 2
π 2 π π 2 π 1  π2
78. (a) V = π  (1 + cos 2 x) dx =
2 0
cos 2 x dx = x + sin 2 x =
0 2  2 0 4
π 2 π 2
(b) A = 0 cos x dx = [sin x]0 =1

Let u = x, dv = cos x dx, du = dx, v = sin x.


π 2 π 2 π 2 π 2 π π −2
x = 0 x cos x dx = [ x sin x]0 − 0 sin x dx = [ x sin x + cos x]0 = −1 =
2 2
π 2
1 π2 1 π 2 1 1  π
(1 + cos 2 x) dx =
20
y = cos 2 x dx =  x + sin 2 x =
4 0 4  2 0 8
π − 2 π 
(x, y) =  , 
 2 8
y

1
( π 2− 2 , π8 (
1
2

x
π π
4 2

© 2018 Cengage Learning. All Rights Reserved. May not be scanned, copied or duplicated, or posted to a publicly accessible website, in whole or in part.
Section 8.3 Trigonometric Integrals 761

79. dv = sin x dx  v = −cos x


u = sin n −1
x  du = ( n − 1) sin n − 2 x cos x dx

 sin
n
x dx = −sin n −1 x cos x + ( n − 1)  sin n − 2 x cos 2 x dx = −sin n −1 x cos x + ( n − 1)  sin n − 2 x(1 − sin 2 x) dx

= −sin n −1 x cos x + ( n − 1)  sin n − 2 x dx − ( n − 1)  sin n x dx

Therefore, n  sin n x dx = −sin n −1 x cos x + ( n − 1)  sin n − 2 x dx


−sin n −1 x cos x n −1
 sin n 
sin n − 2 x dx.
n
x dx = +
n

80. dv = cos x dx  v = sin x


u = cos n −1
x  du = −( n − 1) cos n − 2 x sin x dx

 cos
n
x dx = cos n −1 x sin x + ( n − 1)  cos n − 2 x sin 2 x dx

= cos n −1 x sin x + ( n − 1)  cos n − 2 x(1 − cos 2 x) dx = cos n −1 x sin x + ( n − 1)  cos n − 2 x dx − ( n − 1)  cos n x dx

Therefore, n  cos n x dx = cos n −1 x sin x + ( n − 1)  cos n − 2 x dx


cos n −1 x sin x n −1
 cos n 
cos n − 2 x dx.
n
x dx = +
n

−cos m + 1 x
81. Let u = sin n −1 x, du = ( n − 1) sin n − 2 x cos x dx, dv = cos m x sin x dx, v = .
m +1
−sin n −1 x cos m +1 x n −1
 cos + 1
sin n − 2 x cos m + 2 x dx
m
x sin n x dx = +
m +1 m
−sin n −1 x cos m +1 x n −1
sin n − 2 x cos m x(1 − sin 2 x) dx
+ 1
= +
m +1 m
−sin n −1 x cos m +1 x n −1 n −1
+ 1 m + 1
= + sin n − 2 x cos m x dx − sin n x cos m x dx
m +1 m
n −1
m + n −sin x cos m +1 x n −1
m + 1 + 1
cos m x sin n x dx = + sin n − 2 x cos m x dx
m +1 m
−cos m + 1 x sin n −1 x n −1
 cos + n
cos m x sin n − 2 x dx
m
x sin n x dx = +
m + n m

82. Let u = secn − 2 x, du = ( n − 2) secn − 2 x tan x dx, dv = sec2 x dx, v = tan x.

 sec x dx = sec n − 2 x tan x −  (n − 2) sec x tan x dx


n n−2 2

= sec n − 2 x tan x − ( n − 2)  sec n − 2 x(sec 2 x − 1) dx

= sec n − 2 x tan x − ( n − 2)  sec n x dx −  sec n − 2 x dx

(n − 1)  sec n x dx = sec n − 2 x tan x + ( n − 2)  sec n − 2 x dx


1 n − 2
 sec secn − 2 x tan x +
n − 1
sec n − 2 x dx
n
x dx =
n −1

sin 4 x cos x 4
 sin +  sin 3 x dx
5
83. x dx = −
5 5
sin 4 x cos x 4  sin 2 x cos x 2 
= − + − +  sin x dx 
5 5 3 3 
1 4 4 2 8
= − sin x cos x − sin x cos x − cos x + C
5 15 15
cos x
= −
15
(3 sin 4 x + 4 sin 2 x + 8) + C

© 2018 Cengage Learning. All Rights Reserved. May not be scanned, copied or duplicated, or posted to a publicly accessible website, in whole or in part.
762 Chapter 8 Integration Techniques and Improper Integrals

cos3 x sin x 3
 cos +  cos 2 x dx
4
84. x dx =
4 4
cos3 x sin x 3  cos x sin x 1 
= +  +  dx 
4 4 2 2 
1 3 3
= cos3 x sin x + cos x sin x + x + C
4 8 8
1
= ( 2 cos3 x sin x + 3 cos x sin x + 3 x) + C
8

cos3 x sin 3 x 1
 sin +  cos 2 x sin 2 x dx
4
85. x cos 2 x dx = −
6 2
cos3 x sin 3 x 1  cos3 x sin x 1 
= − + − +  cos 2 x dx 
6 2 4 4 
1 1 1  cos x sin x x
= − cos3 x sin 3 x − cos3 x sin x +  +  +C
6 8 8 2 2
1
= −
48
(8 cos3 x sin 3 x + 6 cos3 x sin x − 3 cos x sin x − 3x) + C
2π x 5  2π x  2π
 sec 2π 
4
86. dx = sec 4   dx
5  5  5
5 1  2π x   2π x  2 2  2π x  2π 
= sec 2   tan   +  sec   dx
2π  3  5   5  3  5  5 
5  2  2π x   2π x   2π x 
= sec   tan   + 2 tan   + C
6π   5   5   5 
5  2π x   2  2π x  
= tan   sec  + 2 + C
6π  5   5  

87. (a) n is odd and n ≥ 3.


π 2
π 2  cos n −1 x sin x  n −1 π 2
0 cos n x dx = 
 n

0
+
n 0
cos n − 2 x dx

n − 1  cos n − 3 x sin x  
π 2
n −3 π2
= 
n  n − 2

0
+
n − 2  0
cos n − 4 x dx 

 
n − 1 n − 3  cos  n−5
x sin x 
π 2
n −5 π2 
n − 40
= ⋅   + cos n−6
x dx 
n n − 2  n − 4  
 0 
n −1 n −3 n −5 π 2
n − 2 n − 40
n−6
= ⋅ ⋅ cos x dx
n
n −1 n −3 n −5 π 2

n − 2 n − 4 0
= ⋅ ⋅  cos x dx
n
π 2
n − 1 n − 3 n − 5 
=  ⋅ ⋅  (sin x)
 n n − 2 n − 4 0
n −1 n −3 n −5
= ⋅ ⋅  1 ( Reverse the order.)
n n − 2 n − 4
 2  4  6   n − 1   2  4  6   n − 1 
= (1)       =       
 3  5  7   n   3  5  7   n 

© 2018 Cengage Learning. All Rights Reserved. May not be scanned, copied or duplicated, or posted to a publicly accessible website, in whole or in part.
Section 8.3 Trigonometric Integrals 763

(b) n is even and n ≥ 2.


π 2 n −1 n −3 n −5 π 2
0 cos n x dx =
n

n − 2 n − 4 0
⋅  cos 2 x dx (From part (a ))
π 2
n − 1 n − 3 n − 5  x 1 
=  ⋅ ⋅   + sin 2 x 
 n n − 2 n − 4 2 4  0
n −1 n −3 n −5 π
= ⋅ ⋅  (Reverse the order.)
n n−2 n−4 4
 π 1  3  5   n − 1   1  3  5   n − 1  π 
=  ⋅       =        
 2 2  4  6   n   2  4  6   n  2 

π
π 1  sin ( m + n) x sin ( m − n) x 
88.  −π cos( mx) cos( nx) dx =  +  = 0, (m ≠ n)
2 m + n m − n  −π
π 1 π
 − π sin(mx) sin(nx) dx = cos( m − n) x − cos( m + n) x dx
2  −π 
π
1  sin ( m − n) x sin ( m + n) x 
=  −  = 0, (m ≠ n)
2 m − n m + n  −π
π 1 π
 − π sin(mx) cos(nx) dx = sin ( m + n) x + sin ( m − n) x dx
2  −π 
π
1  cos( m + n) x cos( m − n) x 
= −  +  , (m ≠ n)
2 m + n m − n  −π

1  cos( m + n)π cos( m − n)π   cos( m + n)( −π ) cos( m − n)( −π ) 


= −  +  − + 
2  m + n m − n   m + n m − n 
= 0, because cos( −θ ) = cos θ .
π
π 1  sin 2 ( mx) 
 −π sin ( mx) cos( mx) dx =   = 0
m 2  −π
N
89. f ( x) =  ai sin(ix)
i =1

N 
(a) f ( x) sin ( nx) =  ai sin (ix) sin ( nx)
 i =1 
π π  N 
 −π f ( x) sin ( nx) dx =   ai sin (ix) sin ( nx) dx
−π
 i =1 
π
 − π an sin (nx) dx (by Exercise 89)
2
=
π
π 1 − cos( 2nx) a  sin ( 2nx)  an
=  −π an dx =  n  x −  = (π + π ) = anπ
2 2
  2 n  −π 2

1 π
f ( x) sin( nx) dx.
π  −π
So, an =

(b) f ( x) = x

1 π

π  −π
a1 = x sin x dx = 2

1 π

π  −π
a2 = x sin 2 x dx = −1

1 π 2
π  −π
a3 = x sin 3 x dx =
3

© 2018 Cengage Learning. All Rights Reserved. May not be scanned, copied or duplicated, or posted to a publicly accessible website, in whole or in part.
764 Chapter 8 Integration Techniques and Improper Integrals

Section 8.4 Trigonometric Substitution


1. (a) Use x = 3 tan θ . 2. In order to use integration by trigonometric substitution,
(b) Use x = 2 sin θ . you need radicals of the form a2 − u 2 , a 2 + u 2 , or

(c) Use x = 5 sin θ . u 2 − a 2 . Completing the square allows you to convert


many integrands to one of these forms.
(d) Use x = 5 sec θ .

3. Let x = 4 sin θ , dx = 4 cos θ dθ , 16 − x 2 = 4 cos θ .

4
x

16 − x 2

1 4 cos θ 1 1 1 x 
 dx =  (4 cos θ )3 dθ =
16 
sec 2 θ dθ = tan θ + C =   +C
(16 − x )
32
2 16 16  16 − x 2 

4. Same substitution as in Exercise 3


4 4 cos θ 1 1 1 16 − x 2 − 16 − x 2
 x2 16 − x 2
dx = 4 
( 4 sin θ ) ( 4 cos θ )
2
dθ =
4
csc 2 θ dθ = − cot θ + C = −
4 4 x
+C =
4x
+C

5. Same substitution as in Exercise 5


16 − x 2 4 cos θ
 x
dx =  4 sin θ 4 cos θ dθ

cos 2 θ
= 4 dθ
sin θ
1 − sin 2 θ
= 4 dθ
sin θ
= 4 (csc θ − sin θ ) dθ
= − 4 ln csc θ + cot θ + 4 cos θ + C

4 16 − x 2 16 − x 2
= − 4 ln + + 4 +C
x x 4

4+ 16 − x 2
= − 4 ln + 16 − x 2 + C
x

4− 16 − x 2
= 4 ln + 16 − x 2 + C
x

© 2018 Cengage Learning. All Rights Reserved. May not be scanned, copied or duplicated, or posted to a publicly accessible website, in whole or in part.
Section 8.4 Trigonometric Substitution 765

6. Same substitution as in Exercise 5


x3 (4 sin θ )3
 16 − x 2
dx =  4 cos θ
4 cos θ dθ

= 64  sin 3 θ dθ

= 64  (1 − cos 2 θ ) sin θ dθ

 cos3 θ 
= 64 − cos θ +  +C
 3 
 
(16 − x 2 )
32
16 − x 2
= 64 − +  +C
 4 64(3) 
 
1
(16 − x 2 ) + C
32
= −16 16 − x 2 +
3
1
= − 16 − x 2 48 − (16 − x 2 ) + C
3
1
= − 16 − x 2 (32 + x 2 ) + C
3

7. Let x = 5 sec θ , dx = 5 sec θ tan θ dθ ,

x 2 − 25 = 5 tan θ .
1 5 sec θ tan θ
 
x
dx = dθ x 2 − 25
2
x − 25 5 tan θ
θ
=  sec θ dθ 5

= ln sec θ + tan θ + C

x x 2 − 25
= ln + +C
5 5

= ln x + x 2 − 25 + C

8. Same substitution as in Exercise 9


x 2 − 25 5 tan θ
 x
dx =  5 sec θ 5 sec θ tan θ dθ

= 5 tan 2 θ dθ

= 5 (sec 2 θ − 1) dθ
= 5( tan θ − θ ) + C
 x 2 − 25 x
= 5 − arcsec  + C
 5 5 

x
= x 2 − 25 − 5 arcsec + C
5
  x  x 2 − 25 
Note: arcsec  = arctan  
  
5  5 
 

© 2018 Cengage Learning. All Rights Reserved. May not be scanned, copied or duplicated, or posted to a publicly accessible website, in whole or in part.
766 Chapter 8 Integration Techniques and Improper Integrals

9. Same substitution as in Exercise 9

 (5 sec θ ) (5 tan θ )(5 sec θ tan θ ) dθ


3
x
3
x 2 − 25 dx =

= 3125 sec 4 θ tan 2 θ dθ

= 3125 (1 + tan 2 θ ) tan 2 θ sec 2 θ dθ

= 3125 ( tan 2 θ + tan 4 θ ) sec2 θ dθ

 tan 3 θ tan 5 θ 
= 3125 +  +C
 3 5 
 x 2 − 25 3 2
( ) + ( x 2 − 25)  + C
52

= 3125
 125(3) 55 (5) 
 
1 2
( x − 25) 125 + 3( x2 − 25) + C
32
=
15
1 2
( x − 25) (50 + 3x 2 ) + C
32
=
15

10. Same substitution as in Exercise 9


x3 (5 sec θ )3
 x 2 − 25
dx =  5 tan θ
5 sec θ tan θ dθ

= 125 sec 4 θ dθ

= 125 ( tan 2 θ + 1) sec 2 θ dθ

 tan 3 θ 
= 125 + tan θ  + C
 3 

125 ( x − 25)
32
2
x 2 − 25
= + 125 +C
3 125 5
1
= ( x 2 − 25) + 25( x 2 − 25) + C
32 12

3
1
= x 2 − 25 ( x 2 − 25 + 75) + C
3
1
= x 2 − 25 (50 + x 2 ) + C
3

11. Let x = 2 tan θ , dx = 2 sec2 θ dθ , 4 + x 2 = 2 sec θ .


x
 4 + x 2 dx =  tan θ (2 sec θ )(2 sec θ ) dθ
2
2
= 4  sec 2 θ (sec θ tan θ ) dθ
4
= sec3 θ + C
3

4 (4 + x )
32
2
4 + x2
= ⋅ +C x
3 8
1
= (4 + x2 ) + C
32 θ
6 2

© 2018 Cengage Learning. All Rights Reserved. May not be scanned, copied or duplicated, or posted to a publicly accessible website, in whole or in part.
Section 8.4 Trigonometric Substitution 767

12. Same substitution as in Exercise 11


x3 8 tan 3 θ
  4(2 sec θ ) (2 sec θ ) dθ
2
dx =
4 4 + x2
= 2  tan 3 θ sec θ dθ

= 2  tan 2 θ ( tan θ sec θ ) dθ

= 2  (sec 2 θ − 1)( tan θ sec θ ) dθ


2
= sec3 θ − 2 sec θ + C
3

2 (4 + x ) (4 + x2 ) + C
32 12
2
= ⋅ − 2⋅
3 8 2
1
= 4 + x 2 ( 4 + x 2 − 12) + C
12
4 + x2
= ( x 2 − 8) +C
12

13. Same substitution as in Exercise 11


4 4
  (2 sec θ )4 (2 sec θ ) dθ
2
dx =
(4 + x )
2
2

1
2
= cos 2 θ dθ

1 1 + cos 2θ
=  dθ
2 2
1 1 1 1
= θ + sin 2θ + C = θ + sin θ cos θ + C
4 8 4 4
1 x 1 x  2 
= arctan +    +C
4 2 4 x 2 + 4  x2 + 4 
1 x 2x 
=  arctan + 2  +C
4 2 x + 4

14. Same substitution as in Exercise 11


2( 2 tan θ )
2
2x2
  (2 sec θ )4 (2 sec θ ) dθ
2
dx =
(4 + x 2 )
2

tan 2 θ
=  sec2 θ dθ

 sin θ dθ
2
=
1 − cos 2θ
=  2

1 sin 2θ
= θ − +C
2 4
1 1
= θ − sin θ cos θ + C
2 2
1 x 1 x  2 
= arctan −    +C
2 2 2  x 2 + 4  x2 + 4 
1 x x
= arctan − 2 +C
2 2 x + 4

© 2018 Cengage Learning. All Rights Reserved. May not be scanned, copied or duplicated, or posted to a publicly accessible website, in whole or in part.
768 Chapter 8 Integration Techniques and Improper Integrals

15. Let u = 4 x, a = 7, du = 4 dx.


1
7 2 − ( 4 x) 4 dx
2
 49 − 16 x 2 dx =
4
1  1  4x 
=   4 x 49 − 16 x 2 + 49 arcsin  + C
4  2  4 
1 49 4x
= x 49 − 16 x 2 + arcsin + C
2 8 7

16. Let u = 5 x, a = 1, du = 5 dx.

1
( )
2
 5 x 2 − 1 dx =
5
5x −1 5 dx

=
1 1
 
5  2
( 5 x 5 x 2 − 1 − ln 5x + 5x2 − 1 )+C
x 5
= 5x2 − 1 − ln 5x + 5x2 − 1 + C
2 10

17. Let u = 5 x, du = 5 dx, a = 6.


1
 36 − 5 x 2 dx =
5  36 − 5 x 2 ( 5 dx )
1  5x 
=  5 x 36 − 5 x 2 + 36 arcsin  +C
2 5  6 

18. Let u = 2 x, du = 2 dx, a = 3.


1
 9 + 4 x 2 dx =
2
9 + 4 x 2 2 dx

1 1 
=   2 x 9 + 4 x 2 + 9 ln 2 x + 9 + 4 x2  + C
2  2   
x 9
= 9 + 4 x2 + ln 2 x + 9 + 4 x2 + C
2 4

19. Let x = 2 sin θ , dx = 2 cos θ dθ , 20. Let x = 2 sec θ , dx = 2 sec θ tan θ dθ ,


4 − x 2 = 2 cos θ . x 2 − 4 = 2 tan θ .

 16 − 4 x 2 dx = 2  4 − x 2 dx
x
x2 − 4
= 2  2 cos θ ( 2 cos θ dθ )
θ
2
= 8 cos 2 θ dθ
1 2 sec θ tan θ
= 4  (1 + cos 2θ ) dθ  x − 42
dx =  2 tan θ

 1 
= 4θ + sin 2θ  + C
 2 
=  sec θ dθ
= 4θ + 4 sin θ cos θ + C = ln sec θ + tan θ + C

 x x x2 − 4
= 4 arcsin   + x 4 − x 2 + C = ln + +C
 2 2 2

2 = ln x + x2 − 4 + C
x

4 − x2

© 2018 Cengage Learning. All Rights Reserved. May not be scanned, copied or duplicated, or posted to a publicly accessible website, in whole or in part.
Section 8.4 Trigonometric Substitution 769

21. Let x = sin θ , dx = cos θ dθ , 1 − x 2 = cos θ .

1 − x2 cos θ (cos θ dθ )
 x4
dx =  sin 4 θ

 cot θ csc 2 θ dθ
2
=
1
= − cot 3 θ + C
3
(1 − x 2 )
32

= − +C
3x3

1
x

1 − x2

22. Let 5x = 2 tan θ , 5dx = 2 sec2 θ dθ , 25x 2 + 4 = 4 tan 2 θ + 4 = 2 sec θ .

25 x 2 + 4 2 sec θ
2 
 x4
dx =  2 4 
 5
sec 2 θ  dθ

 tan θ 
 5 
125 cos θ
4  sin 4 θ
= dθ

125  1 
=   +C
4  ( − 3)sin θ 
3

125
= − csc3 θ + C
12
3
125  25 x 2 + 4 
= −   25x 2 + 4
12  5x 
 5x

(25x 2 + 4)
32 θ
2
= − +C
12 x3

3 3
23. Let 2 x = 3 tan θ  x = tan θ , dx = sec 2 θ dθ , 4 x 2 + 9 = 3 sec θ .
2 2
1 (3 2) sec2 θ dθ
x 4 x2 + 9
dx =  (3 2) tan θ 3 sec θ
1
3
= csc θ dθ 4x 2 + 9
2x
1
= − ln csc θ + cot θ + C θ
3 3
1 4x2 + 9 + 3
= − ln +C
3 2x

© 2018 Cengage Learning. All Rights Reserved. May not be scanned, copied or duplicated, or posted to a publicly accessible website, in whole or in part.
770 Chapter 8 Integration Techniques and Improper Integrals

24. Let 3x = tan θ , 3dx = sec2 θ dθ , 9 x 2 + 1 = sec θ .


1 1 1 
   sec θ  dθ
2
dx =
x 9x2 + 1 1  3 
tan θ sec θ
3
sec θ
=  tan θ

=  csc θ dθ
= ln csc θ − cot θ + C

9 x2 + 1 1
= ln − +C
3x 3x
9x 2 + 1
2 3x
9x + 1 − 1
= ln +C
3x θ
1
2
9x + 1 + 1
(Note: This equals − ln + C. )
3x

25. Let x = 3 tan θ , dx = 3 sec 2 θ dθ , 27. Let e x = sin θ , e x dx = cos θ dθ , 1 − e2 x = cos θ .


x 2 + 3 = 3 tan 2 θ + 3 = 3 sec 2 θ .
e 1 − e 2 x dx =  cos θ dθ
x 2

−3 −3 1
(1 + cos 2θ ) dθ
( ) 2
=
 dx = 3 3 sec 2 θ dθ
(x + 3)
32
2 3 sec3 θ
1 sin 2θ 
= θ + 
= −  cos θ dθ 2 2 
= − sin θ + C 1
= (θ + sin θ cos θ ) + C
x 2
= − + C
x2 + 3
1
(
= arcsin e x + e x 1 − e 2 x + C
2
)
x2 + 3 1
x ex

θ θ
3
1 − e 2x

26. Let x = 5 tan θ , dx = 5 sec 2 θ , 28. Let x = sin θ , x = sin 2 θ , dx = 2 sin θ cos θ dθ ,
x 2 + 5 = 5 sec 2 θ . 1 − x = cos θ .

1− x cos θ ( 2 sin θ cos θ dθ )


5 + x2
x  x
dx =  sin θ
θ
5 = 2  cos θ dθ 2

1 5 sec 2 θ
=  (1 + cos 2θ ) dθ
 dx =  dθ = (θ + sin θ cos θ ) + C
( x 2 + 5) ( )
32 3
5 sec θ
= arcsin x + x 1− x +C
1
5
= cos θ dθ
1
1 x x
= sin θ + C = +C
5 5 5 + x2 θ

1−x

© 2018 Cengage Learning. All Rights Reserved. May not be scanned, copied or duplicated, or posted to a publicly accessible website, in whole or in part.
Section 8.4 Trigonometric Substitution 771

29. Let x = 2 tan θ , dx = 2 sec 2 θ dθ , x 2 + 2 = 2 sec2 θ .

1 1 2 sec 2 θ dθ
 4 + 4x2 dx =  dx = 
( x2 + 2)
2
+ x 4
4 sec 4 θ

2 21 x2 + 2
  (1 + cos 2θ ) dθ
4  4  2 
= cos 2 θ dθ = x

θ
2 1  2
= θ + sin 2θ  + C = (θ + sin θ cos θ ) + C 2
8  2  8
2 x x 2  1 x 1 x 
=  arctan + ⋅  =  2 + arctan  +C
8  2 2
x + 2 2
x + 2 4 x + 2 2 2

30. Let x = tan θ , dx = sec 2 θ dθ , x 2 + 1 = sec 2 θ .

x3 + x + 1 1 4 x3 + 4 x 1
 dx =
4  x4 + 2x2 + 1
dx +  dx
(x + 1)
2
x4 + 2 x2 + 1 2

1 sec 2 θ dθ
= ln ( x 4 + 2 x 2 + 1) + 
4 sec 4 θ
1 1
= ln ( x 2 + 1) + (1 + cos 2θ ) dθ
2 2
1 1
= ln ( x 2 + 1) + (θ + sin θ cos θ ) + C x2 + 1
x
2 2
1 x 
=  ln ( x 2 + 1) + arctan x + 2
θ
 +C
2 x + 1 1

1
31. Use integration by parts. Because x > ,
2
1
u = arcsec 2 x  du = dx, dv = dx  v = x
x 4x2 − 1 2x
4x 2 − 1
1
 arcsec 2 x dx = x arcsec 2 x −  4x2 − 1
dx θ
1
1
2 x = sec θ , dx = sec θ tan θ dθ , 4 x 2 − 1 = tan θ
2
(1 2) sec θ tan θ dθ 1
 arcsec 2 x dx = x arcsec 2 x −  tan θ
= x arcsec 2 x −
2
sec θ dθ

1 1
= x arcsec 2 x − ln sec θ + tan θ + C = x arcsec 2 x − ln 2 x + 4 x 2 − 1 + C.
2 2

1 x2
32. u = arcsin x  du = dx, dv = x dx  v =
1 − x2 2
x2 1 x2
 x arcsin x dx =
2
arcsin x − 
2 1 − x2
dx 1
x

x = sin θ , dx = cos θ dθ , 1 − x 2 = cos θ θ

x2
1 sin θ x 2
1 2 1 − x2
 x arcsin x dx = arcsin x =  cos θ dθ = arcsin x −  (1 − cos 2θ ) dθ
2 2 cos θ 2 4
x2 1 1  x2 1
= arcsin x − θ − sin 2θ  + C = arcsin x − (θ − sin θ cos θ ) + C
2 4 2  2 4

=
x2
2
1
( 1
)
arcsin x − arcsin x − x 1 − x 2 + C = ( 2 x 2 − 1) arcsin x + x 1 − x 2  + C
4 4 

© 2018 Cengage Learning. All Rights Reserved. May not be scanned, copied or duplicated, or posted to a publicly accessible website, in whole or in part.
772 Chapter 8 Integration Techniques and Improper Integrals

x x
33.  dx =  dx
4 − ( x − 2)
2 2
4x − x

Let x − 2 = 2 sin θ , dx = 2 cos θ dθ ,

4 − ( x − 2)
2
= 4 − 4 sin 2 θ = 4 cos 2 θ = 2 cos θ .

x 2 + 2 sin θ
 dx =  2 cos θ
2 cos θ dθ
4 − ( x − 2)
2

=  (2 + 2 sin θ ) dθ
= 2θ − 2 cos θ + C

4 − ( x − 2)
2
x − 2 2
= 2 arcsin − 2 +C x−2
2 2
θ
x − 2
= 2 arcsin − 4x − x2 + C 4 − (x − 2)2
2

1 − ( x − 1)
2
34. Let x − 1 = sin θ , dx = cos θ dθ , = 2 x − x 2 = cos θ .

x2 x2
 dx =  dx
2 x − x2 1 − ( x − 1)
2

(1 + sin θ ) (cos θ dθ )
2

=  cos θ
=  (1 + 2 sin θ + sin 2 θ ) dθ

3 1 
=   2 + 2 sin θ −
2
cos 2θ  dθ
 1
x−1
3 1
= θ − 2 cos θ − sin 2θ + C
2 4 θ

3 1 1 − (x − 1)2
= θ − 2 cos θ − sin θ cos θ + C
2 2
3 1
= arcsin ( x − 1) − 2 2 x − x 2 − ( x − 1) 2 x − x 2 + C
2 2
3 1
= arcsin ( x − 1) − 2x − x2 ( x + 3) + C
2 2

( 3)
2
35. x 2 + 6 x + 12 = x 2 + 6 x + 9 + 3 = ( x + 3) +
2

x 2 + 6x + 12
Let x + 3 = 3 tan θ , dx = 3 sec 2 θ dθ . x+3

( 3)
2
(x + 3) +
2
x 2 + 6 x + 12 = = 3 sec θ 3

x 3 tan θ − 3
 2
x + 6 x + 12
dx =  3 sec θ
3 sec 2 θ dθ

=  3 sec θ tan θ dθ − 3 sec θ dθ

= 3 sec θ − 3 ln sec θ + tan θ + C


 x 2 + 6 x + 12  x 2 + 6 x + 12 x +3
= 3  − 3 ln + +C
 3  3 3
 
= x 2 + 6 x + 12 − 3 ln x 2 + 6 x + 12 + ( x + 3) + C

© 2018 Cengage Learning. All Rights Reserved. May not be scanned, copied or duplicated, or posted to a publicly accessible website, in whole or in part.
Section 8.4 Trigonometric Substitution 773

(x − 3) − 4 = 2 tan θ .
2
36. Let x − 3 = 2 sec θ , dx = 2 sec θ tan θ dθ ,

x x
 dx =  dx
2
x − 6x + 5 (x − 3) − 4
2

(2 sec θ + 3)
=  (2 sec θ tan θ ) dθ
2 tan θ x−3
(x − 3)2 − 4

 (2 sec θ + 3 sec θ ) dθ
2
=
θ
= 2 tan θ + 3 ln sec θ + tan θ + C1 2

 − 3) − 4 
(x (x − 3) − 4
3 2
x −3
= 2 + 3 ln + + C1
 2  2 2
 

= x 2 − 6 x + 5 + 3 ln ( x − 3) + x2 − 6x + 5 + C

37. Let t = sin θ , dt = cos θ dθ , 1 − t 2 = cos 2 θ .

t2 sin 2 θ cos θ dθ t
   tan θ dθ =  (sec θ − 1) dθ = tan θ − θ + C =
2 2
(a) dt = = − arcsin t + C
(1 − t ) cos3 θ
32
2 1 − t2

3 2
3 2 t2  t  3 2 3 π
So, 0 dt =  − arcsin t  = − arcsin = 3 − ≈ 0.685.
(1 − t )
32
2
 1−t
2
0 14 2 3

(b) When t = 0, θ = 0. When t = 3 2, θ = π 3. So, 1


t

3 2 t 2
π 3 π
0 dt =[tan θ − θ ]0 = 3 − ≈ 0.685. θ
(1 − t 2 )
32
3
1 − t2

38. Same substitution as in Exercise 37


1 cos θ dθ
   sec θ dθ =  ( tan θ + 1) sec2 θ dθ
4 2
(a) dt = =
(1 − t 2 ) cos5 θ
52

3
1 1 t  t
= tan 3 θ + tan θ + C =   + +C
3 3 1 − t2  1 − t2
3 2
 
3 2 1 t3 t 3 3 8 3 2
So, 0 dt =  +  = + = 3 + 3 = 2 3 ≈ 3.464.
3 1 − t 2 3 2 1 − t2 
(1 − t ) ( ) 3(1 4)
52 32
2 14
 0
(b) When t = 0, θ = 0. When t = 3 2, θ = π 3. So,
π 3
1 1  1
( 3)
3 2 3
0 dt =  tan 3 θ + tan θ  = + 3 = 2 3 ≈ 3.464.
(1 − t )
52
2 3 0 3

© 2018 Cengage Learning. All Rights Reserved. May not be scanned, copied or duplicated, or posted to a publicly accessible website, in whole or in part.
774 Chapter 8 Integration Techniques and Improper Integrals

39. (a) Let x = 3 tan θ , dx = 3 sec2 θ dθ , x 2 + 9 = 3 sec θ .

x3 (27 tan 3 θ )(3 sec2 θ dθ )


 x2 + 9
dx =  3 sec θ
x2 + 9
x

= 27  (sec 2 θ − 1) sec θ tan θ dθ θ


3
1 
= 27  sec3 θ − sec θ  + C = 9 sec3 θ − 3 sec θ  + C
 3 
 3

x2 + 9   x 2 + 9  1
 + C = ( x 2 + 9) − 9
32
= 9   − 3 x2 + 9 + C
 3   3  3
   
3
x3 1 
dx =  ( x 2 + 9) − 9
3 32
So, 0 2
x +9  3
x2 + 9 
0
1 
( )
=  54 2 − 27 2  − (9 − 27) = 18 − 9 2 = 9 2 −
 3 
( )
2 ≈ 5.272.

(b) When x = 0, θ = 0. When x = 3, θ = π 4. So,

x3
( ) ( )
3 π 4
0 dx = 9 sec3 θ − 3 sec θ  = 9 2 2 − 3 2 − 9(1 − 3) = 9 2 − 2 ≈ 5.272.
2 0
x +9

3
40. (a) Let 5 x = 3 sin θ , dx = cos θ dθ , 9 − 25 x 2 = 3 cos θ .
5
3
 9 − 25 x 2 dx =  (3 cos θ ) 5 cos θ dθ
9 1 + cos 2θ
5
= dθ
2
9 1 
= θ + sin 2θ  + C 3
10  2  5x

9
= (θ + sin θ cos θ ) + C θ
10 9 − 25x 2

9 5x 5x 9 − 25 x  2
=  arcsin + ⋅ +C
10  3 3 3 

35
35 9 5 x 5 x 9 − 25 x 2  9 π  9π
0
2
So, 9 − 25 x dx = arcsin +  =   = .
10  3 9  10  2  20
0

3 π
(b) When x = 0, θ = 0. When x = ,θ = .
5 2
π 2
35 9  9 π  9π
So, 0 9 − 25 x 2 dx =  (θ + sin θ cos θ ) =   = .
10 0 10  2  20

© 2018 Cengage Learning. All Rights Reserved. May not be scanned, copied or duplicated, or posted to a publicly accessible website, in whole or in part.
Section 8.4 Trigonometric Substitution 775

41. (a) Let x = 3 sec θ , dx = 3 sec θ tan θ dθ , x 2 − 9 = 3 tan θ .

x
x2 − 9
θ
3

x2 9 sec2 θ
 2
x −9
dx =  3 tan θ
3 sec θ tan θ dθ

= 9  sec3 θ dθ

1 1 
= 9 sec θ tan θ +  sec θ dθ  (8.3 Exercise 102 or Example 5, Section 8.2)
2 2 
9
= (sec θ tan θ + ln sec θ + tan θ )
2
9 x x2 − 9 x x2 − 9 
=  ⋅ + ln + 
2 3 3 3 3 
 
So,
6
6 x2 9x x2 − 9 x x2 − 9 
4 dx = 
2
+ ln + 

x2 − 9 9 3 3
 4

9  6 27 27   4 7 4 7 
=  + ln 2 +  − + ln + 
2  9 3   9 3 3 

9   6 + 27   4 + 7 
= 9 3 − 2 7 + ln   − ln  
2   3   3 
9 6 + 3 3
= 9 3 − 2 7 + ln   ≈ 12.644.
2  4 + 7 

 4 π
(b) When x = 4, θ = arcsec . When x = 6, θ = arcsec( 2) = .
 3 3
6 x2 9 π 3
4 2
x −9
dx =
2
sec θ tan θ + ln sec θ + tan θ  arcsec 4 3
( )

  7 
=
9
2
(
2⋅ 3 + ln 2 + 3 ) − 92  43   + ln
3 
4
3
+
3
7


 
9 6 + 3 3
= 9 3 − 2 7 + ln   ≈ 12.644
2  4 + 7 

© 2018 Cengage Learning. All Rights Reserved. May not be scanned, copied or duplicated, or posted to a publicly accessible website, in whole or in part.
776 Chapter 8 Integration Techniques and Improper Integrals

42. (a) Let x = 4 sec θ , dx = 4 sec θ tan θ dθ , x 2 − 16 = 4 tan θ .

x 2 − 16 4 tan θ
 x2
dx =  16 sec2 θ (4 sec θ tan θ ) dθ
tan 2 θ
=  sec θ

sin 2 θ
=  cos θ

1 − cos 2 θ
=  cos θ

=  sec θ dθ −  cos θ dθ
= ln sec θ + tan θ − sin θ + C

x x 2 − 16 x 2 − 16
= ln + − +C
4 4 x

So,
8
8 x 2 − 16  x x 2 − 16 x 2 − 16 
4 x 2
dx = ln
 4
+
4

x


4

  48  48 
= ln  2 +  −  − ln (1)
  4  8  

3
= ln 2 + ( )
3 −
2
.

x
x 2 − 16

θ
4

π
(b) When x = 4, θ = 0, and when x = 8, θ = . So,
3
8 x 2 − 16 π 3
4 x 2
dx = ln sec θ + tan θ − sin θ  0

3
= ln 2 + 3 − .
2

43. Substitution: u = x 2 + 1, du = 2 x dx

44. Trigonometric substitution: x = sec θ

© 2018 Cengage Learning. All Rights Reserved. May not be scanned, copied or duplicated, or posted to a publicly accessible website, in whole or in part.
Section 8.4 Trigonometric Substitution 777

45. (a) u-substitution: Let u = 1 − x 2 , du = − 2 x dx.


x 1
(1 − x ) (− 2 x) dx
−1 2

2
dx = −
1− x 2 2
1
(1 − x 2 ) (2) + C = − 1 − x 2 + C
12
= −
2
Trigonometric substitution
Let x = sin θ , dx = cos θ dθ , a = 1, 1 − x 2 = cos θ .
x sin θ
 1− x 2
dx =  cos θ cos θ dθ =  sin θ dθ
= − cos θ + C = − 1 − x 2 + C

1
x

1 − x2

The answers are equivalent.

x2 x2 + 9 − 9  9   x
(b)  x2 + 9
dx =  x2 + 9
dx =  1 − 2  dx = x − 3 arctan   + C
x + 9  3
Let x = 3 tan θ , x 2 + 9 = 9 sec 2 θ , dx = 3 sec 2 θ dθ .

x2 9 tan 2 θ
 x2 +9
dx =  9 sec2 θ 3 sec2 θ dθ

= 3 tan 2 θ dθ = 3 (sec2 θ − 1) dθ
= 3 tan θ − 3θ + C1
 x
= x − 3 arctan   + C1
 3
The answers are equivalent.

46. (a) The graph of f is increasing when 49. False


f ′ > 0 : 0 < x < ∞. 3 dx π 3 sec 2 θ dθ π 3
0 = 0 = 0 cos θ dθ
The graph of f is decreasing when
( )
3
sec3 θ
1 + x2
f ′ < 0 : −∞ < x < 0.
(b) The graph of f is concave upward when the graph of
50. True
f ′ is increasing. There are no such intervals.
1 1
 −1 x 1 − x 2 dx = 2 x 2 1 − x 2 dx
2
The graph of f is concave downward when the graph 0
of f ′ is decreasing: π 2

−∞ < x < 0 and 0 < x < ∞.


= 2
0
(sin 2 θ )(cos θ )(cos θ dθ )
π 2
= 2 sin 2 θ cos 2 θ dθ
47. True 0

dx cos θ dθ
 1− x 2
=  cos θ
=  dθ
48. False
x2 − 1 tan θ
  sec θ (sec θ tan θ dθ ) =  tan θ dθ
2
dx =
x

© 2018 Cengage Learning. All Rights Reserved. May not be scanned, copied or duplicated, or posted to a publicly accessible website, in whole or in part.
778 Chapter 8 Integration Techniques and Improper Integrals

a b 52. x 2 + y 2 = a 2
51. A = 4  a 2 − x 2 dx
0 a
x = ± a2 − y 2
4b a
a 0
2 2
= a − x dx a
A = 2 a 2 − y 2 dy
a h
 4b  1  x 
=    a 2 arcsin + x a 2 − x 2   
a
 a  2  a  0  y
= a 2 arcsin   + y a2 − y2  (Theorem 8.2)
 a h
2b  2  π  
=  a    = π ab  π  h 
a   2  =  a 2  −  a 2 arcsin   + h a 2 − h 2 
 2  a 
Note: See Theorem 8.2 for  a 2 − x 2 dx.
a 2π h
= − a 2 arcsin   − h a 2 − h 2
y b
y= a a2 − x2
2 a

x
−a a

−b

1 1 x2 + 1
, 1 + ( y′) = 1 + 2 =
2
53. y = ln x, y′ =
x x x2

Let x = tan θ , dx = sec2 θ dθ , x 2 + 1 = sec θ .

x2 + 1
5 5 x2 + 1
s = 1 x2
dx = 
1 x
dx

b sec θ b sec θ
=  sec 2 θ dθ =  a tan θ (1 + tan θ ) dθ
2
a tan θ

b b
=  a (csc θ + sec θ tan θ ) dθ = −ln csc θ + cot θ + sec θ  a x2 + 1
x
5
 2
x +1 1  θ
= −ln + + x 2 + 1
 
x x 1
1

  26 + 1  
= −ln 
  5
 + 26  − −ln
 
( 2 +1 + ) 2


5 2 + 1 
= ln 
(
 +
) 26 −

2 ≈ 4.367 or ln 
26 − 1

 + 26 − 2
 26 + 1 
 
5 2 − 1
 ( ) 

x2 x
54. y = − 2 x , y′ = − 2,
4 2
2
x  x2 x2 1
1 + ( y′) = 1 +  − 2  = 1 + − 2 x + 5 = ( x 2 − 8 x + 20)
2
− 2x + 4 =
2  4 4 4

1 2
( x − 8x + 20) dx = 12
8 8
s = 4 x 2 − 8 x + 16 + 4 dx
4 4
1 8
4 ( x − 4) (u = x − 4, a = 2)
2
= + 4 dx
2
8
1  1  
  ( x − 4) ( x − 4) + 4 + 4 ln ( x − 4) + (x − 4) + 4 
2 2
=
2  2   4
1
(
=  4 20 + 4 ln 4 + 20 − (0 + 4 ln 2)
4
(
 ))
≈ 5.916

© 2018 Cengage Learning. All Rights Reserved. May not be scanned, copied or duplicated, or posted to a publicly accessible website, in whole or in part.
Section 8.4 Trigonometric Substitution 779

1 − ( x − 3)
2
55. Let x − 3 = sin θ , dx = cos θ dθ , = cos θ .

Shell Method:
4
V = 4π  x 1 − ( x − 3) dx
2
y
2
π 2 2
= 4π 
−π 2
(3 + sin θ ) cos 2 θ dθ
1

3 π 2 π 2
= 4π   (1 + cos 2θ ) dθ +  − π 2 cos
2
θ sin θ dθ 1 3
x
1
x−3
 2 −π 2 −1
π 2 θ
3 1  1  −2
= 4π  θ + sin 2θ  − cos3 θ  = 6π 2 1 − (x − 3)2
 2  2  3  −π 2

r 2 − ( x − h)
2
56. Let x − h = r sin θ , dx = r cos θ dθ , = r cos θ .

Shell Method:
h+r
x r 2 − ( x − h) dx
2
V = 4π 
h−r
π 2 π 2
= 4π 
−π 2
(h + r sin θ )r cos θ ( r cos θ ) dθ = 4π r 2 
−π 2
(h + r sin θ ) cos 2 θ dθ

h π 2
(1 + cos 2θ ) dθ + r  − π 2 sin θ cos 2 θ dθ 
π 2
= 4π r 2  
2 − π 2

π 2 π 2
 1    cos3 θ 
= 2π r 2 h θ + sin 2θ  − 4π r 3   = 2π 2 r 2 h
 2  −π 2   3  −π 2
y

r
x−h

x θ
h−r h h+r
r 2 − (x − h) 2

57. Let x = 3 tan θ , dx = 3 sec2 θ dθ , x 2 + 9 = 3 sec θ .


4 3 4 dx b 3 sec 2 θ dθ
A = 2 dx = 6 = 6
0
x2 + 9 0
x2 + 9 a 3 sec θ
4
b  b x2 + 9 + x 
= 6  sec θ dθ = 6 ln sec θ + tan θ  a = 6 ln  = 6 ln 3
a
 3 
0

x = 0 ( by symmetry)
2 4
1 1  4  3  9 4 1 3 1 x 2 4
2  A  − 4  
y =     dx = 2
dx =  arctan  = arctan ≈ 0.422
12 ln 3 −4 x + 9 4 ln 3
2
x + 9  3 3  −4 4 ln 3 3

 1 4
(x, y) =  0, arctan  ≈ (0, 0.422)
 2 ln 3 3
y

3
4

1
2
(0, 0.422)
1
4

x
−4 −2 2 4

© 2018 Cengage Learning. All Rights Reserved. May not be scanned, copied or duplicated, or posted to a publicly accessible website, in whole or in part.
780 Chapter 8 Integration Techniques and Improper Integrals

58. First find where the curves intersect.


1 4
y 2 = 16 − ( x − 4) =
2
x
16
162 − 16( x − 4) = x 4
2

162 − 16 x 2 + 128 x − 162 = x 4


x 4 + 16 x 2 − 128 x = 0
x( x − 4)( x 2 + 4 x + 32) = 0
 x = 0, 4
4
4 1 1 1  16
π ( 4) =  x3  + 4π =
2
0 4x + 4π
2
A = dx +
4 12 0 3 y

4 1  8
16 − ( x − 4) dx
2
0 x x 2  dx + 4 x
6
My = (4, 4)
4  4 (4.89, 1.55)
2
4 4
x  8 8
 4 ( x − 4) 16 − ( x − 4) dx + 16 − ( x − 4) dx
2 2
4 4
x
=   + −2 2 4 6 10
16  0 −2

−4
8 8
 −1
( )   x − 4 2
32
= 16 +  16 − ( x − 4)  + 2 16 arcsin 4 + ( x − 4) 16 − ( x − 4) 
16 − (x − 4) 2
2 −6 y=
3 4  4

= 16 +
3
(16 ) + 216 π2  = 16 + 643 + 16π = 112
1 32
3
+ 16π
 
8
2
 1 x5 
4
 ( x − 4)3  = 32 +  64 − 64  − 32 = 416
Mx = 0
4 1 1 2 
 x  dx +
2 4  4
8 1
2
( )
16 − ( x − 4) dx =  ⋅  + 8 x −
2

 32 5  0  6  5

 6

15
4

My 112 3 + 16π 112 + 48π 28 + 12π


x = = = = ≈ 4.89
A 16 3 + 4π 16 + 12π 4 + 3π
Mx 416 15 104
y = = = ≈ 1.55
A (16 3) + 4π 5(4 + 3π )
(x, y) ≈ ( 4.89, 1.55)

59. (a) Place the center of the circle at (0, 1); x 2 + ( y − 1) = 1. The depth d satisfies 0 ≤ d ≤ 2. The volume is
2

d
1
arcsin ( y − 1) + ( y − 1) 1 − ( y − 1) 
d
V = 3 ⋅ 2
0
1 − ( y − 1) dy = 6 ⋅
2

2
2

0
(Theorem 8.2 (1))
= 3arcsin ( d − 1) + ( d − 1) 1 − ( d − 1) − arcsin ( −1)
2

 

= + 3 arcsin ( d − 1) + 3( d − 1) 2d − d .
2
2
10
(b)

0 2
0

(c) The full tank holds 3π ≈ 9.4248 cubic meters. The horizontal lines
3π 3π 9π
y = ,y = ,y =
4 2 4
intersect the curve at d = 0.596, 1.0, 1.404. The dipstick would have these markings on it.

© 2018 Cengage Learning. All Rights Reserved. May not be scanned, copied or duplicated, or posted to a publicly accessible website, in whole or in part.
Section 8.4 Trigonometric Substitution 781

d
1 − ( y − 1) dy
2
(d) V = 6
0

dV dV dd 1 1
= 6 1 − ( d − 1) ⋅ d ′(t ) =  d ′(t ) =
2
= ⋅
dt dd dt 4 24 1 − ( d − 1)
2

0.3
(e)

0 2
0

The minimum occurs at d = 1, which is the widest part of the tank.

60. Let r = L tan θ , dr = L sec 2 θ dθ , r 2 + L2 = L2 sec 2 θ .


b R
1 R 2mL 2mL b L sec2 θ dθ 2m b  2m   2m r  2m

R ( r 2 + L2 )
0 32
dr =
R  a L sec θ
3 3
=
RL  a
cos θ dθ = 
 RL
sin θ  = 
a  RL 2 2
 =
r + L 0 L R 2 + L2

r2 + L2
r

θ
L

dy y ( 0, y + 144 − x 2 (
61. (a) m =
dx 12

=
y − y + ( 144 − x 2 ) 12
x −0 144 − x 2

(x, y)
144 − x 2
= − y x
x x
2 4 6 8 10 12

144 − x 2
(b) y = −  dx
x

Let x = 12 sin θ , dx = 12 cos θ dθ , 144 − x 2 = 12 cos θ .

12 cos θ 1 − sin 2θ
y = − 12 cos θ dθ = −12 dθ
12 sin θ sin θ
= −12  (csc θ − sin θ ) dθ = −12 ln csc θ − cot θ − 12 cos θ + C

12 144 − x 2  144 − x 2  12 − 144 − x 2


= −12 ln − − 12  + C = −12 ln − 144 − x 2 + C
x x  12  x
 
 12 − 144 − x 2 
When x = 12, y = 0  C = 0. So, y = −12 ln  − 144 − x 2 .
 x 
 
12 − 144 − x 2
Note: > 0 for 0 < x ≤ 12
x
30

12
x

144 − x 2 0 12
0

© 2018 Cengage Learning. All Rights Reserved. May not be scanned, copied or duplicated, or posted to a publicly accessible website, in whole or in part.
782 Chapter 8 Integration Techniques and Improper Integrals

(c) Vertical asymptote: x = 0


(d) y + 144 − x 2 = 12  y = 12 − 144 − x 2
 12 − 144 − x 2 
So, 12 − 144 − x 2 = −12 ln  − 144 − x 2
 x 
 
 12 − 144 − x 2 
−1 = ln  
 x 
 
xe−1 = 12 − 144 − x 2

( )
2
( xe−1 − 12)
2
= − 144 − x 2

x 2e −2 − 24 xe −1 + 144 = 144 − x 2
x 2 (e −2 + 1) − 24 xe−1 = 0

x  x(e −2 + 1) − 24e −1  = 0
24e −1
x = 0 or x = ≈ 7.77665.
e −2 + 1

x 2 + (144 − x 2 )
2
 144 − x 2 
12 12
Therefore, s =  1 + −  dx =  dx
7.77665  x  7.77665 x2
 
12 12 12
=  dx = 12 ln x  7.77665 = 12(ln 12 − ln 7.77665) ≈ 5.2 meters.
7.77665 x

62. (a) Distance along line:

(a 2 − 0) + ( a − 0)
2 2
d = = a4 + a2 = a a2 + 1
y

(a, a 2)

y = x2

x
(0, 0)

Distance along y = x 2 :

y′ = 2 x, 1 + ( y′) = 1 + 4 x 2
2

d = 0
a
1 + 4 x 2 dx =
1
4 
ln ( )
4a 2 + 1 + 2a + 2a 4a 2 + 1

(b)
a Line Parabola
1 2 ≈ 1.4142 1.4789
10 100.4988 101.0473
100 10,000.5 10,001.6

(c) The difference between the distances approaches 0 as a increases.

© 2018 Cengage Learning. All Rights Reserved. May not be scanned, copied or duplicated, or posted to a publicly accessible website, in whole or in part.
Section 8.4 Trigonometric Substitution 783

63. (a) Area of representative rectangle: 2 1 − y 2 Δy

Force: 2(9800)(3 − y ) 1 − y 2 Δy
1
F = 19,600 
−1
(3 − y ) 1 − y 2 dy

= 19,600 3 1 − y 2 dy 
1 1

 −1
1 − y 2 dy −  −1 y 
1
3
2
( 1 2  3 2
)
= 19,600  arcsin y + y 1 − y 2 +  (1 − y 2 )  = (9800)3arcsin 1 − arcsin ( −1) = 29,400π N
2 3   −1
y

2
x= 1 − y2

x
−2 2

1 1 1
(b) F = 19,600 
−1
(d − y ) 1 − y 2 dy = 19,600d  −1 1 − y 2 dy − 19,600 
−1
y 1 − y 2 dy

d  1
= 19,600   arcsin y + y 1 − y 2  − 19,600(0) = 9800π d N
 2   −1

0.24
64. (a) Finside = 7540 
− 0.3
(0.24 − y )( 2) 0.09 − y 2 dy

= 15,080 0.24  0.09 − y 2 dy


0.24 0.24

 − 0.3
0.09 − y 2 dy −  − 0.3 y 
0.24

(0.09 − y 2 ) 
32
0.24  y 2
= 15,080  0.09 arcsin + y 0.09 − y +
 2  0.3

 3 
  − 0.3
≈ 15,080 (0.03410739) ≈ 514.3 N
0.12
(b) Foutside = 10,045 
− 0.3
(0.12 − y )( 2) 0.09 − y 2 dy

= 20,090 0.12  0.09 − y 2 dy 


0.12 0.12
 − 0.3
0.09 − y 2 dy −  − 0.3 y 
0 .12
 
 (0.09 − y )
32
2
0.12  y
= 20,090  0.09 arcsin + y 0.09 − y 2  + 
 2  0.3  3 
  − 0.3
≈ 20,090 (0.01961302) ≈ 394.0 N

© 2018 Cengage Learning. All Rights Reserved. May not be scanned, copied or duplicated, or posted to a publicly accessible website, in whole or in part.
784 Chapter 8 Integration Techniques and Improper Integrals

65. Let u = a sin θ , du = a cos θ dθ , a 2 − u 2 = a cos θ .


1 + cos 2 θ
 a 2 − u 2 du = a cos 2 θ dθ = a 2  dθ
2
2
a2  1  a2
= θ + sin 2 θ  + C = (θ + sin θ cos θ ) + C
2 2  2

a2  u  u  a 2 + u 2  1 u 
= arcsin +    + C =  a 2 arcsin + u a2 − u2  + C
2 a  a  a 
 2  a 

Let u = a sec θ , du = a sec θ tan θ dθ , u 2 − a 2 = a tan θ .

 u 2 − a 2 du =  a tan θ (a sec θ tan θ ) dθ = a  tan θ sec θ dθ


2 2

= a 2  (sec 2 θ − 1) sec θ dθ = a 2  (sec3 θ − sec θ ) dθ

1 1  1 1 
= a 2  sec θ tan θ +  sec θ dθ  − a 2  sec θ dθ = a 2  sec θ tan θ − ln sec θ + tan θ 
2 2  2 2 
a2 u u 2 − a2 u u2 − a2  1
=  ⋅ − ln +  + C1 = u u 2 − a 2 − a 2 ln u + u2 − a2  + C
2 a a a a  2  

Let u = a tan θ , du = a sec2 θ dθ , u 2 + a 2 = a sec θ .

 u 2 + a 2 du =  (a sec θ )(a sec θ ) dθ


2

1 1 
= a 2  sec3 θ dθ = a 2  sec θ tan θ + ln sec θ + tan θ  + C1
2 2 
a2  u 2 + a2 u u 2 + a2 u 1
=  ⋅ + ln +  + C1 = u u 2 + a 2 + a 2 ln u + u 2 + a2  + C
2 a a a a 2  
 

66. y = sin x on [0, 2]


y′ = cos x
π
s1 = 2 
0
1 + cos 2 x dx (≈ 3.820197789)

Ellipse: x 2 + 2 y 2 = 2

1 2
Upper half: y = 1− x , − 2 ≤ x ≤ 2
2
−x
y′ =
2 1 − (1 2) x 2

2 x2 2 x2
s2 = 2  1+ dx = 2 1+ dx
− 2 4(1 − (1 2) x 2
) − 2 4 − 2 x2

Let x = 2 sin θ , dx = 2 cos θ dθ , x 2 = 2 sin 2 θ , 4 − 2 x 2 = 4 − 4 sin 2 θ = 4 cos 2 θ .

π 2 2 sin 2 θ
s2 = 2  1+ 2 cos θ dθ
−π 2 4 cos 2 θ
π 2 4 cos 2 θ + 2 sin 2 θ
= 2 2 cos θ dθ
−π 2 2 cos θ
π 2 2 + 2 cos 2 θ π 2 π
= 2 dθ = 2  1 + cos 2 θ dθ = 2  1 + cos 2 θ dθ = s1
−π 2 2 −π 2 0

© 2018 Cengage Learning. All Rights Reserved. May not be scanned, copied or duplicated, or posted to a publicly accessible website, in whole or in part.
Section 8.4 Trigonometric Substitution 785

67. Large circle: x 2 + y 2 = 25


y = 25 − x 2 , upper half
From the right triangle, the center of the small circle is (0, 4).

x 2 + ( y − 4) = 9
2

y = 4+ 9 − x2 , upper half

A = 2  4 +
3

0  ( 9 − x2 − ) 25 − x 2  dx

3
 1  x  1  x 
= 2 4 x + 9 arcsin   + x 9 − x 2  − 25 arcsin   + x 25 − x 2 
 2  3  2 5  0
 9 25 3 
= 2 12 + arcsin (1) − arcsin − 6
 2 2 5 
9π 3
= 12 + − 25 arcsin ≈ 10.050
2 5
y

8
6
4 (3, 4)
2
x
−6 −2 2 4 6
−2
−4
−6

68. The left circle has equation ( x + 2) + y 2 = 9. The shaded area is four times the area in the first quadrant, under the curve
2

9 − ( x + 2) .
2
y =
1
9 − ( x + 2) dx
2
A = 4
0

9 − ( x + 2)
2
Let x + 2 = 3 sin θ , dx = 3 cos θ dθ , = 3 cos θ

3 x+2

9 − (x + 2) 2

1 + cos 2θ
9 − ( x + 2) dx =  3 cos θ (3 cos θ ) dθ
2
 = 9
2

9 sin 2θ  9
= θ +  + C = (θ + sin θ cos θ ) + C
2 2  2
  9 − ( x + 2) 
2
9  x + 2   x + 2 
= arcsin   +  +C
2   3   3  3 
  
1
  2 
9  x + 2   x + 2  9 − ( x + 2)   π   2 2 5  2
A = 4 ⋅ arcsin   +  = 18 + 0  −  arcsin +  = 9π − 18 arcsin − 4 5
2  3   3  3 
   2   3 3 3  3
   0

© 2018 Cengage Learning. All Rights Reserved. May not be scanned, copied or duplicated, or posted to a publicly accessible website, in whole or in part.
786 Chapter 8 Integration Techniques and Improper Integrals

1 ln ( x + 1)
69. Let I = 0 x2 + 1
dx

1−u −2
Let x = , dx = du
(1 + u )
2
1+u

2 2 + 2u 2
x +1= , x2 + 1 =
1+u (1 + u )2
 2 
ln  
I =   1 + u   −2  du
0

2 + 2u 2   (1 + u ) 
1  2

 
 (1 + u )2 
 
 2   2 
−ln   ln  
0  1 + u  du = 1  1 + u  du = 1 ln 2 1 ln (1 + u )
du = (ln 2)[arctan u]0 − I
1
=    0 1 + u2 − 0
1 1 + u2 0 1 + u2 1 + u2
π 
 2 I = ln 2  
4
π
I = ln 2 ≈ 0.272198
8

Section 8.5 Partial Fractions


4 4 A B 2 2 A B
1. (a) = = + 4. = = +
2
x − 8x x( x − 8) x x −8 2
9x − 1 (3x − 1)(3x + 1) 3x − 1 3x + 1
2
2x + 1 A B C 2 = A(3 x + 1) + B(3 x − 1)
(b) = + +
(x − 3) x − 3 ( x − 3) ( x − 3)
3 2 3
1
When x = , 2 = 2 A  A = 1.
2x − 3 2x − 3 A Bx + C 3
(c) = = + 2 1
3
x + 10 x x( x + 10)
2
x x + 10 When x = − , 2 = −2 B  B = −1.
3
2x − 1 A Bx + C Dx + E 2 1 −1
(d) = + 2 +
x( x 2 + 1)
2
x x +1 ( x 2 + 1)
2
 9 x2 −1
dx =  3x − 1 dx +  3x + 1 dx
1 1
= ln 3x − 1 − ln 3 x + 1 + C
2. For a basic equation involving quadratic factors, you will 3 3
have to solve a system of linear equations. 1 3x − 1
= ln +C
3 3x + 1
1 1 A B
3. = = +
x −92
( x − 3)( x + 3) x + 3 x − 3 5 5 A B
5. = = +
1 = A( x − 3) + B( x + 3) x 2 + 3x − 4 ( x + 4)( x − 1) x + 4 x − 1
1 5 = A( x − 1) + B( x + 4)
When x = 3, 1 = 6B  B = .
6 When x = 1, 5 = 5 B  B = 1.
1
When x = −3, 1 = −6 A  A = − . When x = − 4, 5 = −5 A  A = −1.
6
5 −1 1

1 1
dx = − 
1
dx + 
1 1
dx  x2 + 3x − 4
dx = 
x + 4
dx + 
x −1
dx
x2 − 9 6 x + 3 6 x −3
= −ln x + 4 + ln x − 1 + C
1 1
= − ln x + 3 + ln x − 3 + C
6 6 x −1
= ln +C
1 x −3 x + 4
= ln +C
6 x + 3

© 2018 Cengage Learning. All Rights Reserved. May not be scanned, copied or duplicated, or posted to a publicly accessible website, in whole or in part.
Section 8.5 Partial Fractions 787

3− x 3− x A B
6. = = +
2
3x − 2 x − 1 (3x + 1)( x − 1) 3x + 1 x − 1
3 − x = A( x − 1) + B(3x + 1)
1
When x = 1, 2 = 4B  B = .
2
1 10 4 5
When x = − , = − A  A = − .
3 3 3 2
3− x 5 1 1 1
 3x 2 − 2x − 1
dx = − 
2 3x + 1
dx + 
2 x −1
dx

−5 1
= ln 3 x + 1 + ln x − 1 + C
6 2

x 2 + 12 x + 12 A B C
7. = + +
x( x + 2)( x − 2) x x+ 2 x−2
x 2 + 12 x + 12 = A( x + 2)( x − 2) + Bx( x − 2) + Cx( x + 2)
When x = 0, 12 = − 4 A  A = −3.
When x = −2, − 8 = 8B  B = −1.
When x = 2, 40 = 8C  C = 5.

x 2 + 12 x + 12 1 1 1
 x3 − 4 x
dx = 5
x − 2
dx −  x + 2 dx − 3 x dx = 5 ln x − 2 − ln x + 2 − 3 ln x + C

x3 − x + 3 2x + 1 A B
8. = x −1+ = x −1+ +
x2 + x − 2 ( x + 2)( x − 1) x + 2 x −1
2 x + 1 = A( x − 1) + B( x + 2)
When x = −2, − 3 = −3 A  A = 1.
When x = 1, 3 = 3B  B = 1.

x3 − x + 3  1 1  x2 x2
 x2 + x −2
dx =   x − 1 + x+ 2
+  dx =
x − 1 2
− x + ln x + 2 + ln x − 1 + C =
2
− x + ln x 2 + x − 2 + C

2 x3 − 4 x 2 − 15 x + 5 x+5 A B
9. = 2x + = 2x + +
x2 − 2 x − 8 ( x − 4)( x + 2) x−4 x + 2
x + 5 = A( x + 2) + B( x − 4)
3
When x = 4, 9 = 6 A  A = .
2
1
When x = −2, 3 = −6 B  B = − .
2
2 x 3 − 4 x 2 − 15 x + 5  32 12  3 1
   2 x +
2
dx = −  dx = x + ln x − 4 − ln x + 2 + C
x2 − 2x − 8 x − 4 x + 2 2 2

© 2018 Cengage Learning. All Rights Reserved. May not be scanned, copied or duplicated, or posted to a publicly accessible website, in whole or in part.
788 Chapter 8 Integration Techniques and Improper Integrals

x + 2 x + 2 A B 4 x2 + 2 x − 1 A B C
10. = = + 11. = + 2 +
2
x + 5x x( x + 5) x x +5 x 2 ( x + 1) x x x +1
x + 2 = A( x + 5) + Bx 4 x 2 + 2 x − 1 = Ax( x + 1) + B( x + 1) + Cx 2
3 When x = 0, B = −1.
When x = − 5, − 3 = − 5 B  B = .
5 When x = −1, C = 1.
2
When x = 0, 2 = 5A  A = . When x = 1, A = 3.
5
x + 2 2 1 3 1 4 x2 + 2x − 1 3 1 1 
 x2 + 5x
dx =  dx + 
5 x 5 x +5
dx  x3 + x 2
dx =   x x

2
+  dx
x + 1
2 3 1
= ln x + ln x + 5 + C = 3 ln x + + ln x + 1 + C
5 5 x
1
= + ln x + x3 + C
4
x

5x − 2 A B
12. = +
(x − 2) x − 2 ( x − 2)2
2

5 x − 2 = A( x − 2) + B
When x = 2, 8 = B.
When x = 0, − 2 = − 2 A + B = − 2 A + 8  A = 5.
5x − 2 5 8
 ( x − 2)2 dx =  x − 2 dx +  ( x − 2)2 dx
8
= 5ln x − 2 − +C
x − 2

x2 − 6x + 2 x2 − 6x + 2 x2 − 6 x + 2 A B C
13. = = = + +
3 2
x + 2x + x x( x + 2 x + 1)
2
x( x + 1)
2
x x + 1 ( x + 1)2

x 2 − 6 x + 2 = A( x + 1) + Bx( x + 1) + Cx
2

When x = 0, A = 2.
When x = −1, C = − 9.
When x = 1, − 3 = 2( 4) + 2 B − 9  B = −1.

x2 − 6x + 2 2 1 9 
 x3 + 2 x 2 + x
dx =   x − −
x + 1 ( x + 1) 2
 dx
 
9
= 2 ln x − ln x + 1 + +C
x +1
x2 9
= ln + +C
x +1 x +1

© 2018 Cengage Learning. All Rights Reserved. May not be scanned, copied or duplicated, or posted to a publicly accessible website, in whole or in part.
Section 8.5 Partial Fractions 789

8x 8x 8x
14. = 2 =
3 2
x + x − x −1 x ( x + 1) − ( x + 1) ( x + 1)( x − 1)( x + 1)
A B C
= + +
x − 1 x + 1 ( x + 1)2

8 x = A( x + 1) + B( x − 1)( x + 1) + C ( x − 1)
2

When x = 1, 8 = 4A  A = 2.
When x = −1, − 8 = − 2C  C = 4.
When x = 0, 0 = A − B − C = 2 − B − 4  B = − 2.
8x 2 −2 4
 x3 + x2 − x − 1
dx =  x − 1 dx +  x + 1 dx +  ( x + 1)2 dx
4
= 2ln x − 1 − 2ln x + 1 − + C
x +1

9 − x2 9 − x2 A Bx + C
15. = = +
3
7x + x x(7 x 2 + 1) x 7 x2 + 1

9 − x 2 = A(7 x 2 + 1) + Bx 2 + Cx

When x = 0, A = 9.
When x = 1, 8 = 9(8) + B + C  B + C = − 64.

When x = −1, 8 = 9(8) + B − C  B − C = − 64

2 B = −128, B = − 64 and C = 0.

9 − x2 9 64 x 
 7 x3 + x
dx =   x −
7 x2 + 1
 dx

32
= 9 ln x − ln (7 x 2 + 1) + C
7

6x 6x A Bx + C
16. = = + 2
x3 − 8 ( x − 2)( x + 2 x + 4)
2
x − 2 x + 2x + 4

6 x = A( x 2 + 2 x + 4) + ( Bx + C )( x − 2)

When x = 2, 12 = 12 A  A = 1.
When x = 0, 0 = 4 − 2C  C = 2.
When x = 1, 6 = 7 + ( B + 2)( −1)  B = −1.

6x 1 −x + 2
 x3 −8
dx =  x − 2 dx +  x 2 + 2x + 4
dx

1 −x − 1 3
=  x − 2 dx +  x2 + 2x + 4
dx +  ( x2 + 2 x + 1) + 3
dx

1 3  x + 1
= ln x − 2 − ln x 2 + 2 x + 4 + arctan   +C
2 3  3 
1  3 ( x + 1) 
= ln x − 2 − ln x 2 + 2 x + 4 + 3 arctan   +C
2  3 
 

© 2018 Cengage Learning. All Rights Reserved. May not be scanned, copied or duplicated, or posted to a publicly accessible website, in whole or in part.
790 Chapter 8 Integration Techniques and Improper Integrals

x2 A B Cx + D
17. = + + 2
x − 2x2 − 8
4
x − 2 x + 2 x + 2
x 2 = A( x + 2)( x 2 + 2) + B( x − 2)( x 2 + 2) + (Cx + D)( x + 2)( x − 2)

When x = 2, 4 = 24 A.
When x = −2, 4 = −24 B.
When x = 0, 0 = 4 A − 4 B − 4 D.
When x = 1, 1 = 9 A − 3B − 3C − 3D.
1 1 1
Solving these equations you have A = , B = − , C = 0, D = .
6 6 3
x2 1 1 1 1  1 x − 2 x 
 x4 − 2 x2 − 8
dx =  
6 x − 2
dx −  x + 2 dx + 2 x2 dx  =  ln
+ 2  6 x + 2
+ 2 arctan +C
2

x A B Cx + D
18. = + +
(2 x − 1)( 2 x + 1)( 4 x 2 + 1) 2x − 1 2x + 1 4x2 + 1

x = A( 2 x + 1)( 4 x 2 + 1) + B( 2 x − 1)( 4 x 2 + 1) + (Cx + D)( 2 x − 1)( 2 x + 1)

1 1
When x = , = 4 A.
2 2
1 1
When x = − , − = −4 B.
2 2
When x = 0, 0 = A − B − D.
When x = 1, 1 = 15 A + 5B + 3C + 3D.
1 1 1
Solving these equations you have A = , B = , C = − , D = 0.
8 8 2
x 1 1 1 x  1 4x2 − 1
 16 x 4 −1
dx =
8  2x − 1
 dx +  2 x + 1 dx − 4 4 x 2 dx  =
+ 1  16
ln
4x2 + 1
+C

x2 + 5 A Bx + C
19. = +
( x + 1)( x 2 − 2 x + 3) x + 1 x 2 − 2 x + 3
x 2 + 5 = A( x 2 − 2 x + 3) + ( Bx + C )( x + 1)
= ( A + B) x 2 + ( −2 A + B + C ) x + (3 A + C )
When x = −1, A = 1.
By equating coefficients of like terms, you have A + B = 1, − 2 A + B + C = 0, 3 A + C = 5.
Solving these equations you have A = 1, B = 0, C = 2.

x2 + 5 1 1  x − 1
 x3 − x2 + x + 3
dx =  x + 1 dx + 2 ( x − 1)2 + 2
dx = ln x + 1 + 2 arctan 
 2 
+C

© 2018 Cengage Learning. All Rights Reserved. May not be scanned, copied or duplicated, or posted to a publicly accessible website, in whole or in part.
Section 8.5 Partial Fractions 791

x2 + 6x + 4 x2 + 6 x + 4 Ax + B Cx + D
20. = = 2 +
( ) ( x 2 + 4)
4 2 2 2
x + 8 x + 16 x 2
+ 4 x + 4

x 2 + 6 x + 4 = ( Ax + B)( x 2 + 4) + Cx + D
= Ax3 + Bx 2 + ( 4 A + C ) x + 4 B + D
By equating coefficients of like terms, you have
A = 0, B = 1, 4 A + C = 6, 4 B + D = 4.
Solving these equations you have A = 0, B = 1, C = 6, D = 0.

x2 + 6x + 4 1 6x
 x4 dx =  x2 dx +  dx
(x + 4)
2
+ 8 x 2 + 16 + 4 2

1 x 3
= arctan − 2 + C
2 2 x + 4

3 3 A B x +1 A Bx + C
21. = = + 23. = + 2
4 x2 + 5x + 1 (4 x + 1)( x + 1) 4 x + 1 x + 1 x( x 2 + 1) x x +1
3 = A( x + 1) + B( 4 x + 1) x + 1 = A( x 2 + 1) + ( Bx + C ) x
When x = −1, 3 = −3B  B = −1. When x = 0, A = 1.
1 3 When x = 1, 2 = 2 A + B + C.
When − , 3 = A  A = 4.
4 4
When x = −1, 0 = 2 A + B − C.
2 3 2 4 2 −1
0 4 x2 + 5x + 1
dx = 0 4x + 1
dx + 0 x +1
dx Solving these equations we have
2 A = 1, B = −1, C = 1.
= ln 4 x + 1 − ln x + 1  0
2 x +1 21 2 x 2 1
= ln 9 − ln 3  1 x( x 2 + 1) dx = 1 x
dx − 1 x2 + 1
dx + 1 x2 + 1
dx
= 2 ln 3 − ln 3 = ln 3
2
 1 
= ln x − ln( x 2 + 1) + arctan x
x −1 A B C  2 1
22. = + 2 +
x ( x + 1)
2
x x x +1 1 8 π
= ln − + arctan 2
x − 1 = Ax( x + 1) + B( x + 1) + Cx 2 2 5 4
≈ 0.557
When x = 0, B = −1.
When x = −1, C = −2. 1 x2 − x 1 1 2x + 1
24.  0 x2 + x +1
dx =  0 dx −  0 x 2 + x +1
dx
When x = 1, 0 = 2 A + 2 B + C. 1
=  x − ln x 2 + x + 1 
Solving these equations you have 0
A = 2, B = −1, C = −2. = 1 − ln 3
5 x −1 5 1 5 1 5 1
 1 x 2 ( x + 1) dx = 2 1 x
dx − 1 x2
dx − 2
1 x +1
dx

5
 1 
= 2 ln x + − 2 ln x + 1
 x 1
5
 x 1
= 2 ln +
 x + 1 x 1
5 4
= 2 ln −
3 5

© 2018 Cengage Learning. All Rights Reserved. May not be scanned, copied or duplicated, or posted to a publicly accessible website, in whole or in part.
792 Chapter 8 Integration Techniques and Improper Integrals

25. Let u = cos x, du = −sin x dx. 1 A B


28. = + , u = tan x, du = sec 2 x dx
1 A B u (u + 1) u u +1
= +
u (u + 1) u u +1 1 = A(u + 1) + Bu
1 = A(u + 1) + Bu When u = 0, A = 1.
When u = 0, A = 1. When u = −1, 1 = − B  B = −1.
When u = −1, B = −1. sec 2 x dx 1
 tan x( tan x + 1) =  u(u + 1) du
sin x 1
 cos x + cos2 x dx = −
u (u + 1)
du
1 1 
1 1
=   u −  du
u + 1
=  u + 1 du −  u du = ln u − ln u + 1 + C
= ln u + 1 − ln u + C u
= ln +C
u +1 u +1
= ln +C
u tan x
= ln +C
cos x + 1 tan x + 1
= ln +C
cos x
29. Let u = e x , du = e x dx.
= ln 1 + sec x + C
1 A B
= +
26.  2
5 cos x
dx = 5 2
1
du ( u − 1)( u + 4) u − 1 u + 4
sin x + 3 sin x − 4 u + 3u − 4
1 = A(u + 4) + B(u − 1)
u −1
= ln +C 1
u + 4 When u = 1, A = .
5
−1 + sin x
= ln +C 1
4 + sin x When u = −4, B = − .
5
(From Exercise 5 with u = sin x, du = cos x dx)
ex 1
27. Let u = tan x, du = sec x dx. 2  (e x − 1)(e x + 4)
dx =  (u − 1)(u + 4) du
1 1 A B 1 1 1 
5  u − 1  u + 4 du 
= = + =  du −
u 2 + 5u + 6 ( u + 3)( u + 2) u + 3 u + 2
1 = A(u + 2) + B(u + 3) 1 u −1
= ln +C
5 u + 4
When u = −2, 1 = B.
1 ex − 1
When u = −3, 1 = − A  A = −1. = ln x +C
5 e + 4
sec 2 x 1
 tan 2 x + 5 tan x + 6 dx =  u 2 + 5u + 6 du
−1 1
=  u + 3 du +  u + 2 du
= −ln u + 3 + ln u + 2 + C
tan x + 2
= ln +C
tan x + 3

© 2018 Cengage Learning. All Rights Reserved. May not be scanned, copied or duplicated, or posted to a publicly accessible website, in whole or in part.
Section 8.5 Partial Fractions 793

30. Let u = e x , du = e x dx.


1 A Bu + C
= + 2
(u 2
+ 1)( )
u − 1 u − 1 u +1

1 = A(u 2 + 1) + ( Bu + C )(u − 1)

1
When u = 1, A = .
2
When u = 0, 1 = A − C.
When u = −1, 1 = 2 A + 2 B − 2C.
1 1 1
Solving these equations you have A = , B = − , and C = − .
2 2 2
ex 1
 (e 2 x + 1)(e x − 1)
dx =  (u 2 + 1)(u − 1)
du

1 1 u +1 
2  u − 1  u2
=  du − du 
+1 
1 1 2 
=  ln u − 1 − ln u + 1 − arctan u  + C
2 2 
1
=
4
(
2 ln e x − 1 − ln e2 x + 1 − 2 arctan e x + C )
31. Let u = x , u 2 = x, 2u du = dx.

x u ( 2u )du  2u 2 − 8 8   8 
 x − 4 dx =  u2 − 4
=   u 2−4
+ 2  du =
u − 4   2 +  du
u2 − 4 
8 8 A B
= = +
u2 − 4 ( u − 2)( u + 2) u − 2 u + 2
8 = A(u + 2) + B(u − 2)
When u = −2, 8 = −4 B  B = −2.
When u = 2, 8 = 4 A  A = 2.

 8   2 2 
  2 +  du = 2u +
u2 − 4    u − 2 −  du
u + 2
= 2u + 2 ln u − 2 − 2 ln u + 2 + C

x − 2
= 2 x + 2 ln +C
x + 2

© 2018 Cengage Learning. All Rights Reserved. May not be scanned, copied or duplicated, or posted to a publicly accessible website, in whole or in part.
794 Chapter 8 Integration Techniques and Improper Integrals

32. Let u = x , u 2 = x, 2u du = dx.


1 2u du 2
 dx =  u2 = u du
x ( 3 − x ) ( 3 −u ) ( 3 −u )
2 A B
= +
u ( 3 −u ) u 3 −u

2 = A ( )
3 − u + Bu

2 2 3
When u = 0, 2 = 3A  A = = .
3 3

2 3
When u = 3, 2 = B 3  B = .
3
2 2 3 1 1 
u du =   u +  du
( 3 −u ) 3 3 − u

=
2 3
3
(
ln u − ln u − 3 )+C
 x ( 3 −
1
x )
dx =
2 3
3
ln ( x − ln x −3 +C )
1 A B x A B
33. = + 35. = +
x( a + bx) x a + bx (a + bx)
2
a + bx ( a + bx)2
1 = A( a + bx) + Bx x = A( a + bx) + B
When x = 0, 1 = aA  A = 1 a. When x = − a b, B = − a b.
When x = − a b, 1 = −( a b) B  B = −b a. When x = 0, 0 = aA + B  A = 1 b.

1 1 1 b  x  1b 
−a b
 x(a + bx) dx =
a x
 −  dx
a + bx   (a + bx)2 dx =   a + bx +  dx
(a + bx) 
2

1 1 1 a 1
= (ln x − ln a + bx )+C =  dx −  dx
a b a + bx b ( a + bx)2
1 x 1 a 1 
= ln +C = ln a + bx + 2 
a a + bx  +C
b2 b  a + bx 
1 a 
1 A B = 2 + ln a + bx  + C
34. = + b  a + bx 
a2 − x2 a − x a + x
1 = A( a + x) + B( a − x)
When x = a, 1 = 2aA  A = 1 2a.
When x = −a, 1 = 2aB  B = 1 2a.

1 1  1 1 
 a2 − x 2
dx =  
2a  a − x
+  dx
a + x
1
=
2a
(−ln a − x + ln a + x )+C
1 a + x
= ln +C
2a a − x

© 2018 Cengage Learning. All Rights Reserved. May not be scanned, copied or duplicated, or posted to a publicly accessible website, in whole or in part.
Section 8.5 Partial Fractions 795

1 A B C 15 15 A B
36. = + 2 + 42. = = +
x ( a + bx)
2
x x a + bx 2
x + 7 x + 12 ( x + 3)( x + 4) x + 3 x + 4
1 = Ax( a + bx) + B( a + bx) + Cx 2 15 = A( x + 4) + B( x + 3)

When x = 0, 1 = Ba  B = 1 a. When x = −a b, When x = − 3, A = 15.


1 = C(a b 2 2
)C 2 2
= b a . When x = 1, When x = − 4, B = −15.

1 = ( a + b) A + ( a + b) B + C  A = −b a 2 . 2 15
A = 1 x 2 + 7 x + 12
dx
1  −b a 2 1 a b2 a 2 
 x 2 (a + bx) dx =   x
+ 2 +
x
 dx
a + bx  =
2
1
 15
 −
15 
 dx
 x + 3 x + 4
b 1 b 2
= − ln x − + ln a + bx + C = 15ln ( x + 3) − ln ( x + 4)1
a2 ax a2
1 b a + bx = 15(ln 5 − ln 6) − (ln 4 − ln 5)
= − + ln +C
ax a 2 x
5(5) 25
1 b x = 15 ln = 15 ln ≈ 0.612
= − − ln +C 6( 4) 24
ax a 2 a + bx
15 −15 A B
37. Substitution: u = x 2 + 2 x − 8 43. = = +
9 − x2 ( x − 3)( x + 3) x − 3 x + 3
38. Partial fractions −15 = A( x + 3) + B( x − 3)

39. Trigonometric substitution (tan) or inverse tangent rule 5


When x = − 3, −15 = − 6 B  B = .
2
40. (a) Yes. Because f ' > 0 on (0, 5), f is increasing, and
5
When x = 3, −15 = 6 A  A = − .
f (3) > f ( 2). Therefore, f (3) − f ( 2) > 0 . 2
(b) The area under the graph of f ′ is greater on the 2 15
interval [1, 2] because the graph is decreasing on
A = 0 9 − x2
dx

[1, 4]. 2  −5 2 52 
= 0 
x − 3
+  dx
x + 3
12 12 A B 5 2
41. = = + = ln x + 3 − ln x − 3  0
2
x + 5x + 6 ( x + 2)( x + 3) x + 2 x + 3 2
5 5
12 = A( x + 3) + B( x + 2) = (ln 5 − ln 1) − (ln 3 − ln 3) = ln 5 ≈ 4.024
2 2
When x = − 3, B = −12.
7 −7 A B
When x = − 2, A = 12. 44. = = +
16 − x 2 ( x − 4)( x + 4) x − 4 x + 4
4 12
A = 0 dx − 7 = A( x + 4) + B( x − 4)
x2 + 5x + 6
4  12 12  7
When x = 4, − 7 = 8 A  A = − .
= 0 
 x + 2

x + 3
 dx 8

= 12 ln x + 2 − ln x + 3  0
4 7
When x = − 4, − 7 = − 8B  B = .
8
= 12 (ln 6 − ln 7) − (ln 2 − ln 3)
3 7 3  −7 8 78 
 6(3)  9
A = 1 16 − x 2
dx = 1 
 x − 4
+
x
 dx
+ 4
= 12 ln  = 12 ln ≈ 3.016
 7( 2)  7
  7 3
= ln x + 4 − ln x − 4 1
8
7
= (ln 7 − ln 1) − (ln 5 − ln 3)
8
7 21
= ln ≈ 1.256
8 5

© 2018 Cengage Learning. All Rights Reserved. May not be scanned, copied or duplicated, or posted to a publicly accessible website, in whole or in part.
796 Chapter 8 Integration Techniques and Improper Integrals

1 80 124 p
45. Average cost =
80 − 75  75 (10 + p)(100 − p)
dp

1 80  −124 1240 
5  75  (10 + p )11 (100 − p)11 
=  +  dp

80
1  −124 1240 
= ln (10 + p) − ln (100 − p)
5  11 11  75
1
≈ ( 24.51) = 4.9
5
Approximately $490,000

1 1 A B
46. = = +
4 x2 − 1 (2 x + 1)(2 x − 1) 2 x + 1 2 x − 1
1 = A( 2 x − 1) + B( 2 x + 1)

1 1
When x = − , 1 = − 2 A  A = − .
2 2
1 1
When x = , 1 = 2B  B = .
2 2
4 1 4  −1 2 12 
1 4 x2 − 1
dx = 1 
 2 x + 1
+
2 x
 dx
− 1
4
1 1 1 
=  ln 2 x − 1 − ln 2 x + 1 
2 2 2 1
1
= (ln 7 − ln 9) − (ln 1 − ln 3)
4
1 7
= ln
4 3
1 4 1
Average value =
4 −1 1 4 x2 − 1
dx

1 1 7 1 7
=  ln  = ln
3 4 3 12 3

2
3 2x  3 x2
47. (a) V = π   2 dx = 4π  dx
0 x + 1
( x2 + 1)
2
  0

 
1 1
= 4π   2  dx
3

0 x + 1
− 2 (partial fractions)

 ( x 2
+ 1) 
3
 1 x 
= 4π arctan x −  arctan x + 2  ( trigonometric substitution )
 2 x + 1  0
3
 x   3
= 2π arctan x − 2  = 2π  arctan 3 − 10  ≈ 5.963
 x + 10  

© 2018 Cengage Learning. All Rights Reserved. May not be scanned, copied or duplicated, or posted to a publicly accessible website, in whole or in part.
Section 8.5 Partial Fractions 797

2x 3
dx = ln ( x 2 + 1) = ln 10
3
(b) A =  0 x2 +1 0

1 3 2x2 1 3 2  1 2
[2 x − 2 arctan x]0 =
3
(3 − arctan 3) ≈ 1.521
A  0 x2 + 1 ln 10  0 
x = dx = 2 − 2  dx =
x + 1 ln 10 ln 10
2
1  1  3  2x  2 3 x2
y =   0  2  dx =  dx
A 2   x + 1 ln 10 0 ( x 2 + 1)2

 
2 3 1 1  dx ( partial fractions)
ln 10  0  x 2 + 1 ( x 2 + 1)2 
= −
 
3
2  1 x 
= arctan x −  arctan x + 2  ( trigonometric substitution )
ln 10  2 x + 1  0
3
2 1  3
x 1  x  1  3
=  arctan x −  = arctan x − =  arctan 3 −  ≈ 0.412
ln 10  2 2( x 2
+ 1)  ln 10  x 2 + 1 0 ln 10  10 
 0
(x, y) ≈ (1.521, 0.412)
y

(1.521, 0.412)
1

x
1 2 3

−1

(2 − x) , 0, 1
2

48. y 2 = [ ]
(1 + x)
2

( 2 − x)
2
1
V = 0 π
(1 + x)
2
dx

 1 4 1 4x 1 x2  y

= π  dx −  0 (1 + x)2 dx +  0 (1 + x)2 dx 2−x


 0 (1 + x)
2 2 y=
 1+x

 3 
= π 2 − ( 4 ln 2 − 2) + − 2 ln 2 x
 2  −2 2

 11  π
= π  − 6 ln 2  = (11 − 12 ln 2) −2
2  2

© 2018 Cengage Learning. All Rights Reserved. May not be scanned, copied or duplicated, or posted to a publicly accessible website, in whole or in part.
798 Chapter 8 Integration Techniques and Improper Integrals

1 A B 1
49. = + ,A = B =
(x + 1)( n − x) x +1 n − x n +1
1  1 1 
n + 1  x + 1 n − x 
 +  dx = kt + C

1 x +1
ln = kt + C
n +1 n − x
1 1
When t = 0, x = 0, C = ln .
n +1 n
1 x +1 1 1
ln = kt + ln
n +1 n − x n +1 n
1  x +1 1
ln − ln  = kt
n + 1  n − x n
nx + n
ln = ( n + 1)kt
n − x
nx + n
= e(n + 1)kt
n − x
n e(n + 1)kt − 1
x =   Note: lim x = n
n + e(n + 1)kt t →∞

1 A B
50. (a) = + , (b) (1) If y0 < z0 , lim x = y0 .
( y0 − x )( z0 − x) y0 − x z0 − x t →∞

1 1 (2) If y0 > z0 , lim x = z0 .


A = ,B = − , (Assume y0 ≠ z0 .) t →∞
z 0 − y0 z0 − y0
(3) If y0 = z0 , then the original
1  1 1  equation is:
z 0 − y0   y0 − x
 −  dx = kt + C
z0 − x  1
1 z − x
 (y − x)
2
dx =  k dt
ln 0 = kt + C , when t = 0, x = 0 0
z 0 − y0 y0 − x −1
( y0 − x) = kt + C1
1 z
C = ln 0 1
z0 − y0 y0 x = 0 when t = 0  = C1
y0
1  z0 − x  z 
ln − ln  0  = kt 1 1 kty0 + 1
z 0 − y0  y0 − x  y0  = kt + =
y0 − x y0 y0
 y0 ( z 0 − x ) 
ln   = ( z0 − y0 )kt y0 − x =
y0
 z0 ( y0 − x )  kty0 + 1
y0 ( z 0 − x ) y0
= e( z0 − y0 )kt x = y0 −
z0 ( y0 − x) kty0 + 1
y0 z0 e( z0 − y0 )kt − 1 As t → ∞, x → y0 = z0 .
x =  
z0e( z0 − y0 )kt − y0

© 2018 Cengage Learning. All Rights Reserved. May not be scanned, copied or duplicated, or posted to a publicly accessible website, in whole or in part.
Section 8.5 Partial Fractions 799

x Ax + B Cx + D
51. 4
= 2 + 2
1+ x x + 2x + 1 x − 2x + 1
x = ( Ax + B) x 2 − ( )
2 x + 1 + (Cx + D) x 2 + ( 2x + 1 )
= ( A + C ) x3 + B + D − ( 2A + 2C ) x 2
(
+ A+C − 2B + )
2 D x + ( B + D)

0 = A + C  C = −A
0 = B + D − 2A + 2C −2 2 A = 0  A = 0 and C = 0
2 2
1= A+C − 2B + 2D −2 2 B = 1  B = − and D =
4 4
0 = B + D  D = −B
So,
1 x 1 − 2 4 2 4 
0 1 + x4
dx =  0  x 2 +
+  dx
2 x + 1 x 2 − 2 x + 1 

 
2 1 −1 1 
4  0   x +
= +  dx
( ) ( )
2 2
2 2  + (1 2) x − 2 2  + (1 2) 
    
1

=
2


1 
x +
−arctan 
( 2 2  ) x −
 + arctan  ( )
2 2 

4 1 2  1 2   1 2 
     0
1
( ) ( )
1
= −arctan 2 x + 1 + arctan 2x − 1 
2 0
1
=  −arctan
2
( ( )
2 + 1 + arctan ( ))
2 − 1 − ( −arctan 1 + arctan ( −1))

1 π π
=
2 
arctan ( )
2 − 1 − arctan ( 2 +1 + ) 4
+
4 
.

Because arctan x − arctan y = arctan ( x − y ) (1 + xy ), you have:

1 x 1
 

2 −1 − ( ) ( 2 + 1  π )
 +  = 1 arctan −2  + π  = 1  − π + π  = π
 0 1 + x4 dx = arctan
2 
 
2 
 
2 



1+ 2 −1 ( )( 2 +1 

2
)  2 2 4 2 8

52. The partial fraction decomposition is:


x 4 (1 − x)
4
4
2
= x 6 − 4 x5 + 5 x 4 − 4 x 2 + 4 −
1+ x 1 + x2
1
x (1 − x)
4 4
1  x7 2 x6 4 
0 1 + x2
dx = 
7

3
+ x5 − x3 + 4 x − 4 arctan
3
x
0
1 2 4 π 
= − + 1 − + 4 − 4 
7 3 3 4
22
= −π
7
Note: You can easily verify this calculation with a graphing utility.

© 2018 Cengage Learning. All Rights Reserved. May not be scanned, copied or duplicated, or posted to a publicly accessible website, in whole or in part.
800 Chapter 8 Integration Techniques and Improper Integrals

(
53. The answer is 3984. Use the division algorithm to write p( x) = x3 − x q( x) + r ( x), where the degree of r ( x) is less than 3, )
and the degree of q( x) is less than 1989. Hence,

d 1992  p( x)  d 1992  ( x 3 − x ) q( x) + r ( x) 
 3  = 1992  
dx1992  x − 3 dx  x3 − x 

d 1992  r ( x) 
=  .
dx1992  x3 − x 
Using partial fractions,
r ( x) r ( x) A B C
= = + +
x3 − x (x − 1) x( x + 1) x −1 x x +1

Since p( x) has no common factors with x3 − x, then r ( x) does not either. Hence, A, B, and C are all non zero.

d 1992  r ( x)  d 1992  A B C 
 3  = 1992 
+ +
dx1992  x − x  dx  x − 1 x x + 1
 A B C 
= 1992!  + 1993 + 
 ( x − 1) ( x + 1) 
1993 1993
x

 1993 1993 
Ax ( x + 1) + B( x − 1) ( x + 1) + Cx1993 ( x − 1) 
1993 1993 1993

= 1992! 
 
( x3 − x)
1993
 
Now expand the numerator to obtain an expression of the form
(A + B + C ) x3986 + 1993( A − C ) x3985 + 1993(996 A − B + 996C ) x3984 + 
From A = C = 1 and B = − 2, you see that the degree could be 3984. A lower degree would imply that
A + B + C = 0, A − C = 0, and 996 A − B + 996C = 0, which means A = B = C , a contradiction.

Section 8.6 Numerical Integration


1. No. The integral can easily be evaluated using basic integration rules:
2
 5 
0 (e + 5 x ) dx = e x + x 2  = e 2 + 10 − 1 = e 2 + 9.
2
x

 2 0

2. You can decrease the error by making Δ x smaller.

2 2
3. Exact: 0 x 2 dx =  13 x 3  =
0
8
3
≈ 2.6667

1 0
( 12 ) ( 32 ) + ( 2)  =
2 2 2
+ 2(1) + 2
2 2
Trapezoidal: 0 x 2 dx ≈ 4
+ 2

11
4
= 2.7500

1 0
( 12 ) ( 32 ) + ( 2)  =
2 2 2
+ 2(1) + 4
2 2
Simpson’s: 0 x 2 dx ≈ 6
+ 4

8
3
≈ 2.6667

2
2  x2   x3  19
4. Exact: 1 
 4
+ 1 dx =  + x =
 12 1 12
≈ 1.5833

2  x2  1  12   (5 4)2   (3 2)2   (7 4)2   22  203


Trapezoidal: 1 
 4
+ 1 dx ≈  + 1 + 2
 8  4   4
+ 1 + 2
  4
+ 1 + 2
  4
+ 1 + 
 4
+ 1 =
 128
≈ 1.5859
       

1  x2  1  12   (5 4) 2   (3 2) 2   (7 4)2   22  19
Simpson’s: 0 
 4
+ 1

dx ≈
12



4
+ 1 + 4
 4
+ 1

+ 2
 4
+ 1

+ 4
 4
+ 1 + 
 4
+ 1 =
 12
≈ 1.5833
         

© 2018 Cengage Learning. All Rights Reserved. May not be scanned, copied or duplicated, or posted to a publicly accessible website, in whole or in part.
Section 8.6 Numerical Integration 801

4 1 4
5. Exact: 3 x − 2
dx = ln x − 2  3 = ln 2 − ln 1 = ln 2 − 0 = ln 2 ≈ 0.6931

4 1 1  4  2  4  1
Trapezoidal Rule: 3 x − 2
dx ≈ 1 + 2  + 2  + 2  +  ≈ 0.6970
8 5  3  7  2
4 1 1  4 2  4  1
Simpson’s Rule: 3 x − 2
dx ≈
12 
1 + 4  + 2  + 4  +  ≈ 0.6933
5 3  7  2

3
3 2  2 2 2 1
6. Exact: 2 x 2
dx = −  = − +
 x 2 3 2
=
3

3 2 1 2  2   2   2  2
Trapezoidal: 2 x2
dx ≈ 
8  22
+ 2
 ( 9 4) 
2
 + 2
 (10 4) 
2
 + 2  +
 (11 4)  32 
2
 ≈ 0.3352
       

3 2 12  2   2   2  2
Simpson’s: 2 x 2
dx ≈  2 + 4
12  2  (9 4) 
2
 + 2
 (10 4) 
2
 + 4  + 2  ≈ 0.3334
 (11 4)  3 
2
       

3
3  x4  81 1
7. Exact: 1 x 3 dx =   =
 4 1 4

4
= 20

1 
3 3 3 3
3  4 5 7 8
1 + 2  + 2  + 2( 2) + 2  + 2  + 27 ≈ 20.2222
3
Trapezoidal: 1 x3 dx ≈
6  3
  3
  3
  3
  

1 
3 3 3 3
3  4 5 7 8
1 + 4  + 2  + 4( 2) + 2  + 4  + 27 = 20.0000
3
Simpson’s: 1 x3 dx ≈
9  3
  3
  3
  3
  

8 8
0 x dx =  34 x 4 3  = 12.0000
3
8. Exact:
0

8
0 1 0 + 2 + 2 3 2 + 2 3 3 + 2 3 4 + 2 3 5 + 2 3 6 + 2 3 7 + 2 ≈ 11.7296
3
Trapezoidal: x dx ≈ 2

8
0 1 0 + 4 + 2 3 2 + 4 3 3 + 2 3 4 + 4 3 5 + 2 3 6 + 4 3 7 + 2 ≈ 11.8632
3
Simpson’s: x dx ≈ 3

9 9
9. Exact: 4 x dx =  23 x3 2  = 18 −
4
16
3
= 38
3
≈ 12.6667

9 5 37 21 47 26 57 31 67 
Trapezoidal: 4 x dx ≈ 2 + 2
16  8
+ 2
4
+ 2
8
+ 2
4
+ 2
8
+ 2
4
+ 2
8
+ 3 ≈ 12.6640

9 5 37 47 57 67 
Simpson’s: 4 x dx ≈ 2 + 4
24  8
+ 21 + 4
8
+ 26 + 4
8
+ 31 + 4
8
+ 3 ≈ 12.6667

4
 x3  16 11
 1 (4 − x ) dx
4
2
10. Exact: = 4 x −  = − − = −9
 3 1 3 3

1   3   5   7  


2 2 2

 1 (4 − x ) dx
4
3 + 2 4 −    + 2(0) + 2 4 −    + 2( −5) + 2 4 −    − 12 ≈ −9.1250
2
Trapezoidal: ≈
4   2     2     2  
 
1  9  25   49  
 1 (4 − x ) dx
4
2
Simpson’s: ≈ 3 + 4 4 −  + 0 + 4 4 −  − 10 + 4 4 −  − 12 = −9
6   4  4  4 

© 2018 Cengage Learning. All Rights Reserved. May not be scanned, copied or duplicated, or posted to a publicly accessible website, in whole or in part.
802 Chapter 8 Integration Techniques and Improper Integrals

1
1 2  −2  −2 2 1
11. Exact:  0 ( x + 2)2 dx =   =
 ( x + 2)  0 3
+
2
=
3

       2
1 2 1 1 2 2 2 + 
Trapezoidal:  0 ( x + 2)2 dx ≈ + 2  + 2  + 2
82  ((1 4) + 2) 
2
 ((1 2) + 2) 
2
 ((3 4) + 2)  9 
2
       
1 1  32  8  32  2 
= + 2  + 2  + 2  +  ≈ 0.3352
8  2  81   25   121  9 
       2
1 2 1 1 2 2 2 + 
Simpson’s:  0 ( x + 2) 2 dx ≈ + 4  + 2  + 4
12  2  ((1 4) + 2)2   ((1 2) + 2) 2   ( ( 3 4) + 2) 2  9 
       
1 1  32  8  32  2 
= + 4  + 2  + 4  +  ≈ 0.3334
12  2  81   25   121  9 

32 2
1  =
( x2 + 1) (53 2 − 1) ≈ 3.393
2
12. Exact: 0 x x 2 + 1 dx =

3  0
1
3

1 0 
( 12 ) ( 12 ) ( 23 ) ( 23 )
2 2 2
Trapezoidal: 0 x x 2 + 1 dx ≈ 4
+ 2 + 1 + 2(1) 12 + 1 + 2 + 1 + 2 22 + 1 ≈ 3.4567
 

1 
( 12 ) ( 12 ) ( 32 ) ( 32 )
2 2 2
Simpson’s: 0 x x 2 + 1 dx ≈ 6
0 + 4 + 1 + 2(1) 12 + 1 + 4 + 1 + 2 22 + 1 ≈ 3.3922
 

2 2
13. Exact: 0 xe − x dx = − e − x ( x + 1) = − 3e − 2 + 1 ≈ 0.5940
0

2 1 2.2824
Trapezoidal: 0 xe − x dx ≈
4
0 + e −1 2 + 2e −1 + 3e − 3 2 + 2e − 2  ≈
4
≈ 0.5706

2 1 3.5583
Simpson’s: 0 2 xe − x dx ≈
6
0 + 2e −1 2 + 2e −1 + 6e − 3 2 + 2e − 2  ≈
6
≈ 0.5930

1 2 2 3
( 2 x − 2) ln ( x + 1) − x 2 + 2 x = ln 3 ≈ 1.6479
2
14. Exact: 0 x ln ( x + 1) dx = 
4 0 2
2 1
Trapezoidal: 0 x ln ( x + 1) dx ≈ 0 + 2(0.5) ln (1.5) + 2 ln ( 2) + 2(1.5) ln ( 2.5) + 2 ln (3) ≈ 1.6845
4
2 1
Simpson’s: 0 x ln ( x + 1) dx ≈ 0 + 4(0.5) ln (1.5) + 2 ln ( 2) + 4(1.5) ln ( 2.5) + 2 ln (3) ≈ 1.6487
6

1 1
( 18 ) + 2 ( 278 ) + 3
2
15. Trapezoidal: 0 1 + x 3 dx ≈ 4
+ 2 1+ 2 + 2 1+ ≈ 3.2833

1
( 18 ) + 2 ( 278 ) + 3
2
Simpson’s: 0 1 + x 3 dx ≈ 6

1 + 4 1+ 2 + 4 1+ ≈ 3.2396

Graphing utility: 3.2413

1 1
16. 0 x 1 − x dx = 0 x(1 − x) dx

1
(1 − 14 ) + 2 12 (1 − 12 ) + 2 43 (1 − 43 ) 
1
Trapezoidal: 0 x(1 − x) dx ≈ 0 + 2 1
≈ 0.3415

8 4

1 0
(1 − 14 ) + 2 12 (1 − 12 ) + 4 43 (1 − 43 ) 
1
Simpson’s: 0 x(1 − x) dx ≈ + 4 1 ≈ 0.3720

12  4

Graphing utility: 0.3927

© 2018 Cengage Learning. All Rights Reserved. May not be scanned, copied or duplicated, or posted to a publicly accessible website, in whole or in part.
Section 8.6 Numerical Integration 803

 
 
1 1 1 1 2 2 2 1 
17. Trapezoidal: 0 1 + x2
dx ≈
8 1 + 0
+
1
2
+
1
2
+
 3
2
+
1 + 12 
≈ 0.7828
 1+   1+  1+  
  4  2  4 

 
 
1 1 1 1 4 2 4 1 
Simpson’s: 0 1 + x2
dx ≈
12 1 + 0
+
1
2
+
1
2
+
 3
2
+
1 + 12 
≈ 0.7854
 1+  1+  1+  
  4  2  4 
Graphing utility: 0.7854

     
2 1 1
1 + 2
1  + 2 1 
+ 2
1  + 1  ≈ 1.3973
18. Trapezoidal: 0 dx ≈
4  3 
   3  
1 + x3
  1 + (1 2) 
  1 + 13 
 1 + (3 2)  3 

     
2 1 1
1 + 4
1  + 2 1 
+ 4
1  + 1  ≈ 1.4052
Simpson’s: 0 dx ≈
6  3 

    3
3 
 1 + (1 2)   1 + (3 2) 
1 + x3  1 + 13 
 
Graphing utility: 1.4022

4 1
19. Trapezoidal: 0 xe x dx =
2
0 + 2e + 2 2e 2 + 2 3e3 + 2e 4  ≈ 102.5553

4 1
Simpson’s: 0 xe x dx =
3
0 + 4e + 2 2e 2 + 4 3e3 + 2e 4  ≈ 93.3752

Graphing utility: 92.7437

3 1 3 5 
20. Trapezoidal: 1 ln x dx =
4 
0 + 2 ln + 2 ln 2 + 2 ln + ln 3 ≈ 1.2821
2 2 
3 1 3 5 
Simpson’s: 1 ln x dx =
6 
0 + 4 ln + 2 ln 2 + 4 ln + ln 3 ≈ 1.2953
2 2 
Graphing utility: 1.2958

21. Trapezoidal:
π 2  π  
2 2 2 2
π 2  π 2  π 2 3 π 2  
0 sin ( x 2
) dx ≈ sin 0 + 2 sin   + 2 sin   + 2 sin   + sin   ≈ 0.5495
8   4 2 4 2 
        

π 2  π  
2 2 2 2
π 2  π 2  π 2 3 π 2  
Simpson’s: 0 sin ( x 2
) dx ≈ sin 0 + 4 sin   + 2 sin   + 4 sin   + sin   ≈ 0.5483
12   4 2 2 
      4   
Graphing utility: 0.5493

π π π 5π  5π  3π  3π  7π  7π  
22. Trapezoidal: π 2 x sin x dx ≈  (1) + 2 sin   + 2 sin   + 2 sin   + 0 ≈ 1.4299
16  2 8  8  4  4  8  8  

π π  π 5π  5π  3π  3π  7π  7π  
Simpson’s: π 2 x sin x dx ≈ 
24  2
+ 4
8
sin   + 2
 8  4
sin   + 4
 4  8
sin   + 0 ≈ 1.4583
 8 
Graphing utility: 1.4579

© 2018 Cengage Learning. All Rights Reserved. May not be scanned, copied or duplicated, or posted to a publicly accessible website, in whole or in part.
804 Chapter 8 Integration Techniques and Improper Integrals

π 4 π  π  π   2π   2π   3π   3π  π 
23. Trapezoidal: 0 x tan x dx ≈
32 
0 + 2  tan   + 2  tan   + 2  tan   +  ≈ 0.1940
 16   16   16   16   16   16  4

π 4 π  π  π   2π   2π   3π   3π  π 
Simpson’s: 0 x tan x dx ≈
48 
0 + 4  tan   + 2  tan   + 4  tan  +  ≈ 0.1860
 16   16   16   16   16   16  4
Graphing utility: 0.1858

π sin x π 2 sin (π 4) 2 sin (π 2) 2 sin (3π 4) 


24. Trapezoidal: 0 x
dx ≈ 1 +
8 π 4
+
π 2
+
3π 4
+ 0 ≈ 1.8355

π sin x π 4 sin (π 4) 2 sin (π 2) 4 sin (3π 4) 
Simpson’s: 0 x
dx ≈ 1 +
12  π 4
+
π 2
+
3π 4
+ 0 ≈ 1.8522

Graphing utility: 1.8519

25. f ( x) = x 2 + 2 x
f ′( x) = 2 x + 2
f ′′( x) = 2
f ′′′( x) = 0
f ( 4) ( x ) = 0

(a) Trapezoidal Rule: Because f ′′( x) is maximum for all x in [0, 2] and f ′′( x) = 2, you have

(2 − 0) 2
3
1
2 ( )
Error ≤ = ≈ 0.0833.
12( 4) 12

( 2 − 0) 0
5

(b) Simpson’s Rule: Because f (4) ( x) is maximum for all x in [0, 2] and f (4) ( x) = 0, you have Error ≤ 4 ( )
= 0.
180( 4)

26. f ( x) = 2 x3 27. f ( x) = ( x − 1)
−2

f ′( x) = 6 x 2 f ′( x) = −2( x − 1)
−3

f ′′( x) = 12 x −4
f ′′( x) = 6( x − 1)
f ′′′( x) = 12 −5
f ′′′( x) = −24( x − 1)
f (4) ( x) = 0
f (4) ( x) = 120( x − 1)
−6

(3 − 1)
3

(a) Trapezoidal: Error ≤ (36) = 1.5 because


( 4 − 2)
3
12( 42 ) (a) Trapezoidal: Error ≤ (6) =
1
because
12( 4 2
) 4
f ′′( x) is maximum in [1, 3] when x = 3.
f ′′( x) is a maximum of 6 at x = 2.
(3 − 1)
5

(b) Simpson’s: Error ≤ (0) = 0 because


(4 − 2)
5
( )
180 44 (b) Simpson’s: Error ≤ (120) =
1
because
180( 44 ) 12
f (4) ( x) = 0.
f (4) ( x) is a maximum of 120 at x = 2.

© 2018 Cengage Learning. All Rights Reserved. May not be scanned, copied or duplicated, or posted to a publicly accessible website, in whole or in part.
Section 8.6 Numerical Integration 805

3
(b) Maximum of f (4) ( x ) = 24(1 + x )
−5
28. f ( x) = e x is 24.
2 x3
f ′( x) = 3 x e Simpson’s:
f ′′( x) = 3(3 x + 2 x)e
4 x3
Error ≤
1
( 24) ≤ 0.00001
180n 4
f ′′′( x) = 3(9 x 6 + 18 x3 + 2)e x
3

n 4 ≥ 13,333.33
( )
3
f (4) ( x) = 9 9 x8 + 36 x5 + 20 x 2 e x
n ≥ 10.75
(a) Trapezoidal Rule: Because f ′′( x) is maximum in Let n = 12. (In Simpson’s Rule n must be even.)

[0, 1] when x = 1 and f ′′(1) = 15e, you have


f ( x ) = ( x + 2) , 0 ≤ x ≤ 2
12
31.
(1 − 0) 15e
3
5e 1
2 ( ) −1 2
Error ≤ = ≈ 0.212. f ′( x) = ( x + 2)
12( 4) 64 2
1 −3 2
(b) Simpson’s Rule: Because f (4) ( x) is maximum in f ′′( x) = − ( x + 2)
4
[0, 1] when x = 1 and f (4) (1) = 585e, you have 3
f ′′′( x) = ( x + 2)
−5 2

8
(1 − 0) 585e
5
13e −15
4 ( ) f ( 4) ( x ) =
−7 2
Error ≤ = ≈ 0.035. ( x + 2)
180( 4) 1024 16

−1
29. f ( x) = x −1 , 1≤ x ≤ 3 (a) Maximum of f ′′( x) = is
4( x + 2)
32

f ′( x) = − x −2

f ′′( x) = 2 x −3 2
≈ 0.0884.
16
f ′′′( x) = −6 x −4
Trapezoidal:
f (4) ( x) = 24 x −5
Error ≤
(2 − 0)  2 
3

(a) Maximum of f ′′( x) = 2 x −3 is 2.   ≤ 0.00001


12 n 2  16 
Trapezoidal: 8 2 5 2 5
23 n2 ≥ 10 = 10
Error ≤ (2) ≤ 0.00001, n 2 ≥ 133,333.33, 12(16) 24
12n 2
n ≥ 76.8. Let n = 77.
n ≥ 365.15 Let n = 366.
(b) Maximum of f (4) ( x) = 24 x −5 is 24. −15
(b) Maximum of f (4) ( x) = is
16( x + 2)
72
5
2
Simpson’s: Error ≤ (24) ≤ 0.00001, 15 2
180n 4 ≈ 0.0829.
256
n 4 ≥ 426,666.67, n ≥ 25.56 Let n = 26.
Simpson’s:
−1
30. f ( x) = (1 + x) , 0 ≤ x ≤1 25  15 2 
Error ≤   ≤ 0.00001
f ′( x) = −(1 + x)
−2
180n 4  256 
−3
f ′′( x) = 2(1 + x) 32(15) 2 5
n4 ≥ 10
f ′′′( x) = −6(1 + x)
−4
180( 256)

f (4) ( x) = 24(1 + x)
−5
2 5
= 10
−3
96
(a) Maximum of f ′′( x ) = 2(1 + x ) is 2.
n ≥ 6.2. Let n = 8 (even ).
Trapezoidal:
1
Error ≤ (2) ≤ 0.00001
12n 2
n 2 ≥ 16,666.67
n ≥ 129.10. Let n = 130.

© 2018 Cengage Learning. All Rights Reserved. May not be scanned, copied or duplicated, or posted to a publicly accessible website, in whole or in part.
806 Chapter 8 Integration Techniques and Improper Integrals

32. f ( x) = e 2 x , 1 ≤ x ≤ 3
f ′( x) = 2e 2 x
f ′′( x) = 4e 2 x
f ′′′( x) = 8e 2 x
f (4) ( x) = 16e 2 x

(a) Maximum of f ′′( x) = 4e2 x is 4e 2(3) ≈ 1613.7152

(3 − 1)3
Trapezoidal: Error ≤ (1613.7152) ≤ 0.00001
12n 2
8
n2 ≥ (1613.7152) 105
12
n ≥ 10,372.1.
Let n = 10,373.

(b) Maximum of f (4) ( x) = 16e 2 x is 16e 2(3) ≈ 6454.8607

(3 − 1)
5

Simpson’s: Error ≤ (6454.8607) ≤ 0.00001


180n 4
32
n4 ≥ (6454.8607) 105
180
n ≥ 103.5.
Let n = 104(even ).

33. f ( x) = tan x 2( )
(a) f ′′( x) = 2 sec2 ( x 2 ) 1 + 4 x 2 tan ( x 2 ) in [0, 1].

f ′′( x) is maximum when x = 1 and f ′′(1) ≈ 49.5305.

(1 − 0)
3
Trapezoidal: Error ≤ 2 (49.5305) ≤ 0.00001, n 2 ≥ 412,754.17, n ≥ 642.46; let n = 643.
12n

(b) f (4) ( x) = 8 sec 2 ( x 2 ) 12 x 2 + (3 + 32 x 4 ) tan ( x 2 ) + 36 x 2 tan 2 ( x 2 ) + 48 x 4 tan 3 ( x 2 ) in [0, 1]

f (4) ( x) is maximum when x = 1 and f (4) (1) ≈ 9184.4734.

(1 − 0)
5
Simpson’s: Error ≤ (9184.4734) ≤ 0.00001, n 4 ≥ 5,102,485.22, n ≥ 47.53; let n = 48.
180n 4

34. f ( x) = ( x + 1)
23 56
(b) f (4) ( x) = − in [0, 2].
81( x + 1)
10 3

2
(a) f ′′( x) = − in [0, 2].
9( x + 1) f (4) ( x) is maximum when x = 0 and
43

2 56
f ′′( x) is maximum when x = 0 and f ′′(0) = . f ( 4) ( 0 ) = .
9 81
8  2 32  56 
Trapezoidal: Error ≤   ≤ 0.00001, Simpson’s: Error ≤   ≤ 0.00001,
12n 4  9  180n 4  81 

n 2 ≥ 14,814.81, n ≥ 121.72; let n = 122. n 4 ≥ 12,290.81, n ≥ 10.53; let n = 12. (In


Simpson’s Rule n must be even.)

© 2018 Cengage Learning. All Rights Reserved. May not be scanned, copied or duplicated, or posted to a publicly accessible website, in whole or in part.
Section 8.6 Numerical Integration 807

35. n = 4, b − a = 4 − 0 = 4
4
Trapezoidal:  0 f ( x) dx ≈ 4 3
8
+ 2(7) + 2(9) + 2(7) + 0 = 1
2 ( 49) = 49
2
= 24.5

4
Simpson’s:  0 f ( x) dx ≈ 4 3
12 
+ 4(7) + 2(9) + 4(7) + 0 = 77
3
≈ 25.67

36. n = 8, b − a = 8 − 0 = 8
8
Trapezoidal:  0 f ( x) dx ≈ 8 0
16 
+ 2(1.5) + 2(3) + 2(5.5) + 2(9) + 2(10) + 2(9) + 2(6) + 0 = 1
2 (88) = 44

8
Simpson’s:  0 f ( x) dx ≈ 8 0
24 
+ 4(1.5) + 2(3) + 4(5.5) + 2(9) + 4(10) + 2(9) + 4(6) + 0 = 1
3 (134) = 134
3

π 2
37. A = 0 x cos x dx

Simpson’s Rule: n = 14
π 2 π  π π π π 3π 3π π π
0 x cos x dx ≈  0 cos 0 + 4
84  28
cos
28
+ 2
14
cos
14
+ 4
28
cos
28
++
2
cos  ≈ 0.701
2
y

1
2

x
π π
4 2

2 40. For a linear function, the Trapezoidal Rule is exact. The


38. (a) The integral  0 f ( x)dx would be overestimated
(b
− a)
3

because the trapezoids would be above the curve. error formula says that E ≤ max f ′′( x) 
2
12n 2  
Similarly, the integral  0 g ( x)dx would be and f ′′( x) = 0 for a linear function. Geometrically, a
underestimated. linear function is approximated exactly by trapezoids:
y

(b) Simpson’s Rule would be more accurate because it


takes into account the curvature of the graph.

39. The Trapezoidal Rule is the average of the left-hand


Riemann Sum and the right-hand Riemann Sum,
1
Tn = ( Ln + Rn ).
2 a b
x

1000
41. Area ≈ 125 + 2(125) + 2(120) + 2(112) + 2(90) + 2(90) + 2(95) + 2(88) + 2(75) + 2(35) = 89,250 m 2
2(10) 

42. Simpson’s Rule: n = 8


π 2 2 3π  2 2 2 π 2 π 2 π 
8 3 1− sin 2 θ dθ ≈ 2
 1 − sin 0 + 4 1 − sin + 2 1 − sin 2 ++ 1− sin 2 
0 3 6  3 3 16 3 8 3 2
≈ 17.476

© 2018 Cengage Learning. All Rights Reserved. May not be scanned, copied or duplicated, or posted to a publicly accessible website, in whole or in part.
808 Chapter 8 Integration Techniques and Improper Integrals

5
43. W =  0 100 x 125 − x3 dx

Simpson’s Rule: n = 12

5 
3 3
5 5 5  10   10 
 0 100 x 125 − x3 dx ≈
3(12) 
0 + 400  125 −   + 200  125 −  
 12   12   12   12 

3 
 15   15 
+ 400  125 −   +  + 0 ≈ 10,233.58 J
 12   12  

44. (a) Trapezoidal:
2 2
 0 f ( x) dx ≈ 4.32 + 2(4.36) + 2( 4.58) + 2(5.79) + 2(6.14) + 2(7.25) + 2(7.64) + 2(8.08) + 8.14 ≈ 12.518
2(8) 
Simpson’s:
2 2
 0 f ( x) dx ≈ 4.32 + 4( 4.36) + 2( 4.58) + 4(5.79) + 2(6.14) + 4(7.25) + 2(7.64) + 4(8.08) + 8.14 ≈ 12.592
3(8) 
(b) Using a graphing utility,
2
y = −1.37266 x3 + 4.0092 x 2 − 0.620 x + 4.28. Integrating, 0 y dx ≈ 12.521.

t
45.  0 sin x dx = 2, n = 10

By trial and error, you obtain t ≈ 2.477.

46. Let f ( x) = Ax3 + Bx 2 + Cx + D. Then f (4) ( x) = 0.

(b − a )
5
Simpson’s: Error ≤ ( 0) = 0
180n 4
So, Simpson’s Rule is exact when approximating the integral of a cubic polynomial.

1 
3
1 1 1
0 x
3
Example: dx = 0 + 4  + 1 =
6  2
   4

This is the exact value of the integral.

47. The quadratic polynomial

p( x ) =
(x − x2 )( x − x3 )
y1 +
( x − x1 )( x − x3 ) y + ( x − x1 )( x − x2 ) y
( x1 − x2 )( x1 − x3 ) ( x2 − x1 )( x2 − x3 ) 2 ( x3 − x1 )( x3 − x2 ) 3
passes through the three points.

Section 8.7 Integration by Tables and Other Integration Techniques


1. Use Formula 40. 3. By Formula 6: ( a = 5, b = 1)

2. A reduction formula reduces an integral to the sum of a x2  x 


5 + dx = − (10 − x) + 25 ln 5 + x  + C
function and a simpler integral. For example, see x  2 
Formulas 50, 54.

4. By Formula 13: ( a = 4, b = 3)

2  −1   4 + 6 x 6 x 
 x 2 ( 4 + 3 x)2 dx = 2   + ln
 16   x( 4 + 3x ) 4
+C
4 + 3 x 

= −
( 2 + 3 x) −
3
ln
x
+C
4 x ( 3 x + 4) 16 4 + 3x

© 2018 Cengage Learning. All Rights Reserved. May not be scanned, copied or duplicated, or posted to a publicly accessible website, in whole or in part.
Section 8.7 Integration by Tables and Other Integration Techniques 809

1 1 − x2
5. By Formula 44:  x2 1 − x2
dx = −
x
+ C

6. Let u = x 2 , du = 2 x dx.

64 − x 4 1 64 − x 4
 du =  (2 x dx)
x 2 x2
1 64 − u 2
=
2  u
du

By Formula 39: ( a = 8)

64 − u 2 1 8+ 64 − u 2 
 u
dx = 
2
64 − u 2 − 8 ln
u
 +C


1 8+ 64 − x 4
= 64 − x 4 − 4 ln +C
2 x2

7. By Formulas 51 and 49:


1
 cos cos 4 3 x (3) dx
3
4
3x dx =

1  cos3 3 x sin 3 x 3 
=  +  cos 2 3 x dx
3 4 4 
1 1 1
cos 2 3 x (3) dx
4 3
= cos3 3x sin 3x + ⋅
12
1 1 1
= cos3 3x sin 3x + ⋅ (3x + sin 3 x cos 3 x) + C
12 12 2
1
=
24
(2 cos3 3x sin 3x + 3 x + sin 3 x cos 3 x) + C

1
8. Let u = x, du = dx.
2 x
sin 4 x
 x
dx = 2 sin 4 u du

 sin 3 u cos u 3 
= 2 − +  sin 2 u du  ( Formula 50, n = 4)
 4 4 
 sin 3 u cos u 3 1 
= 2 − + ⋅ (u − sin u cos u ) + C ( Formula 48)
 4 4 2 
1 3 3
= − sin 3 u cos u + u − sin u cos u + C
2 4 4
1 3 3
= − sin 3 x cos x + x − sin x cos x +C
2 4 4

1 1  1 
9. By Formula 57:  dx = 2   (
 dx = −2 cot x + csc )
x +C
(
x 1 − cos x ) 1 − cos x2 x 

1
u = x , du = dx
2 x

© 2018 Cengage Learning. All Rights Reserved. May not be scanned, copied or duplicated, or posted to a publicly accessible website, in whole or in part.
810 Chapter 8 Integration Techniques and Improper Integrals

10. Let u = 4 x, du = 4 dx.


By Formula 72:
1 1 1
 1 + cot 4 x dx (4 dx)
4  1 + cot 4 x
=

1 1
= ⋅ ( 4 x − ln sin 4 x + cos 4 x ) + C
4 2
1 1
= x − ln sin 4 x + cos 4 x + C
2 8

11. By Formula 84:


1 1
 1 + e2 x dx = 2 x − ln (1 + e 2 x ) + C
2

12. By Formula 85: ( a = − 4, b = 3)

e− 4 x
e
−4x
sin3x dx = ( − 4 sin 3x − 3 cos 3x) + C
(− 4)2 + 32
e− 4 x
= ( − 4 sin 3x − 3 cos 3x) + C
25

13. By Formula 89: ( n = 6)

x7
x
6
ln x dx = (−1 + 7 ln x) + C
49

 (ln x) dx = x(ln x) − 3 (ln x) dx


3 3 2
14. By Formulas 90 and 91:

= x(ln x) − 3 x 2 − 2 ln x + (ln x)  + C
3 2
 
= x (ln x) − 3(ln x ) + 6 ln x − 6 + C
3 2
 

1 1
15. (a) Let u = x  du = dx (b) Integration by parts:
3 3
x
dv = dx  v =  dx = x
 ln 3 dx = 3  ln u du
x 1 1 1
u = ln  du = ⋅ = dx
By Formula 87: 3 x3 3 x

x 1  x  x x 1
 ln 3 dx = 3 x −1 + ln  + C  ln 3 dx = x ln
3 
− x ⋅ dx
x
3  3 
x
 x  = x ln −  dx
= x ln − 1 + C 3
 3  x
= x ln − x + C
3
 x 
= x ln − 1 + C
 3 

© 2018 Cengage Learning. All Rights Reserved. May not be scanned, copied or duplicated, or posted to a publicly accessible website, in whole or in part.
Section 8.7 Integration by Tables and Other Integration Techniques 811

16. Let u = 3 x, du = 3 dx.


(a) By Formula 48: (b) Power-reducing formula:
1 1 1
 sin 2 3 x dx = ⋅ (3 x − sin 3 x cos 3 x) + C  sin sin 2 3x (3 dx)
3
2
3 x dx =
2 3
1 1  sin 3 x cos 3x 1 
= (3 x − sin 3 x cos 3 x) + C
6
= −
3 2
+
2  3 dx
1 1
= − sin 3x cos 3 x + x + C
6 2

17. (a) By Formula 12: (a = −1, b = 1) (b) Partial fractions:


1 A B C
= + 2 +
1 −1  1 1 x  x 2 ( x − 1) x x x −1
 x 2 ( x − 1)
dx =  + ln  +C
(−1)  x (−1) −1 + x  1 = Ax( x − 1) + B( x − 1) + Cx 2
1 x When x = 1, C = 1.
= − ln +C
x x −1
When x = 0, B = −1.
1 x −1
= + ln +C When x = −1, 1 = 2 A + 2 + 1  A = −1.
x x
1  −1 −1 1 
 x 2 ( x − 1) dx =   x +
x 2
+  dx
x − 1
1
= − ln x + + ln x − 1 + C
x
1 x −1
= + ln +C
x x

18. (a) By Formula 36: (n = 2)


dx x
 = + C
(4 + x )
32
2 4 4 + x2

(b) Trigonometric substitution:


x = 2 tan θ , dx = 2 sec 2 θ dθ

4 + x 2 = 4 sec 2 θ

dx 2 sec2 θ dθ
 = 
(4 + x 2 ) 8 sec3 θ
32

1
4
= cos θ dθ
4 + x2
x
1 x
= sin θ + C = +C θ
4 4 4 + x2 2

19. By Formula 80:


1
 x arccsc( x + 1) dx =
2
arccsc( x 2 + 1)( 2 x) dx
2

1 2 
( x + 1)arccsc( x + 1) + ln x + 1 + ( x2 + 1) − 1  + C
2
2 2
=
2 

=
1 2
2
(
( x + 1)arccsc( x 2 + 1) + 12 ln x 2 + 1 + x4 + 2x2 + C )

© 2018 Cengage Learning. All Rights Reserved. May not be scanned, copied or duplicated, or posted to a publicly accessible website, in whole or in part.
812 Chapter 8 Integration Techniques and Improper Integrals

20. By Formula 78: u = 4 x − 5, du = 4 dx


1
 arccot (4 x − 5) dx =
4
arccot ( 4 x − 5) ( 4 dx)

1
= ( 4 x − 5) arccot ( 4 x − 5) + ln 1 + ( 4 x − 5)  + C
2

4 

21. Let u = x 2 , du = 2 x dx.


2 2x du
 x3 x4 − 1
dx =  x4 x4 − 1
dx =  u2 u2 − 1

du u2 − 1 x4 − 1
By Formula 35:  u2 u2 − 1
=
u
=
x2
+ C

22. By Formula 14: a = 8, b = 4, c = 1, b 2 < 4ac 25. By Formula 76: u = e x , du = e x dx


1 2 2x + 4
e
x
arccos e x dx = e x arccos e x − 1 − e2 x + C
 x2 + 4x + 8
dx =
16
arctan
16
+C

1  x + 2 26. By Formula 71: u = e x , du = e x dx


= arctan  +C
2  2 
ex 1 x
23. By Formula 4: a = 7, b = − 6
 1 − tan e x dx =
2
(e − ln cos e x − sin e x )+C
x 1  7  27. By Formula 73:
 (7 − 6 x)2 dx = 2
( − 6)  7 − 6 x
+ ln 7 − 6 x  + C
 x 1 2x
1 7   1 − sec x 2 dx =
2  1 − sec x 2
dx
=  + ln 7 − 6 x  + C
36  7 − 6 x  1 2
=
2
( x + cot x 2 + csc x 2 ) + C
24. By Formula 56: u = θ 4 , du = 4θ 3 dθ

θ3 1 1
 1 + sin θ 4 dθ =
4  1 + sin θ 4
4θ 3 dθ

1
=
4
( tan θ 4 − sec θ 4 ) + C
1
28. By Formula 23: u = ln t , du = dt
t
1 1 1
 t 1 + (ln t )2  dt =  1 + (ln t )2  t  dt = arctan(ln t ) + C
 

29. By Formula 14: u = sin θ , du = cos θ dθ


cos θ 2  1 + sin θ 
 3 + 2 sin θ + sin 2 θ
dθ =
2
arctan   +C (b2 = 4 < 12 = 4ac)
 2 

x
2
30. 3 + 25 x 2 dx, u = 5 x, du = 5 dx
1 1
125  (5 x )
2
3 + (5 x )
2
(5 dx) =
125 u
2
3 + u 2 du (a = 3 )
By Formula 27:
1 1 1
u
2
3 + u 2 du = ⋅ u ( 2u 2 + 3) u 2 + 3 − 9 ln u + u 2 + 3  + C
125 125 8  
1 
= 5 x(50 x 2 + 3) 25 x 2 + 3 − 9 ln 5 x + 25 x 2 + 3  + C
1000  

© 2018 Cengage Learning. All Rights Reserved. May not be scanned, copied or duplicated, or posted to a publicly accessible website, in whole or in part.
Section 8.7 Integration by Tables and Other Integration Techniques 813

1 3 3 2 + 9 x2 2 + 9x2
31. By Formula 35:  x2 dx = 3 dx = − +C = − +C
2 + 9 x2
( 2) 6x 2x
2
(3 x ) + (3 x )
2 2

 x arctan ( x 3 2 ) dx = 2
 arctan( x )( 32 ) 2 x3 2 arctan ( x 3 2 ) − ln 1 + x3  + C
32
32. By Formula 77: x dx = 3
3 

1
33. By Formula 3: u = ln x, du = dx
x
ln x 1
 x(3 + 2 ln x) dx =
4
(
2 ln x − 3 ln 3 + 2 ln x )+C

34. By Formula 45: u = e x , du = e x dx

ex ex
 dx = +C
(1 − e2 x )
32
1 − e2 x

x 1 2x − 6 + 6
35. By Formulas 1, 23, and 35:  dx =  dx
( x2 − 6 x + 10) 2 ( x 2 − 6 x + 10)2
2

1 1
( x 2 − 6 x + 10) ( 2 x − 6) dx + 3
−2
=
2  ( x − 3)2 + 1
2
dx
 
1 3 x −3 
= − +  2 + arctan ( x − 3) + C
2( x 2 − 6 x + 10) 2  x − 6 x + 10 
3 x − 10 3
= + arctan ( x − 3) + C
2( x 2 − 6 x + 10) 2

36. By Formula 41:


5− x 5− x 5− x
 5+ x
dx =  5+ x

5− x
dx

5− x
=  25 − x 2
dx

5 dx x
=  25 − x 2
−  25 − x 2
dx

 x
= 5 arcsin   + 25 − x 2 + C
5

37. By Formula 31: u = x 2 − 3, du = 2 x dx


x 1 2x 1
 dx =
2
dx =
2
ln x 2 − 3 + x4 − 6x2 + 5 + C
(x − 3) − 4
4 2 2
x − 6x + 5 2

38. By Formula 31: u = sin x, du = cos x dx

cos x
 sin 2 x + 1
dx = ln sin x + sin 2 x + 1 + C

© 2018 Cengage Learning. All Rights Reserved. May not be scanned, copied or duplicated, or posted to a publicly accessible website, in whole or in part.
814 Chapter 8 Integration Techniques and Improper Integrals

39. By Formula 8: u = e x , du = e x dx 42. Let u = x 2 , du = 2 x dx


1 2 1
2 x2
(e x ) 0 2 x 0 x e (2 x dx)
2 3
e x dx =
e3 x
 dx =  (e ) dx
x

(1 + e x ) (1 + e x )
3 3
 ue du
4
=

=
2

1
+ ln 1 + e x + C = (u − 1)eu  ( Formula 82)
1 + e x 2(1 + e x ) 2 1
= ( x 2 − 1)e x 
2

 0
40. By Formulas 64 and 68: = 0 − ( −1) = 1
cot 3 θ
 cot θ dθ = − 3 −  cot θ dθ
4 2
43. By Formula 89: n = 4
cot 3 θ 2
= − + θ + cot θ + C 2  x5 
3 1 x 4 ln x dx =  ( −1 + 5 ln x)
 25 1
41. By Formula 21: a = 1, b = 1, u = x, du = dx 32 1
= [−1 + 5 ln 2] − [−1 + 0]
1 25 25
1 x  − 2( 2 − x) 
31 32
0 1+ x
dx = 
 3(1)
2
1 + x

= − + ln 2 ≈ 3.1961
0 25 5
1
2  44. By Formula 52: u = 2 x, du = 2dx
=  ( x − 2) 1 + x
3 0
π 2 1 π2
0 (2 x) sin 2 x (2dx)
40
2 2 x sin 2 x dx =
= (−1) 2 − (− 2) 1
3 3 1 π 2
2 = [sin 2 x − 2 x cos 2 x]0
= −
3
(
2 − 2 ) 4
1
≈ 0.3905 = 0 − π ( −1)
4
π
=
4

45. By Formula 23: u = sin x, du = cos x


π 2 cos x π 2
 − π 2 1 + sin 2 x dx = arctan (sin x) −π 2

π
= arctan (1) − arctan ( −1) =
2

46. By Formula 7: a = 5, b = 2
5
5 x2 1 25 
 0 (5 + 2 x ) 2 dx =
8 
2x −
5 + 2x
− 10 ln 5 + 2 x 
0
1  25  
= 10 − − 10 ln 15  − ( − 5 − 10 ln 5)
8  15  
5 1  15 
= − (10) ln  
3 8 5
5 5
= − ln 3
3 4

© 2018 Cengage Learning. All Rights Reserved. May not be scanned, copied or duplicated, or posted to a publicly accessible website, in whole or in part.
Section 8.7 Integration by Tables and Other Integration Techniques 815

47. By Formulas 54 and 55:

t cos t dt = t 3 sin t − 3 t 2 sin t dt


3

(
= t 3 sin t − 3 −t 2 cos t + 2  t cos t dt )
(
= t 3 sin t + 3t 2 cos t − 6 t sin t −  sin t dt )
3 2
= t sin t + 3t cos t − 6t sin t − 6 cos t + C
So,
π 2 3 π 2
0 t cos t dt = t 3 sin t + 3t 2 cos t − 6t sin t − 6 cos t 
0

π 3  π3
=  − 3π  + 6 = + 6 − 3π ≈ 0.4510.
 8  8

48. By Formula 26: a = 4


3
1
x x 2 + 16 + 16 ln x + x 2 + 16 
3
0 x 2 + 16 dx =
2 
 
 0
1
= (3(5) + 16 ln 3 + 5 ) − (16 ln 4)
2
15
= + 8 ln 8 − 8 ln 4
2
15
= + 8 ln 2
2

u2 1 (2a b)u + (a 2 b 2 ) 1 A B
49. = − = 2 + +
(a + bu ) ( a + bu ) ( a + bu )
2 2 2
b2 b a + bu
2a a2
− u − 2 = A( a + bu ) + B = ( aA + B) + bAu
b b
Equating the coefficients of like terms you have aA + B = − a 2 b 2 and bA = − 2a b. Solving these equations you have
A = − 2a b 2 and B = a 2 b 2 .

u2 1 2a  1  1 a2  1  1 1 2a a2  1 
 (a + bu )2 du =
b2  du −   
b 2  b  a + bu
b du +   
b 2  b  ( a + bu )2
b du =
b2
u −
b3
ln a + bu −   +C
b3  a + bu 
1 a2 
= 3
bu − − 2a ln a + bu  + C
b  a + bu 

du 2
50. Integration by parts: w = u n , dw = nu n −1 du, dv = ,v = a + bu
a + bu b
un 2u n 2n n −1
 a + bu
du =
b
a + bu −
b 
u a + bu du

2u n 2n n −1 a + bu
b 
= a + bu − u a + bu ⋅ du
b a + bu
2u n 2n au n −1 + bu n
b 
= a + bu − du
b a + bu
2u n 2na u n −1 un
=
b
a + bu −
b  a + bu
du − 2n 
a + bu
du

un 2 u n −1 
Therefore, ( 2n + 1)  du = u n a + bu − na  du  and
a + bu b a + bu 
un 2  n u n −1 
 a + bu
=
(2n + 1)b 
u a + bu − na 
a + bu
du .

© 2018 Cengage Learning. All Rights Reserved. May not be scanned, copied or duplicated, or posted to a publicly accessible website, in whole or in part.
816 Chapter 8 Integration Techniques and Improper Integrals

51. When you have u 2 + a 2 :


u = a tan θ u2 + a2
u
du = a sec 2 θ dθ
θ
u 2 + a 2 = a 2 sec 2 θ a

1 a sec θ dθ
2
1 1 u
 du =  = 2  cos θ dθ = 2 sin θ + C = +C
(u )
32
2
+ a 2 a 3 sec3 θ a a a2 u2 + a2

When you have u 2 − a 2 :


u
u = a sec θ u2 − a2

du = a sec θ tan θ dθ θ
a
u 2 − a 2 = a 2 tan 2 θ
1 a sec θ tan θ dθ 1 cos θ 1 1 −u
 du =  = 2  2 dθ = 2  csc θ cot θ dθ = − 2 csc θ + C = +C
(u ) a 3 tan 3 θ a sin θ
32
2
−a 2 a a a2 u 2 − a2

 u (cos u ) du = u n sin u − n  u n −1 (sin u ) du  


n
52. 2 du
1  2  θ
1 + u
n
w = u , dv = cos u du , dw = nu n −1
du , v = sin u 55.  2 − 3 sin θ
dθ =    2u  
, u = tan
2
 2 − 3 1 + u 2  
1 2u
53.  (arctan u ) du = u arctan u −
2  1 + u2
du 2
=  du
1 2(1 + u 2 ) − 6u
= u arctan u − ln (1 + u 2 ) + C
2 1
= u arctan u − ln 1 + u2 + C
=  u2− 3u + 1
du

1
w = arctan u , dv = du , dw =
du
,v = u =  2
du
1 + u2  3 5
 u −  −
 2 4
n −1  1 
 (ln u ) du = u (ln u ) −  n(ln u )
n n
54.  u du  3 5
u 1 u −  −
= ln  2 2
+C
n −1
= u (ln u ) − n  (ln u )
n
du 5  3 5
u −  +
 2 2
n −1  1 
w = (ln u ) , dv = du , dw = n(ln u )
n
  du , 1 2u − 3 − 5
u = ln +C
v = u 5 2u − 3 + 5
θ 
2 tan   − 3 − 5
1  2
= ln +C
5 θ 
2 tan   − 3 + 5
 2

sin θ −sin θ
56.  1 + cos2 θ dθ = − 
1 + (cos θ )
2

= −arctan (cos θ ) + C

© 2018 Cengage Learning. All Rights Reserved. May not be scanned, copied or duplicated, or posted to a publicly accessible website, in whole or in part.
Section 8.7 Integration by Tables and Other Integration Techniques 817

2 du   4 4
π 2 1 1 + u 1
2  62.  csc θ − cot θ
dθ = 1   cos θ 

57. 0 1 + sin θ + cos θ
dθ =  
2u 1 − u 2   − 
 sin θ   sin θ 
0
1 + + 
 1 + u 2 1 + u 2  sin θ
1 1 = 4 dθ
=  du 1 − cos θ
01 + u

1
= 4 ln 1 − cos θ + C
= ln 1 + u  0
u = 1 − cos θ , du = sin θ dθ
= ln 2
θ 63. By Formula 21: a = 3, b = 1
u = tan
2 6
6 x  −2(6 − x) 
 
A = 0 x +3
dx = 
 3
x + 3
0
2u
 
π 2 1 1
1+u 2
= 4 3 ≈ 6.928 square units
58. 0 dθ =   
3 − 2 cos θ 0 2(1 − u 2 )  y
3 − 
 1 + u2  4

1 1 2
= 2 du
0 5u 2 + 1 x
−4 −2 2 4
1
 2  −2
= 
 5
arctan ( 5u
0
) −4

−6
2
= arctan 5
5
2 x
u = tan
θ 64. A =  0 1 + e x2 dx
2
1 2 2 x dx
2  0 1 + e x2
=
sin θ 1 2 sin θ
59.  3 − 2 cos θ dθ =
2  3 − 2 cos θ

( )
2
1 2
x − ln 1 + e x 
2
=
1 2   0
= ln u + C
2 1 1
= 4 − ln (1 + e 4 ) + ln 2
1 2 2
= ln (3 − 2 cos θ ) + C
2 ≈ 0.337 square units
u = 3 − 2 cos θ , du = 2 sin θ dθ y

cos θ (1 − cos θ )
1
cos θ 2
60.  1 + cos θ dθ =  (1 + cos θ )(1 − cos θ ) dθ
1
4
cos θ − cos 2 θ
=  dθ
sin 2 θ x
1 2

 (csc θ cot θ − cot θ ) dθ


2
=

 (csc θ cot θ − (csc θ )


− 1) dθ
2
=
= −csc θ + cot θ + θ + C

sin θ  1 
61.  θ
dθ = 2  sin θ  dθ
2 θ 
= −2 cos θ +C
1
u = θ , du = dθ
2 θ

© 2018 Cengage Learning. All Rights Reserved. May not be scanned, copied or duplicated, or posted to a publicly accessible website, in whole or in part.
818 Chapter 8 Integration Techniques and Improper Integrals

1 x2
65. (a) n = 1: u = ln x, du = dx, dv = x dx, v =
x 2
x2  x2  1 x2 x2
 x ln x dx = 2
ln x −  2 x
  dx =
2
ln x −
4
+C

1 x3
n = 2: u = ln x, du = dx, dv = x 2 dx, v =
x 3
x3  x3  1 x3 x3
x   3  x dx
2
ln x dx = ln x − = ln x − +C
3 3 9
1 x4
n = 3: u = ln x, du = dx, dv = x3 dx, v =
x 4
x4  x4  1 x4 x4
x   4  x dx
3
ln x dx = ln x − = ln x − +C
4 4 16

x n +1 x n +1
 x ln x dx =
n
(b) ln x − +C
n +1 ( n + 1)2
1 x n +1
(c) Let u = ln x, du = dx, dv = x n dx, v =
x n +1

x n +1 x n +1  1 
 x n ln x dx = (ln x) −   dx 
n +1 n + 1x 
n +1
x x n +1
= ln x − + C
( n + 1)
2
n +1

66. (a) Arctangent formula, Formula 23, 2 12,000 x


1
68. W = 0 4.3 − x 2
dx
 u 2 + 1 du , u = e
x
2 x
= 12,000  dx
1 0
(b) Log Rule:  du , u = e x + 1 4.3 − x 2
u 2
= −12,000 4.3 − x 2  dx
(c) Substitution: u = x 2 , du = 2 x dx, then Formula 81  0
≈ 18,311 J
(d) Integration by parts
(e) Formula 16 with u = e2 x

2
67. W = 0 30,000 xe − x dx
2
= − 30,000  − xe − x dx
0
2
= 30,000 
0
(− x)e− x (−1) dx
2
= 30,000 ( − x )e − x − e − x 
0

 3 
= 30,000  − 2 + 1
 e 
≈ 17,819.825 J

© 2018 Cengage Learning. All Rights Reserved. May not be scanned, copied or duplicated, or posted to a publicly accessible website, in whole or in part.
Section 8.7 Integration by Tables and Other Integration Techniques 819

1 2 5000 2500 2 −1.9 dt


2 − 0  0 1 + e 4.8 −1.9t −1.9  0 1 + e 4.8 −1.9t
69. dt =

2500  2
= −  ( 4.8 − 1.9t ) − ln(1 + e4.8 −1.9t ) 0
1.9
2500 
= −
1.9 
(1 − ln(1 + e)) − 4.8 − ln(1 + e4.8 )  ( )
2500   1 + e 
= 3.8 + ln  4.8 
≈ 401.4
1.9   1 + e 

70. (a) The slope of f at x = −1 is approximately 1 0.3


72. (a) V = 6( 2)  0 dy
0.5 ( f ′ > 0 at x = − 1). 0.25 + y 2
1
(b) f ′ > 0 on ( − ∞, 0), so f is increasing on ( − ∞, 0). = 3.6 ln y + 0.25 + y 2 
  0
f ′ < 0 on (0, ∞), so f is decreasing on (0, ∞).
 (
= 3.6 ln 1 + )
1.25 − ln 0.25 

71. y
≈ 5.197 m3

( ( )
8

6
W = 23,2333.6 ln 1 +

1.25 − ln) 0.25 

4 ≈ 120,744 kg
2 (b) By symmetry, x = 0.
x 1 0.3
2 4 6 8 M = ρ ( 2)  dy
0
0.25 + y 2
4
V = 2π  x x 16 − x 2 dx
0 ( ) = 0.6 ρ ln y + 0.25 + y 2 
1

4
  0
= 2π  x 2 16 − x 2 dx
0 = 0.6 ρ ln (1 + .25) − ln 0.25 
By Formula 38: ( a = 4) 1 0.3 y
Mx = 2ρ  dy
0
4 0.25 + y 2
1   x  
V = 2π   x( 2 x 2 − 16) 16 − x 2 + 256 arcsin    1
 8   4   0 = 0.6 ρ 0.25 + y 2 
 0
  π 
= 2π 32  = 32π 2
  2 
= 0.6 ρ ( 1.25 − 0.25 )
y =
Mx
=
0.6 ρ 1.25 − 0.25( )
M 
 (
0.6 ρ ln 1 + 1.25 − ln 0.25 
 )
≈ 0.428
Centroid: ( x , y ) ≈ (0, 0.428)

π 2 dx
73. Let I = 0 1 + ( tan x )
2
.

π
For x = − u , dx = − du , and
2

0 − du π 2 du π 2 ( tan u ) 2

I = π 2 = 0 = 0 du.
1 + ( tan (π 2 − u )) 1 + (cot u ) ( tan u )
2 2 2
+1

π 2 dx π 2 ( tan x) 2
π 2 π
2I = 0 1 + ( tan x)
+ 0 dx = 0 dx =
2
2
( tan x) 2 +1
π
So, I = .
4

© 2018 Cengage Learning. All Rights Reserved. May not be scanned, copied or duplicated, or posted to a publicly accessible website, in whole or in part.
820 Chapter 8 Integration Techniques and Improper Integrals

Section 8.8 Improper Integrals


1. An integral is improper if one or both of the limits of ∞ sin x
integration are infinite, or the function has a finite
11.  −∞ 4 + x2
dx is improper because the limits of
number of infinite discontinuities on the interval of integration are −∞ and ∞.
integration.
π 4
2. An improper integral converges when the limit of the 12. 0 csc x dx is improper because f ( x) = csc x is
integral exists. undefined at x = 0.

3. To evaluate the improper integral a f ( x) dx, find the 13. Infinite discontinuity at x = 0.

limit as b → ∞ when f is continuous on a a , ∞ or find ) 0


4 1
dx = lim b
4 1
dx
x b → 0+ x
the limit as a → − ∞ when f is continuous on ( −∞, b]. 4
= lim 2 x 
b → 0+ b
4. (a) The function has a discontinuity at x = − 2. So, the
integral is improper on [a, 5] for a ≤ − 2. The (
= lim 4 − 2 b = 4
b → 0+
)
integral is also improper when the lower limit of Converges
integration is infinite, that is, when a = − ∞.
14. Infinite discontinuity at x = 3.
1
(b) The function has a discontinuity at x = . So, the
3 4 1 4 −3 2

1
 3 ( x − 3)3 2 dx = lim
b → 3+
 b ( x − 3) dx
integral is improper on [a, 4] for a ≤ . The
3 4
= lim −2( x − 3) 
−1 2
integral is also improper when the lower limit of b → 3+  b
integration is infinite, that is, when a = − ∞.
 2 
= lim −2 +  = ∞
1 dx b→3 
+ b − 3
5.  0 5 x − 3 is improper because 5x − 3 = 0 when
Diverges
3 3
x = , and 0 ≤ ≤ 1. 15. Infinite discontinuity at x = 1.
5 5
2 1 1 1 2 1
dx
2 1
6.  3 is not improper because f ( x) = 3 is continuous
 0 ( x − 1)2 dx =  0 ( x − 1)2 dx +  1 ( x − 1)2 dx
1 x x
b 1 2 1
on [1, 2]. = lim  0 ( x − 1)2 dx + clim  dx
→1+ c ( x − 1)
2
b →1−

1 2x − 5 1 2x − 5  1   1 
b 2
7.  0 x2 − 5x + 6
dx =  0 ( x − 2)( x − 3) dx is not = lim −  + clim
b →1−  x − 1 0
− 
→1+  x − 1 c

improper because = (∞ − 1) + (−1 + ∞)


2x − 5
is continuous on [0, 1]. Diverges
( x − 2)( x − 3)
16. Infinite limit of integration.

 1 ln( x ) dx is improper because the upper limit of



2 0 0
8.
 −∞ e b e
3x 3x
dx = lim dx
b → −∞
integration is ∞. 0
= lim  13 e3 x 
b → −∞ b
2
9. 0 e
−x
dx is not improper because f ( x) = e− x is
= lim  13 − 13 e3b  = 1
3
b → −∞
continuous on [0, 2].
Converges

10. 0 cos x dx is improper because the upper limit of
integration is ∞.

© 2018 Cengage Learning. All Rights Reserved. May not be scanned, copied or duplicated, or posted to a publicly accessible website, in whole or in part.
Section 8.8 Improper Integrals 821

∞ 1 ∞ ∞ 3 b
2 2 x − 3 dx 1  1 3x
−1 3
17. dx = 19. dx = lim dx
x3 3
x b→∞
b b
= lim
b→∞ 2 x − 3 dx 9 
= lim  x 2 3  = ∞
b→∞2 1
b
 x−2 
= lim   Diverges
b→∞ − 2
 2
b ∞ 4 b
 1  1 1 4 x
−1 4
= lim − 2  20. 4
dx = lim dx
b →∞  2x  x b→∞
2
b
 1 1 1 16 
= lim − 2 +  = = lim  x3 4  = ∞ Diverges
b→∞ 3 1
b → ∞  2b 8 8
∞ b
18.
∞ 1
3 ( x − 1)4 dx = lim
b → ∞ 3
b −4
( x − 1) dx 21. 0 e x 3 dx = lim
b→∞ 0 e x 3 dx
b
b
= lim 3e x 3 
b→∞ 0
 −1 
= lim   = lim (3eb 3 − 3)
 ( ) 3
b → ∞ 3 x − 1 3  b→∞

= ∞
 −1 1  1
= lim  +  = Diverges
 (
b → ∞ 3 b − 1
) 3
3(8) 

24

0 0
 − ∞ xe  b xe
−4x −4x
22. dx = lim dx
b → −∞
0
 − x 1  −4x 
= lim 
b → −∞  4
− e  ( Integration by parts)
 16  b
 1 b 1 
= lim − + + e − 4b  = −∞
b → −∞  16 4 16 
Diverges

∞ b ∞ 1 b −3 1
0 x 2e − x dx = lim 0 x e  4 x(ln x)3 dx  4 (ln x)
2 −x
23. dx 25. = lim dx
b →∞ b→∞ x
b
= lim −e − x ( x 2 + 2 x + 2) b
 1 −2 
b →∞ 0
= lim − (ln x) 
b→∞ 2 4
 b 2 + 2b + 2 
= lim  − + 2 = 2
b →∞
 e b
  1 −2 1 −2 
= lim − (ln b) + (ln 4) 
b→∞ 2 2 
 b 2 + 2b + 2 
Because lim  −  = 0 by L’Hôpital’s Rule. 1 1 1
b→∞
 eb  = =
2 ( 2 ln 2)2 2(ln 4)
2

∞ b
24. 0 e − x cos x dx = lim
b→∞ 2 
1 e − x
(−cos x + sin x) ∞ ln x b ln x
1 1
0
26. dx = lim dx
= 1 0 − ( −1) = 1 x b→∞ x
2 2 b
 (ln x)2 
= lim   = ∞
b→∞ 2 
 1
Diverges

© 2018 Cengage Learning. All Rights Reserved. May not be scanned, copied or duplicated, or posted to a publicly accessible website, in whole or in part.
822 Chapter 8 Integration Techniques and Improper Integrals

∞ 4 0 4 ∞ 4
27.  − ∞ 16 + x 2
dx =  − ∞ 16 + x 2
dx + 0 16 + x 2
dx

0 4 c 4
b → −∞  b 16 + c → ∞  0 16 +
= lim 2
dx + lim dx
x x2
0 c
  x    x 
= lim arctan   + lim arctan  
b → −∞
  4 b c→∞
  4  0
  b   c 
= lim 0 − arctan   + lim arctan  − 0
b → −∞
  4  c → ∞
  4 
 π π
= − −  + = π
 2 2
b
x3 x x 1 1  1
dx = lim  ln ( x 2 + 1) +
∞ b b
28. 0 dx = lim 0 dx − lim   = ∞−
2( x + 1) 
( x2 + 1) ( x + 1)
2 2 2
b →∞ x +1 b →∞ 0 2 b →∞  2 2
2
0

Diverges

∞ 1 b ex x  x
b

0 0

dx = lim
29.
e + e− x
x b→∞ 1 + e2 x
dx 32. 0 sin
2
dx = lim −2 cos 
b →∞ 20
b
= lim arctan (e x ) x
b→∞ 0 Diverges because cos does not approach a limit as
π π π 2
= − = x → ∞.
2 4 4
1
ex 1 1  −1  1
0
b
dx = lim  ln (1 + e x ) = ∞ − ln 2

33. dx = lim   = lim  −1 +  = −1 + ∞
30. 0 1 + ex b→∞ 0 x2 b→0 
+ x b b→0 
+ b 
Diverges Diverges

b 5 10 5 10
31. 0
∞ 1 
cos π x dx = lim  sin π x
34. 0 x
dx = lim
b → 0+
b x
dx
b → ∞ π 0
= lim [10 ln x]b
5

Diverges because sin π b does not approach a limit as b → 0+

b → ∞. = lim (10 ln 5 − 10 ln b) = ∞
b → 0+

Diverges

b 2
2 1 1 1 2 1 3 2 3 3 2 3 −3 3
35. 0 3
dx = 0 3
dx + 1 3
dx = lim  ( x − 1)  + lim  ( x − 1)  = + = 0
x −1 x −1 x −1 b →1−  2  0 c →1+  2 c 2 2

8 3 b e e
 0 ln x  0 2 ln x dx
−1 2
0 dx = lim 3  (8 − x)
2
36. dx 38. dx = lim
8− x b → 8− 0 b → 0+

= lim [2 x ln x − 2 x]b
b e
= lim − 6 8 − x  b → 0+
b → 8− 0

= lim ( 2e − 2e) − ( 2b ln b − 2b)


(
= lim − 6 8 − b + 6 8
b → 8−
) b → 0+

= 0
= 12 2
π 2 b
1  x2 x2 
1 39. 0 tan θ dθ = lim ln sec θ  0 = ∞
b → (π 2)−
37.  x ln x dx = lim  ln x − 
0 b→0  2
+ 4 b
Diverges
 −1 b 2 ln b b2  −1
= lim  − +  =
b → 0+  4 2 4 4

because lim (b 2 ln b ) = 0 by L’Hôpital’s Rule.


b → 0+

© 2018 Cengage Learning. All Rights Reserved. May not be scanned, copied or duplicated, or posted to a publicly accessible website, in whole or in part.
Section 8.8 Improper Integrals 823

π 2 b 6 1 b 1
40. 0 sec θ dθ = lim ln sec θ + tan θ  0 = ∞
b → (π 2) 
42. 3 36 − x 2
dx = lim
b → 6−
3 36 − x 2
dx

Diverges x b

= lim arcsin 
b → 6−  6 3
4 2 4 2
41. 2 x 2
x − 4
dx = lim
b → 2+
b x x2 − 4
dx
 b 1
= lim arcsin − arcsin 
4 b → 6−  6 2
 x
= lim arcsec  π π π
b → 2+  2 b = − =
2 6 3
  b 
= lim  arcsec 2 − arcsec   5
+
 2  1
b→2 
dx = lim ln x + x2 − 9 
5

π π
43. 3 x2 − 9 
b → 3+   b
= −0 =
3 3 = lim ln 9 − ln b +

b → 3+ 
( )
b2 − 9 

= ln 9 − ln 3
9
= ln = ln 3
3

5 1 b 1
44.  0 25 − x 2
dx = lim
b → 5−
 0 25 −
x2
dx

b 1  1 1 
= lim   −  dx (partial fractions)
b → 5− 0 10  x + 5 x − 5
b
1 x +5
= lim  ln
b → 5 10
− x − 5  0
= ∞ −0 Diverges

∞ 1 5 1 ∞ 1
45. 3 x 2
x −9
dx = lim
b → 3+
b x 2
x −9
dx + lim
c→∞ 5 x x2 − 9
dx

5 ∞
1 x 1  x 
= lim  arcsec  + lim  arcsec 
b → 3+  3 3 b c → ∞ 3  3  5
1 5 1  b  1 c 1  5  1 π  π
= lim  arcsec  − arcsec  + lim  arcsec  − arcsec  = −0 +   =
+
b→3  3 3
  3 3
  c → ∞
 3 3
  3 3
  3 2
  6

∞ x 2 − 16 b x 2 − 16
46. 4 x2
dx = lim
b→∞ 4 x2
dx
b
− x 2 − 16 
= lim  + ln x + 2
x − 16  (Formula 30)
b→∞
 x  4
 b 2 − 16 
= lim − + ln b + b 2 − 16 − ln 4 = ∞
b→∞
 b 
Diverges

© 2018 Cengage Learning. All Rights Reserved. May not be scanned, copied or duplicated, or posted to a publicly accessible website, in whole or in part.
824 Chapter 8 Integration Techniques and Improper Integrals

∞ 4 1 4 ∞ 4
47. 0 x ( x + 6)
dx = 0 x ( x + 6)
dx + 1 x ( x + 6)
dx

Let u = x , u 2 = x, 2u du = dx.

4 4( 2u du ) du 8  u  8  x
 x ( x + 6)
dx =  u(u 2 + 6)
= 8 2
u + 6
=
6
arctan 
 6
+C =
6
arctan  +C
6 

1 c
∞ 4  8  x   8  x 
So, 0 x ( x + 6)
dx = lim 
+
arctan   + lim  arctan  
6 1
b→0 
 6 6  b  6
c → ∞
 
 8  1    8 π  1  8π 2π 6
(0) +    −
8 8
=  arctan  − arctan   = = .
 6  6 6   62 6  6  2 6 3

1
48.  x ln x dx = ln ln x + C

So,
∞ 1 e 1 ∞ 1 e ∞
1 dx = 1 dx + e dx = lim ln (ln x)1 + lim ln (ln x) e .
x ln x x ln x x ln x b →1+ c →∞

Diverges

∞ 1 b 1 1 1
dx = lim [ln x]1 dx = lim [ln x]a = lim − ln a = ∞.
b 1
49. If p = 1,  dx = lim  50. If p = 1, 
1 x b→∞ 1 x b→∞ 0 x a→0 + a → 0+

= lim (ln b) = ∞. Diverges. For p ≠ 1,


b→∞
1
Diverges. For p ≠ 1, 1 1  x1 − p   1 a1 − p 
b
0 xp dx = lim 
a → 0+ 1 − p 
 = lim 
a → 0+  1 − p

1 − p
.
∞ 1  x1 − p   b1 − p 1  a

1 x p
dx = lim 
b→∞ 1 − p


1
= lim 
b→∞ 1 − p


1 −
.
p 1
This converges to if 1 − p > 0 or p < 1.
1− p
1
This converges to if 1 − p < 0 or p > 1.
p −1

51. For n = 1:
∞ b
0 xe − x dx = lim
b→∞  0
−x
xe dx

(Parts: u = x, dv = e − x dx)
b
= lim −e − x x − e − x 
b→∞ 0

= lim ( −e −bb − e− b + 1)
b→∞

 −b 1 
= lim  b − b + 1 = 1
b → ∞ e
( L'Hôpital's Rule)
e 

Assume that 0 x ne − x dx converges. Then for n + 1 you have

x
n +1 − x
e dx = − x n +1e − x + ( n + 1)  x ne − x dx

(
by parts u = x n + 1 , du = ( n + 1) x n dx, dv = e− x dx, v = −e− x . )
So,
∞ b ∞ ∞
0 x n + 1e − x dx = lim − x n + 1e− x  + ( n + 1)  x ne − x dx = 0 + ( n + 1)  x ne − x dx, which converges.
b→∞ 0 0 0

© 2018 Cengage Learning. All Rights Reserved. May not be scanned, copied or duplicated, or posted to a publicly accessible website, in whole or in part.
Section 8.8 Improper Integrals 825

∞ b b
1 e − x dx = lim 1 e dx = lim −e − b  = 1
−x
52. (a)
b→∞ b→∞ 1

2
Because e− x ≤ e− x on [1, ∞) and

1 e − x dx

converges, then so does


∞ 2
1 e − x dx.

∞ 1
(b) 1 x5
dx converges (see Exercise 49).

1 1 ∞ 1
Because 5
x +1
< 5 on [1, ∞ ), then
x 1 x +15
dx also converges.

1 1 1 1 10 10
53. 0 6
x
dx = 0 x1 6
dx converges by Exercise 50. 62.
x2 − 2 x
=
x( x − 2)
 x = 0, 2.

 1 You must analyze three improper integrals, and each


p = 
 6 must converge in order for the original integral to
converge.
1 1
54. 0 dx diverges by Exercise 50. ( p = 9). 3 1 2 3
x9  0 f ( x) dx =  0 f ( x) dx +  1 f ( x) dx +  2 f ( x) dx
∞ 1 7
dx converges by Exercise 49. ( p = 5)
−1
55. 1 x5
63. A = −∞ − ( x − 1)3 dx
−1 −3

x 4e − x dx converges by Exercise 51. ( n = 4) = lim b − 7( x − 1) dx
56. 0 b → −∞
−1
7 −2 
1 1 ∞ 1 = lim  ( x − 1) 
57. Because 2 ≤ 2 on [1, ∞) and  dx b → −∞  2 b
x + 5 x 1 x2
 7 7 
∞ 1 = lim  − 
converges by Exercise 49,  dx converges.
 ( ) 2(b − 1) 
b → −∞  2 − 2 2 2
1 x2 + 5

7 7
1 1 ∞ 1 = −0 =
58. Because ≥ on [2, ∞) and  dx diverges 8 8
x −1 x 2 x

∞ 1 1
by Exercise 55, 
2 x −1
dx diverges. 64. A =  0 − ln x dx
1

∞ 2 1 − sin x 2
= − lim
b → 0+
 b ln x dx
59. 1 x 2
dx converges, and
x 2
≤ 2 on [1, ∞), so
x = − lim [ x ln x − x]b
1

b → 0+
∞ 1 − sin x
 1 x 2 dx converges. = − lim (0 − 1) − b ln b + b
b → 0+
=1
∞ 1 ∞ 1 1
60. 0 ex
dx = 0 e − x dx converges, and
ex
≥ x
e + x
on
Note: lim b ln b = lim
ln b
= lim
1b
= 0
2
b → 0+ b → 0+ 1b b → 0 −1 b
+
∞ 1
[0, ∞), so 0 ex + x
dx converges.

1 1 0 1 1 1
61.  −1 x3 dx =  −1 x3 dx +  0 x3 dx
These two integrals diverge by Exercise 50.

© 2018 Cengage Learning. All Rights Reserved. May not be scanned, copied or duplicated, or posted to a publicly accessible website, in whole or in part.
826 Chapter 8 Integration Techniques and Improper Integrals

∞ 1 ∞
65. A =  −∞
x +1
dx2
67. (a) A = 0 e − x dx
b
= lim  2
0 1 b
dx + lim  2
1
dx = lim −e − x  = 0 − ( −1) = 1
b→∞ 0
b → −∞ b x +1 b→∞ 0 x +1
0 b (b) Disk:
= lim arctan ( x) b + lim arctan ( x) 0
(e − x )
b → −∞ b→∞ ∞ 2
V = π dx
= lim 0 − arctan (b) + lim arctan (b) − 0 0
b → −∞ b→∞ b
 1  π
 π π = lim π − e −2 x  =
= − −  + = π b→∞  2 0 2
 2 2
(c) Shell:
∞8 ∞
66. A =  dx V = 2π  xe − x dx
− ∞ x2 + 4 0

0 8 b 8 = lim 2π −e − x ( x + 1) = 2π


b
= lim  2 dx + lim  2 dx b→∞ 0
b → −∞ b x + 4 b→∞ 0 x + 4
0 b
  x    x  ∞
= lim 4 arctan   + lim 4 arctan   ∞ 1  1
b → −∞
  2  b b → ∞
  2  0 68. (a) A = 1 x 2
dx = −  = 1
 x 1
  b   b  (b) Disk:
= lim 0 − 4 arctan  + lim 4 arctan  − 0
b → −∞
  2  b→∞
  2   b
∞ 1  π  π
 −π  π  V = π dx = lim − 3  =
= − 4  + 4  = 4π x4 b → ∞  3x  3
1
1
 2  2
(c) Shell:
∞  1  b
V = 2π  x 2  dx = lim 2π (ln x)1 = ∞
1
x  b → ∞

Diverges

69. y = 16 − x 2 , 0 ≤ x ≤ 4
−x
y′ =
16 − x 2
t
4 x2 4 4 t 4   x  t
s = 0 1+
16 − x 2
dx = 0 16 − x 2
dx = lim 
t → 4− 0 16 − x 2
dx = lim 4 arcsin   = lim 4 arcsin   = 2π
t → 4−   4  0 t → 4−  4

70. y = 2e− x
y′ = −2e − x

(2e− x )

S = 2π  1 + 4e −2 x dx
0

Let u = e − x , du = −e − x dx.

e 1 + 4e −2 x dx = − 
−x
1 + 4u 2 du

= − 14 2u 4u 2 + 1 + ln 2u + 4u 2 + 1  + C
 

= − 14 2e − x 4e −2 x + 1 + ln 2e − x + 4e −2 x + 1  + C
 
y

(e − x )
b
S = 4π lim  1 + 4e −2 x dx 2
b →∞ 0
y = 2e−x
b
= −π lim 2e− x

b →∞ 
4e −2 x
+ 1 + ln 2e −x
+ 4e −2 x
+ 1  = π 2 5 + ln 2 +
 0  (  )
5  ≈ 18.5849
x
3 4

−2

© 2018 Cengage Learning. All Rights Reserved. May not be scanned, copied or duplicated, or posted to a publicly accessible website, in whole or in part.
Section 8.8 Improper Integrals 827

K K
71. (a) F ( x) = ,5 = , K = 80,000,000
(4000)
2 2
x
b
∞ 80,000,000  −80,000,000 
W =  4000 x2
dx = lim 
b→∞ x 
 4000
= 20,000 mi-ton ≈ 2.328 × 1011 ft-lb ≈ 3.16 × 1011 joules

b
W  −80,000,000  −80,000,000
(b) = 10,000 =   = + 20,000
2  x  4000 b
80,000,000
= 10,000
b
b = 8000
Therefore, the rocket has traveled 4000 miles, or about 6437 kilometers above Earth’s surface.

k k 75. (a)
, k = 10( 4000 2 )
0.06
72. (a) F ( x) = , 10 =
x2 40002
b
∞ 10( 40002 )  −10( 40002 ) 
W =  4000 x2
dx = lim 
b→∞ x


 4000
150
0
210

10( 4000 2
) Using a graphing utility, the area under the curve is
= = 40,000 mi-ton
4000 approximately 1.
≈ 4.657 × 1011 ft-lb ≈ 6.31 × 1011 joules ∞
(b) P(185 ≤ x ≤ ∞) = 185 f ( x) dx ≈ 0.1587
b
W  −10( 4000 2
)  185
(b) = 20,000 =  (c) 0.5 − P(178 ≤ x ≤ 185) = 0.5 − 178 f ( x) dx
2  x 
4000
≈ 0.5 − 0.3413
−10( 40002 )
= + 40,000 = 0.1587
b
The answers are the same by symmetry:
10( 40002 ) ∞ 185 ∞
= 20,000
b 0.5 = 178 f ( x) dx = 178 f ( x) dx + 185 f ( x) dx.
b = 8000
Therefore, the rocket has traveled 4000 miles, or 76. (a) The area under the curve is greater on the interval
about 6437 kilometers above Earth’s surface. 11 ≤ x ≤ 12 than on the interval 9 ≤ x ≤ 10. So,
the probability is greater for choosing a car getting
∞ ∞ 1 −t between 42 and 44 kilometers per liter.
−∞ f (t ) dt 0
9
73. (a) = e dt
9 (b) The area under the curve is greater on the interval
b
= lim − e − t 9  =1 x ≥ 13 than on the interval 8 ≤ x ≤ 9. So, the
b→∞  0
probability is greater for choosing a car getting at
6 1 −t least 13 kilometers per liter.
(b) P(0 ≤ x ≤ 6) = 0 e 9
dt
9
6
= − e −t 9 
0

= − e− 2 3 + 1
≈ 0.4866 = 48.66%

∞ ∞ 5 − 5t
−∞ f (t ) dt 0
6
74. (a) = e dt
6
b
= lim − e − 5t 6  = 1
b→∞ 0

6 5 − 5t
(b) P(0 ≤ x ≤ 6) = 0 e 6
dt
6
6
= − e − 5t 6 
0

= 1 − e−5
≈ 0.9933 = 99.33%

© 2018 Cengage Learning. All Rights Reserved. May not be scanned, copied or duplicated, or posted to a publicly accessible website, in whole or in part.
828 Chapter 8 Integration Techniques and Improper Integrals

5
77. (a) C = 700,000 + 0 25,000e − 0.06t dt
5
 25,000 − 0.06t 
= 700,000 −  e  ≈ $807,992.41
 0.06 0
10
10  25,000 − 0.06t 
(b) C = 700,000 + 0 25,000e − 0.06t dt = 700,000 − 
 0.06
e  ≈ $887,995.15
0

(c) C = 700,000 + 0 25,000e− 0.06t dt
b
 25,000 − 0.06t 
= 700,000 − lim  e 
b → ∞  0.06 0
≈ $1,116,666.67

5
78. (a) C = 700,000 + 0 25,000(1 + 0.08t ) e − 0.06t dt
5
 1  t 1 
= 700,000 + 25,000 − e − 0.06t − 0.08 e − 0.06t + e − 0.06t 
 0.06  0.06 (0.06)
2 
   0
≈ $828,512.58
10
(b) C = 700,000 + 0 25,000(1 + 0.08t )e− 0.06t dt
10
 1  t 1 
= 700,00 + 25,000 − e− 0.06t − 0.08 e − 0.06t + e− 0.06t 
 0.06  0.06 (0.06)
2 
   ∞
≈ $955,718.14

(c) C = 700,000 + 0 25,000(1 + 0.08t )e − 0.06t dt
b
 1  t 1 
= 700,000 + 25,000 lim − e − 0.06t − 0.08 e − 0.06t + e − 0.06t 
b → ∞  0.06  0.06 (0.06)
2 
   0
≈ $1,672,222.22

2π NI r
79. Let K = . Then
k
∞ 1
P = K dx.
(r + x2 )
c 32
2

Let x = r tan θ , dx = r sec2 θ dθ , r 2 + x 2 = r sec θ .

1 r sec 2 θ dθ 1
 dx =  r 3 sec3 θ = r 2  cos θ dθ
(r 2 + x 2 )
32

1 1 x
= 2
sin θ + C = 2 +C
r r r + x2
2

So,
b
1  x  K  c
P = K 2 lim   = 2 1 −  r2 + x2
r b→∞ 2 2
r + x c r  2
r + c  2 x

=
K ( r 2 + c2 − c ) = 2π NI ( r 2 + c2 − c ). θ
r
2 2 2 2 2
r r + c kr r + c

© 2018 Cengage Learning. All Rights Reserved. May not be scanned, copied or duplicated, or posted to a publicly accessible website, in whole or in part.
Section 8.8 Improper Integrals 829

∞ GM δ (c) As n → ∞, the integral approaches 4(π 4) = π .


80. F =  0 ( a + x)2 dx
π 2 4
b
(d) I n = 0 1 + ( tan x)
n
dx
 −GM δ 
= lim 
b→∞ a + x 0 I 2 ≈ 3.14159
GM δ I 4 ≈ 3.14159
=
a I 8 ≈ 3.14159
81. False. f ( x) = 1 ( x + 1) is continuous on I12 ≈ 3.14159

[0, ∞), xlim


→∞
1 (x + 1) = 0, but 89. You know that
b
a f ( x) dx ≤
b
a f ( x) dx
∞ 1 b Therefore,
0 x +1
dx = lim ln x + 1  0 = ∞. ∞ b
a f ( x) dx a f ( x) dx
b → ∞
= lim
b→∞
Diverges b
f ( x) dx, which coverges.
b → ∞ a
≤ lim
82. False. This is equivalent to Exercise 81.

83. True
So, a f ( x) dx converges.
84. True ∞ 1 b
90. (a) 1 x
dx = lim ln x 1 = ∞
b→∞
85. True b
∞ 1  1
1 1 x2
dx = lim −  = 1
b →∞ x 
86. False. For example, let f ( x) = . 1
x 1∞

1 1 xn
dx will converge if n > 1 and will diverge if
Then lim f ( x ) = lim = 0 = L.
x→∞ x→∞ x n ≤ 1.
∞ ∞ 1 (b) It would appear to converge.
However, 0 f ( x) dx = 0 x
dx diverges.
1.00

∞ 0 ∞
87. (a)  − ∞ sin x dx =  − ∞ sin x dx +  0 sin x dx
0 c
= lim  sin x dx + lim  sin x dx 0 20
b → −∞ b c→∞ 0
−0.25
= lim [−cos x]b + lim [−cos x]0
0 c
b → −∞ c→∞ (c) Let dv = sin x dx  v = −cos x
Because lim [−cos b] diverges, as does lim [−cos c], 1 1
b → −∞ c →∞ u =  du = − dx.
x x2

 − ∞ sin x dx diverges. ∞ sin x  cos x  ∞ cos x
b

1 x
dx = lim −
b → 0 x 1
− 
1 x2
dx
a
sin x dx = lim [−cos x]− a
a
(b) lim  ∞ cos x
a→∞ −a a →∞ = cos 1 −  dx
1 x2
= lim −cos( a ) + cos( − a ) = 0
a →∞
∞ cos x ∞ cos x ∞ 1
∞ Because 1 dx ≤ 1 dx ≤ 1 dx,
(c) The definition of  − ∞ f ( x) dx is not x2 x2 x2
1

lim 
a
f ( x) dx.
and 1x2
dx converges, then Exercise 89 implies
a →∞ − a ∞ cos x
that  dx converges. Finally, you see that
1 x2
88. (a) Yes, the integrand is not defined at x = π 2.
∞ sin x
(b) 5
1 x dx converges.
91. f (t ) = 1
π
0 b
2
∞  1  1
F ( s) = 0 e − st dt = lim − e − st  = , s > 0
−2 b →∞ s 0 s

© 2018 Cengage Learning. All Rights Reserved. May not be scanned, copied or duplicated, or posted to a publicly accessible website, in whole or in part.
830 Chapter 8 Integration Techniques and Improper Integrals

92. f (t ) = t 95. f (t ) = cos at


b ∞
∞ 1  F ( s) = 0 e − st cos at dt
F (s) = 0 te − st dt = lim  2 ( − st − 1)e − st 
b→∞ s 0 b
 e − st 
1
= 2, s > 0
= lim  2
b → ∞ s + a2
(− s cos at + a sin at )
s  0
s s
93. f (t ) = t 2 = 0+ 2 = 2 ,s > 0
s + a2 s + a2
b
1 
t 2e − st dt = lim  3 ( − s 2t 2 − 2 st − 2)e − st 

F (s) = 0 96. f (t ) = sin at
b →∞ s 0

2 F ( s) = 0 e − st sin at dt
= 3, s > 0
s b
 e − st 
94. f (t ) = e at
= lim  2
b → ∞ s + a2
(− s sin at − a cos at )
 0
∞ ∞
F (s) = et(a − s) dt a a
0 e at e − st dt = 0 = 0+ 2
s + a2
= 2
s + a2
,s > 0
b
 1 t(a − s) 
= lim  e 
b →∞a − s 0
1 1
= 0− = ,s > a
a − s s − a

97. f (t ) = cosh at
∞ ∞  e at + e − at  1 ∞ t (− s + a)
F (s) = 0 e − st cosh at dt = 0 e − st   dt =  0 e + et (− s − a)  dt
2 2 
 
b
1 1 1  1 1 1 
= lim  et ( − s + a ) + et ( − s − a )  = 0 −  + 
b → ∞ 2  (− s + a) (− s − a) 2  ( − s + a) ( − s − a ) 
  0
−1  1 1  s
=  +  = 2 ,s > a
2  ( − s + a ) ( − s − a )  s − a2

98. f (t ) = sinh at
∞ ∞  e at − e − at  1 ∞ t (− s + a)
F (s) = 0 e − st sinh at dt = 0 e − st   dt =  0 e − et (− s − a)  dt
2 2 
 
b
1 1 1  1 1 1 
= lim  et ( − s + a ) − et ( − s − a )  = 0 −  − 
b→∞ 2  ( − s + a ) ( − s − a)  0 2  ( − s + a ) ( − s − a ) 

−1  1 1  a
=  −  = 2 ,s > a
2  ( − s + a ) ( − s − a )  s − a2


99. Γ( n) = 0 x n −1e − x dx

∞ b
(a) Γ(1) = 0 e − x dx = lim −e − x  = 1
b→∞ 0
∞ b
Γ ( 2) = 0 xe − x dx = lim −e − x ( x + 1) = 1
b→∞ 0
∞ b
Γ(3) = 0 x 2e− x dx = lim − x 2e − x − 2 xe − x − 2e − x  = 2
b→∞ 0

(u = x n , dv = e − x dx)
∞ b b
(b) Γ( n + 1) = 0 x ne − x dx = lim − x ne − x  + lim n  x n −1e − x dx = 0 + nΓ( n)
b→∞ 0 b→∞ 0

(c) Γ( n) = ( n − 1)!

© 2018 Cengage Learning. All Rights Reserved. May not be scanned, copied or duplicated, or posted to a publicly accessible website, in whole or in part.
Section 8.8 Improper Integrals 831

100. For n = 1,
b
 
x 1 b 1 1  = 1.
dx = lim  ( x 2 + 1) ( 2 x dx) = lim − ⋅
∞ −4
I1 = 0 b→∞ 6 3
( x2 + 1) ( x + 1) 0 6
4 b→∞ 2 0 2

For n > 1,
b
 
∞ x 2 n −1 − x2n − 2  + n −1 ∞ x 2n − 3 n −1
In = 0 dx = lim   dx = 0 + ( I n −1 )
b→∞ 
( x2 + 1)  2( n + 2)( x + 1)  0 ( x + 1)
n+3 n + 2 n+2
2 n + 2 0 2 n + 2

 
 Parts: u = x 2 n − 2 , du = ( 2n − 2) x 2 n − 3 dx, dv = x −1 
dx, v =
 ( x 2 + 1) 2( n + 2)( x 2 + 1) 

n + 3 n + 2

b
 
∞ x 1  = 1
(a) 0 dx = lim −
b→∞ 3
( x2 + 1)  6( x + 1)  0
4
2 6

∞ x3 1 ∞ x 1 1  1
(b) 0 dx =
4  0 ( x 2 + 1)4
dx =   =
( x2 + 1)
5
4 6  24

∞ x5 2 ∞ x3 2 1  1
(c) 0 =  dx =   =
( x2 + 1) 5 0 ( x 2 + 1)5
6
5  24  60

∞ 1 c  b 1 c 
101. 0 
 2
x +1
−  dx = lim  0 
x + 1 b→∞
 2
x +1
−  dx
x + 1
b
= lim ln x + x 2 + 1 − c ln x + 1 

b →∞  0
b + b2 + 1

b →∞
(
= lim ln b + )
b 2 + 1 − ln (b + 1)  = lim ln 
c


 (b + 1)
b→∞  c


This limit exists for c = 1, and you have


b + b2 + 1
lim ln   = ln 2.
b→∞
 (b + 1) 

∞ cx 1 b cx 1
102. 1  2 −  dx = blim
 x + 2 3x  → ∞  1  2 −  dx
 x + 2 3x 
b
c 1 
= lim  ln ( x 2 + 2) − ln x 
b →∞2 3 1
b
 x2 + 2 c 2 
= lim ln 
( ) 
b→∞  x13 
 1
 b2 + 2 c 2
= lim ln
( ) − ln 3c 2 
b →∞ b1 3 
 
This limit exists if c = 1 3, and you have

 b2 + 2 1 6
lim ln
( ) − ln 31 6  = −ln 31 6 = −ln 3.
b →∞ b1 3  6
 

© 2018 Cengage Learning. All Rights Reserved. May not be scanned, copied or duplicated, or posted to a publicly accessible website, in whole or in part.
832 Chapter 8 Integration Techniques and Improper Integrals

x ln x, 0 < x ≤ 2
103. f ( x) = 
0, x = 0
2
( x ln x)
2
V = π dx
0

Let u = ln x, eu = x, eu du = dx. y

e u (e du )
ln 2
V = π 2u 2 4
2
−∞
y = x ln x
ln 2
= π e3u u 2 du 1
−∞
ln 2
 u 2 2u 2  3u  x
= lim π  − + e  1 2

 3
b → −∞  9 27   b
−1

 (ln 2)2 2 ln 2 2
= 8π  − +  ≈ 2.0155 −2
 3 9 27 

1
104. V = π  (−ln x)2 dx 105. u = x , u 2 = x, 2u du = dx
0
sin x sin (u 2 )
 0 2 sin(u ) du
1 1 1
= lim π  (ln x) dx
1 2
 0 x dx = 0 (2u du ) = 2

b → 0+ b u

= lim π x (ln x) − 2 ln x + 2
2 1
Trapezoidal Rule ( n = 5): 0.6278
b → 0+  b

= lim π 2 − b(ln b) − 2b ln b − 2b


2
106. u = 1 − x , 1 − x = u 2 , 2u du = −dx
b→0 +  
= 2π 1 cos x 0 cos(1 − u 2 )
y 0 dx = 1 (−2u du )
1− x u

 0 2 cos(1 − u ) du
3 1
2
=

y = − ln x Trapezoidal Rule ( n = 5): 1.4997


x
1 2

−3

107. Assume a < b. The proof is similar if a > b.


a ∞ a d
 −∞ f ( x) dx +  a f ( x) dx c →−∞  c
= lim f ( x) dx + lim  f ( x) dx
d →∞ a
a
 f ( x) dx +
b
d
= lim  f ( x) dx + dlim
c →−∞ c  a
→∞   b f ( x) dx
y
a b d
3 = lim  f ( x) dx +  a f ( x) dx + dlim
c →−∞ c →∞  b
f ( x) dx
2
= lim  f ( x) dx + 
a b d
1
c →−∞ 
 c  a f ( x) dx + dlim
→∞  b
f ( x) dx
x b d
−1
a b
= lim  f ( x) dx + dlim
c →−∞ c →∞  b
f ( x) dx
−2 b ∞
=  −∞ f ( x) dx +  b f ( x) dx

© 2018 Cengage Learning. All Rights Reserved. May not be scanned, copied or duplicated, or posted to a publicly accessible website, in whole or in part.
Review Exercises for Chapter 8 833

Review Exercises for Chapter 8


1 1
 (x − 27)
12
1. x
2
x3 − 27 dx = 3
3x 2 dx 6. Let u = 2 x − 3, du = 2 dx, x = (u + 3).
3 2

1 ( x − 27)
32
1 2 1 
3 2 1
= +C  3 2 2x 2 x − 3 dx =  0 (u + 3)u   du
 2
3 (3 2)
1 1 32
=
2 3
( x − 27) + C
32 =
20
(u + 3u1 2 ) du
9
1
1 2 5 2 
= u + 2u 3 2 
5 − x2 1 2
dx = −  e5 − x ( − 2 x dx) 2  5 0
2.  xe 2
1 2  6
1 5 − x2 =  + 2 =
= − e +C 2 5  5
2
100  x
3. 
 x + 8
csc 2  2 x + 8 1
 dx = 4  csc   dx
7.  100 − x
dx = 100 arcsin   + C
2
 10 
 4   4 4
 x + 8 2x  6 
= − 4 cot   +C
 4  8.  x − 3 dx =   2 +  dx
x − 3

x 1 = 2 x + 6 ln x − 3 + C
(4 − x 2 ) (−2 x) dx
−1 3
4.  3
4 − x2
dx = −
2
9. Let u = x, du = dx, dv = e1 − x dx, v = − e1 − x .
1 (4 − x )
23
2
= − +C  xe e
1− x
dx = − xe1 − x + 1− x
dx
2 (2 3)
3 = − xe1 − x − e1 − x + C
( 4 − x2 ) + C
23
= −
4
10. Let u = x 2 , du = 2 x dx, dv = e x 2 , v = 2e x 2
1
5. Let u = ln ( 2 x ), du = xe  2e (2 x) dx
2 x2
dx. dx = 2 x 2e x 2 − x2
x
e ln ( 2 x) 1 + ln 2
= 2 x 2e x 2 − 4  xe x 2 dx
1 x
dx =  ln 2 u du
Use integration by parts again with u = x, du = dx,
2 1 + ln 2
u  dv = e x 2 , v = 2e x 2 .
= 
2  ln 2
= 2 x 2e x 2 − 4 2 xe x 2 − (  2e
x2
dx )
1
= 1 + 2 ln 2 + (ln 2) − (ln 2) 
2 2
2 x 2 x2 x2
2  = 2x e − 8 xe + 16e +C

=
1
+ ln 2 ≈ 1.1931
= e x2
(2 x 2
+ 8 x + 16) + C
2

1 2
e sin 3x dx = − e 2 x cos 3x +  e 2 x cos 3 x dx
2x
11.
3 3
1 2x 2 1 2 
= − e cos 3x +  e 2 x sin 3 x −  e2 x sin 3x dx 
3 3 3 3 
13 2 x 1 2x 2 2x
9
e sin 3x dx = − e cos 3x + e sin 3 x
3 9
e2 x
e
2x
sin 3x dx = ( 2 sin 3x − 3 cos 3x) + C
13
1 1
(1) dv = sin 3 x dx  v = − cos 3 x (2) dv = cos 3x dx  v =
sin 3 x
3 3
u = e2 x  du = 2e 2 x dx u = e2 x  du = 2e 2 x dx

© 2018 Cengage Learning. All Rights Reserved. May not be scanned, copied or duplicated, or posted to a publicly accessible website, in whole or in part.
834 Chapter 8 Integration Techniques and Improper Integrals

12. x x − 1 dx = 2x
3
(x − 1)
32
−  23 ( x − 1)
32
dx 13. Let u = x, du = dx, dv = sec 2 x dx, v = tan x.

(x − 1) (x − 1)  x sec  tan x dx
32 52 2
= 2x − 4 +C x dx = x tan x −
3 15

= 2
(x − 1)
32
(5 x − 2( x − 1)) + C = x tan x + ln cos x + C
15

(x − 1) (3 x + 2) + C
2 32
= 15

dv = ( x − 1) (x − 1)
12 2 32
dx  v = 3
h = x  du = dx

1
 ln ln ( x 2 − 4) dx
2
14. x 2 − 4 dx =

1 2x2 
=  x ln ( x 2 − 4) −  x 2 − 4 dx
2
1  4 
= x ln ( x 2 − 4) −  1 + 2  dx
2  x − 4
1 x−2
= x ln ( x 2 − 4) − x − ln +C
2 x+2
dv = dx  v = x
2x
u = ln ( x 2 − 4)  du = dx
x2 − 4

x2 x2
15.  x arcsin 2 x dx =
2
arcsin 2 x −  1 − 4x2
dx

( 2 x)
2
x2 1
= arcsin 2 x −  dx
2 4 1 − ( 2 x)
2

x2 1 1 
arcsin 2 x −   −( 2 x) 1 − 4 x 2 + arcsin 2 x + C ( by Formula 43 of Integration Tables)
=
2 4  2  
1
= ( 4 x 2 − 1)arcsin 2 x + 2 x 1 − 4 x 2  + C
8 
2
x
dv = x dx  v =
2
2
u = arcsin 2 x  du = dx
1 − 4x2

2x
16.  arctan 2 x dx = x arctan 2 x −  1 + 4 x 2 dx
1
= x arctan 2 x − ln (1 + 4 x 2 ) + C
4
dv = dx  v = x
2
u = arctan 2 x  du = dx
1 + 4 x2

cos5 x
17.  sin x cos 4 x dx = −
5
+C (u = cos x, du = − sin x dx)

 sin  sin x cos x cos x dx


2
18. x cos3 x dx = 2 2

 sin x(1 − sin x) cos x dx


2 2
=
sin 3 x sin 5 x
= − +C
3 5

© 2018 Cengage Learning. All Rights Reserved. May not be scanned, copied or duplicated, or posted to a publicly accessible website, in whole or in part.
Review Exercises for Chapter 8 835

 cos (π x − 1) dx  1 − sin (π x − 1)cos(π x − 1) dx


3 2
19. =
1 1 
= sin (π x − 1) − sin 3 (π x − 1) + C
π  3 
1
= sin (π x − 1) 3 − sin 2 (π x − 1) + C

1
= sin (π x − 1) 3 − (1 − cos 2 (π x − 1)) + C

1
= sin (π x − 1) 2 + cos 2 (π x − 1) + C

πx 1 1 1  1
 sin  2 (1 − cos π x) dx  x − sin π x  + C = (π x − sin π x) + C
2
20. dx = =
2 2 π  2π

4 x  2 x  2 x 
21.  sec  2  dx =  tan   + 1sec   dx
 2  2
2 x 2 x  2 x
=  tan   sec   dx +
 2  2
 sec  2  dx
2  x  x 2  x  x 
= tan 3   + 2 tan   + C = tan 3   + 3 tan   + C
3  2  2 3  2  2 

 tan θ sec θ dθ =  ( tan θ + tan θ ) sec2 θ dθ + 1


tan 4 θ + 1
tan 2 θ + C1
4 3
22. 4 2

 tan θ sec θ dθ =  sec θ (sec θ tan θ ) dθ 1


sec4 θ + C2
4 3
or + 4

23. Let u = x 2 , du = 2 x dx.


1
 x tan 2
4
x 2 dx = tan 4 u du

1
tan 2 u (sec2 u − 1) du
2
=

1 1
2 2
= tan 2 u sec 2 u du − tan 2 u du

1 1
tan 2 u sec 2 u du −  (sec 2 u − 1) du
2
=
2
1 tan 3 u 1 1
= ⋅ − tan u + u + C
2 3 2 2
1 1 x2
= tan 3 x 2 − tan x 2 + +C
6 2 2

tan 2 x sin 2 x
24.  sec3 x
dx =  cos 2 x
(cos3 x) dx
 sin
2
= x cos x dx
1
= sin 3 x + C
3

1 1 1 + sin θ 1 + sin θ
 1 − sin θ dθ =  1 − sin θ dθ =  dθ =  (sec θ + sec θ tan θ ) dθ = tan θ + sec θ + C
2
25. ⋅
1 + sin θ cos 2 θ

 cos 2θ (sin θ + cos θ ) dθ =  (cos θ − sin θ )(sin θ + cos θ ) dθ


2 2 2 2
26.

 (sin θ + cos θ ) (cos θ − sin θ ) dθ = 14 (sin θ + cos θ ) + C


3 4
=

© 2018 Cengage Learning. All Rights Reserved. May not be scanned, copied or duplicated, or posted to a publicly accessible website, in whole or in part.
836 Chapter 8 Integration Techniques and Improper Integrals

2
3π 4  1 − cos 2 x  1 1 1
π 4     4 cos
2
27. A = sin 4 x dx =  dx = 2x − cos 2 x +  dx
2  2 4
3π 4
1 1 3x 
=  sin 4 x − sin 2 x +
 32 4 8 π 4

 1 9π   1 3π 
= 0 + +  − 0 − + 
 4 32   4 32 
≈ 1.0890

π 4
30. x = 3 sec θ , dx = 3 sec θ tan θ dθ , x 2 − 9 = 3 tan θ
28. A = 0 sin 3 x cos 2 x dx

1 π /4 x2 − 9 3 tan θ
[sin x + sin 5 x] dx   3 sec θ (3 sec θ tan θ dθ )
20
= dx =
x
π 4
1 1  = 3 tan 2 θ dθ
= −cos x − cos 5 x
2  5 0
= 3 (sec 2 θ − 1) dθ
1 2 1 2 1
= − − −  +1+  = 3( tan θ − θ ) + C
2 2 5 2  5
 x
3 2 = x 2 − 9 − 3 arcsec  + C
= − ≈ 0.317  3
5 5
x
29. x = 2 sin θ , dx = 2 cos θ dθ , 4 − x 2 = 2 cos θ x2 − 9
θ
−12 −24 cos θ dθ
 x2 4− x 2
dx =  (4 sin 2 θ )(2 cos θ ) 3

= −3 csc 2 θ dθ
= 3 cot θ + C
3 4 − x2
= +C
x

2
x

4 − x2

31. x = 2 tan θ , dx = 2 sec 2 θ dθ , 4 + x 2 = 4 sec 2 θ

x3 8 tan 3θ
 4+ x 2
dx =  2 sec θ 2 sec 2 θ dθ

= 8 tan 3 θ sec θ dθ

= 8 (sec 2 θ − 1) tan θ sec θ dθ

 sec3θ 
= 8 − sec θ  + C
 3 
 ( x 2 + 4)3 2 x2 + 4 
= 8 −  +C
 24 2 
1 
= x 2 + 4  ( x 2 + 4) − 4 + C x2 + 4
3 
x
1 8
= x2 x2 + 4 − x2 + 4 +C θ
3 3 2
1 12
= ( x 2 + 4) ( x 2 − 8) + C
3

© 2018 Cengage Learning. All Rights Reserved. May not be scanned, copied or duplicated, or posted to a publicly accessible website, in whole or in part.
Review Exercises for Chapter 8 837

1 2
32.  25 − 9 x 2 dx =
3
52 − (3x) (3) dx

1 1  3x   25  3x  x
= 25 arcsin   + 3x 25 − 9 x 2  + C = arcsin  + 25 − 9 x 2 + C
3 2  5  6 5 2
(Theorem 8.2)

33. x = 4 tan θ , dx = 4 sec2 θ dθ , 16 + x 2 = 4 sec θ

6 ( 4 tan θ )
3
6 x3
 16 + x 2
dx =  4 sec θ
4 sec 2 θ dθ

= 384  tan 3θ sec θ dθ

= 384  (sec 2θ − 1) sec θ tan θ dθ

 sec3 θ 
= 384  − sec θ  + C
 3 

384 (16 + x )
32
2
384 16 + x 2
= ⋅ − + C x 2 + 16
3 64 4 x
= 2 x 2 + 16 (16 + x 2 − 48) + C θ

x + 16 ( x − 32) + C
2 2 4
= 2

6 x3 1
x 2 + 16 ( x 2 − 32)
1
0 dx = 2
16 + x 2  0

= 2 17 ( − 31) − 2( 4)( − 32)


= 256 − 62 17

34. x = 3 sec θ , dx = 3 sec θ tan θ dθ , x 2 − 9 = 3 tan θ

x  27 sec θ (3 tan θ ) 3 sec θ tan θ dθ


3
x 2 − 9 dx = 3

= 243 sec 4θ tan 2 θ dθ

= 243 (1 + tan 2θ ) tan 2 θ sec 2 θ dθ

 tan 3 θ tan 5 θ 
= 243 +  +C
 3 5 
 x2 − 9 3 2 x 2 − 9) 
( ) (
52

= 243  +  +C
 81 1215 
 
4
 x2 − 9 3 2 x 2 − 9) 
( ) (
52
4
 
3 x
3 2
x − 9 dx = 243 +
 81 1215 
 3
 73 2 75 2 
= 243 + 
 81 1215 
7 7 49 7 
= 243 +  x
 81 1215  x2 − 9

154 θ
= 7 3
5

© 2018 Cengage Learning. All Rights Reserved. May not be scanned, copied or duplicated, or posted to a publicly accessible website, in whole or in part.
838 Chapter 8 Integration Techniques and Improper Integrals

x3 x2
35. (a) Let x = 2 tan θ , dx = 2 sec2 θ dθ . (b)  4+ x 2
dx =  4 + x2
x dx

x3 8 tan 3 θ (u 2 − 4)u du
 4+ x 2
dx =  2 sec θ
2 sec 2 θ dθ
=  u
= 8 tan 3θ sec θ dθ =  (u 2 − 4) du
sin 3 θ 1 3
= 8 dθ = u − 4u + C
cos 4 θ 3
= 8 (1 − cos 2 θ ) cos −4θ sin θ dθ u
= (u 2 − 12) + C
3
= 8 (cos −4 θ − cos −2 θ ) sin θ dθ
4 + x2 ( 2
 cos −3 θ cos −1 θ  = x − 8) + C
= 8 −  +C 3
 3 −1 
8 u 2 = 4 + x 2 , 2u du = 2 x dx
= sec θ (sec 2θ − 3) + C
3
8  4 + x 2  4 + x 2 
4 + x2 =   − 3 + C
 
x 3 2  4 
θ 1
= 4 + x 2 ( x 2 − 8) + C
2 3
x3
(c)  4 + x 2
dx = x 2 4 + x2 −  2x 4 + x 2 dx

2( 32 4 + x2 ( 2
= x2 4 + x2 − 4 + x2 ) + C = x − 8) + C
3 3
x
dv = dx  v = 4 + x2
4 + x2
u = x2  du = 2 x dx

36. (a) x 4 + x dx = 64 tan 3 θ sec3 θ dθ (b)  x 4 + x dx = 2 (u 4 − 4u 2 ) du


= 64  (sec 4 θ − sec 2 θ ) sec θ tan θ dθ 2u 3 ( 2
= 3u − 20) + C
64 sec3 θ 15
= (3 sec3 θ − 5) + C 2( 4 + x)
32
15 (3x − 8) + C
=
2( 4 + x)
32 15
= (3 x − 8) + C
15 u 2 = 4 + x, dx = 2u du
x = 4 tan 2 θ , dx = 8 tan θ sec 2 θ dθ ,
4 + x = 2 sec θ
2x
( 4 + x)3 2 − 2  ( 4 + x)3 2 dx
x  (u − 4u1 2 ) du (d)  x 4 + x dx =
32
(c) 4 + x dx =
3 3
2u 3 2 2x
= (3u − 20) + C = ( 4 + x ) − 4 ( 4 + x )5 2 + C
32

15 3 15
32 32
2( 4 + x) 2( 4 + x)
= (3x − 8) + C = (3 x − 8) + C
15 15
u = 4 + x , du = dx 2 32
dv = 4 + x dx  v = ( 4 + x)
3
u = x  du = dx

© 2018 Cengage Learning. All Rights Reserved. May not be scanned, copied or duplicated, or posted to a publicly accessible website, in whole or in part.
Review Exercises for Chapter 8 839

x −8 x −8 A B 5x − 2 5x − 2 A B
37. = = + 38. = = +
x2 − x − 6 ( x − 3)( x + 2 ) x − 3 x + 2 2
x − x (
x x −1 ) x x −1
x − 8 = A( x + 2) + B( x − 3) 5 x − 2 = A( x − 1) + Bx

When x = − 2, − 10 = − 5 B  B = 2. When x = 1, 3 = B.
When x = 3, − 5 = 5 A  A = − 1. When x = 0, − 2 = − A  A = 2.

x −8  −1 2  5x − 2 2 3 
 2
x − x −6
dx =   x − 3 +  dx
x + 2
 x2 − x
dx =   x +  dx
x − 1
= − ln x − 3 + 2 ln x + 2 + C = 2 ln x + 3 ln x − 1 + C

(x + 2)
2
= ln +C
x −3

x2 + 2x A Bx + C
39. = + 2
( x − 1)( x 2 + 1) x −1 x +1
x + 2 x = A( x + 1) + ( Bx + C )( x − 1)
2 2

3
When x = 1, 3 = 2A  A = .
2
3
When x = 0, 0 = A−C  C = .
2
1
When x = 2, 8 = 5 A + 2B + C  B = − .
2
x2 + 2 x 3 1 1 x −3
 x3 − x2 + x − 1
dx = 
2 x −1
dx −  2
2 x +1
dx

3 1 1 2x 3 1
=  dx −  2 dx +  2 dx
2 x −1 4 x +1 2 x +1
3 1 3
= ln x − 1 − ln x 2 + 1 + arctan x + C
2 4 2
1 ( )
= 6 ln x − 1 − ln x + 1 + 6 arctan x + C
2
4

4x − 2 A B
40. 2
= +
3( x − 1) x − 1 ( x − 1)2
4 x − 2 = 3 A( x − 1) + 3B
2
When x = 1, 2 = 3B  B = .
3
4
When x = 2, 6 = 3 A + 3B  A = .
3
4x − 2 4 1 2 1 4 2 2 1 
 3( x − 1)2 dx = 
3 x −1
dx + 
3 ( x − 1) 2
dx = ln x − 1 −
3 (
3x −1)
+ C =  2 ln x − 1 −
3
+C
x − 1

© 2018 Cengage Learning. All Rights Reserved. May not be scanned, copied or duplicated, or posted to a publicly accessible website, in whole or in part.
840 Chapter 8 Integration Techniques and Improper Integrals

x2 2x − 1 A B
41. =1+ 2 =1+ +
2
x − 2x + 1 x − 2x + 1 x − 1 ( x − 1)2
2 x − 1 = A( x − 1) + B
When x = 1, B = 1.
When x = 0, −1 = − A + B = − A + 1  A = 2.

x2  2 1 
 2
x − 2x + 1
dx =  1 + +
x − 1 ( x − 1)
 dx
2
 
1
= x + 2 ln x − 1 + +C
1− x

x3 + 4 16 x + 4
42. 2
= x + 4+ 2
x − 4x x − 4x
16 x + 4 A B
= +
x2 − 4x x x − 4
16 x + 4 = A( x − 4) + Bx
When x = 4, 68 = 4 B  B = 17.
When x = 0, 4 = − 4 A  A = −1.

x3 + 4  1 17 
 x2 − 4 x
dx =   x + 4 − x
+  dx
x − 4
x2
= + 4 x − ln x + 17 ln x − 4 + C
2

43. Let u = e x , du = e x dx.


4 4 A B C
= = + +
(e2 x − 1)(e x + 3) (u 2 − 1)(u + 3) u −1 u +1 u + 3
4 = A(u + 1)(u + 3) + B(u − 1)(u + 3) + C (u − 1)(u + 1)

1
When u = 1, 4 = 8 A  A = .
2
When u = −1, 4 = − 4 B  B = −1.
1
When u = − 3, 4 = 8C  C = .
2
4e x 4
 (e 2 x − 1)(e x + 3)
dx =  (u 2 − 1)(u + 3)
du

 12 1 12 
=   u − 1 − +  du
u + 1 u + 3
1 1
= ln u − 1 − ln u + 1 + ln u + 3 + C
2 2
1 1
= ln e x − 1 − ln e x + 1 + ln e x + 3 + C
2 2

© 2018 Cengage Learning. All Rights Reserved. May not be scanned, copied or duplicated, or posted to a publicly accessible website, in whole or in part.
Review Exercises for Chapter 8 841

44. u = tan θ , du = sec2 θ dθ


1 A B
= +
u (u − 1) u u −1
1 = A(u − 1) + Bu
When u = 0, 1 = − A  A = −1.
When u = 1, 1 = B.
sec θ2
1 1 1
tan θ ( tan θ − 1)
dθ =  u(u − 1) du =  u − 1 du −  u du
tan θ − 1
= ln u − 1 − ln u + C = ln + C = ln 1 − cot θ + C
tan θ

3 2
45. Trapezoidal Rule ( n = 4):  2 1 + x 2 dx
1 2  2   2   2  2 
≈  + 2  + 2  + 2 +  ≈ 0.2848
8 1 + 2 2  1 + (9 4) 
2  1 + (5 2) 
2  1 + (11 4)  1 + 32 
2
       
3 2
Simpson’s Rule ( n = 4):  2 1 + x 2 dx
1 2  2   2   2  2 
≈  + 4  + 2  + 4  +  ≈ 0.2838
12 1 + 2 2  1 + (9 4) 
2  1 + (5 2) 
2  1 + (11 4)  1 + 32 
2
       
Graphing utility: 0.2838

1 2(1 4) 2(1 2) 2(3 4) 1


32 32 32
1 x3 2
46. Trapezoidal Rule ( n = 4): 0 dx ≈ 0 + + + +  ≈ 0.1719
3 − (1 4) 3 − (1 2) 3 − (3 4)
2 2 2 2
3− x 8 2
 

1 4(1 4) 2(1 2) 4(3 4) 1


32 32 32
1 x3 2
Simpson’s Rule ( n = 4): 0 dx ≈  0 + + + +  ≈ 0.1661
3 − (1 4) 3 − (1 2) 3 − (3 4)
2 2 2 2
3− x 12  2 

Graphing utility: 0.1657

π 2 π 2π 2 1 2π 6π 2 1 
47. Trapezoidal Rule ( n = 4): 0 x cos x dx ≈ 0 + + + + − + + 0 ≈ 0.6366
16  2 4 2 2 2 4 2 

π 2 π  2 1 2π 2 1 
Simpson’s Rule ( n = 4): 0 x cos x dx ≈ 0 + 2π + + + 6π − + + 0 ≈ 0.6845
24  4 2 2 4 2 

Graphing utility: 0.7041

π π
48. Trapezoidal Rule ( n = 4): 0 1 + sin 2 x dx ≈ 1+ 6 + 2 2 + 6 + 1 ≈ 3.8199
8
π π
Simpson’s Rule ( n = 4): 0 1 + sin 2 x dx ≈ 1 + 2 6 + 2 2 + 2 6 + 1 ≈ 3.8292
12 
Graphing utility: 3.8202

© 2018 Cengage Learning. All Rights Reserved. May not be scanned, copied or duplicated, or posted to a publicly accessible website, in whole or in part.
842 Chapter 8 Integration Techniques and Improper Integrals

49. Using Formula 4: ( a = 4, b = 5) 51. Let u = x 2 , du = 2 x dx.


x 1 4  π 2 x 1 π4 1
 (4 + 5x)2 dx = 
25  4 + 5 x
+ ln 4 + 5 x  + C
 0 1 + sin x 2
dx = 
2 0 1 + sin u
du

1
50. Using Formula 21: ( a = 4, b = 5) = [tan u − sec u]π0 4
2
−2(8 − 5 x ) 1
x = (1 − 2 ) − (0 − 1)
 4 + 5x
dx =
75
4 + 5x + C
2
10 x − 16 2
= 4 + 5x + C =1−
75 2

52. Let u = x 2 , du = 2 x dx.


1 x 1 1 1
 0 1 + e x2 dx =
2  0 1 + eu
du

1 1
u − ln (1 + e ) 0
u
=
2
1
= (1 − ln (1 + e)) + ln 2
2
1  2 
= 1 + ln  
2   1 + e 

x 1 1 
53.  x2 + 4x + 8
dx = ln x 2 + 4 x + 8 − 4 2
2
dx
x + 4 x + 8 
(Formula 15)

1  2  2 x + 4 
= ln x 2 + 4 x + 8  − 2  arctan   + C (Formula 14)
2  32 − 16  32 − 16 
1  x
= ln x 2 + 4 x + 8 − arctan 1 +  + C
2  2

3 3 1
54.  2x 9x − 1 2
dx = 
2 3 x (3 x ) 2 − 1
3 dx (u = 3 x)

3
= arcsec 3x + C (Formula 33)
2

1 1 1
55.  sin π x cos π x dx =
π  sin π x cos π x
(π ) dx (u = π x)

1
= ln tan π x + C (Formula 58)
π

1 1 1
 1 + tan π x dx (u = π x)
π  1 + tan π x
56. = (π ) dx

1 1 
=
π  2
(π x + ln cos π x + sin π x )  + C (Formula 71)

1
57.  θ sin θ cos θ dθ =
2
θ sin 2θ dθ

1 1 1 1 1
= − θ cos 2θ +  cos 2θ dθ = − θ cos 2θ + sin 2θ + C = (sin 2θ − 2θ cos 2θ ) + C
4 4 4 8 8
1
dv = sin 2θ dθ  v = − cos 2θ
2
u = θ  du = dθ

© 2018 Cengage Learning. All Rights Reserved. May not be scanned, copied or duplicated, or posted to a publicly accessible website, in whole or in part.
Review Exercises for Chapter 8 843

csc 2 x  1 
58.  x
dx = 2  csc 2 x   dx = −
 2x 
2 ln csc 2 x + cot 2x + C

1
u = 2 x , du = dx
2x

x1 4 u (u 3 )
59. 1 + x 12
dx = 4
1 + u2
du

 1 
= 4  u 2 − 1 + 2  du
 u + 1
1 
= 4 u 3 − u + arctan u  + C
3 
4  34
=  x − 3x1 4 + 3 arctan ( x1 4 ) + C
3
4
u = x , x = u 4 , dx = 4u 3 du

4u 5 4u 3 4
(1 + x) (3 x − 2) + C
32
  u (4u − 4u ) du =  (4u − 4u 2 ) du =
3 4
60. 1+ x dx = − +C =
5 3 15
u = 1+ x , x = u 4 − 2u 2 + 1, dx = ( 4u 3 − 4u ) du

1 + cos x 1 − cos x
61.  1 + cos x dx =  1

1 − cos x
dx

sin x
=  1 − cos x
dx

−1 2
=  (1 − cos x) (sin x) dx
= 2 1 − cos x + C
u = 1 − cos x, du = sin x dx

3 x3 + 4 x Ax + B Cx + D
62. 2
= +
( x + 1)
2 x2 + 1 ( x 2 + 1)2
3 x3 + 4 x = ( Ax + B )( x 2 + 1) + Cx + D = Ax 3 + Bx 2 + ( A + C ) x + ( B + D )
A = 3, B = 0, A + C = 4  C = 1,
B + D = 0  D = 0
3x3 + 4 x x x 3 1
 ( x2 dx = 3 dx +  dx = ln ( x 2 + 1) − +C
+ 1)
2
x2 + 1 ( x + 1)
2 2
2 2( x 2
+ 1)

63.  cos x ln(sin x) dx = sin x ln (sin x) −  cos x dx = sin x ln (sin x) − sin x + C


dv = cos x dx  v = sin x
cos x
u = ln (sin x)  du = dx
sin x

 (sin θ + 2 sin θ cos θ + cos 2 θ ) dθ


2
 (sin θ + cos θ ) dθ = 2
64.
1 1
=  (1 + sin 2θ ) dθ = θ −
2
cos 2θ + C = ( 2θ − cos 2θ ) + C
2

© 2018 Cengage Learning. All Rights Reserved. May not be scanned, copied or duplicated, or posted to a publicly accessible website, in whole or in part.
844 Chapter 8 Integration Techniques and Improper Integrals

25 1 x −5
65. y =  x2 − 25
dx = 25  ln
 10  x + 5
+ C

5 x −5
= ln + C
2 x +5
(Formula 24)

4 − x2 2 cos θ ( 2 cos θ ) dθ
66. y =  2x
dx =  4 sin θ
=  (csc θ − sin θ ) dθ
= −ln csc θ + cos θ + cos θ  + C

2+ 4 − x2 4 − x2
= −ln + +C
x 2

x = 2 sin θ , dx = 2 cos θ dθ , 4 − x 2 = 2 cos θ

2 x2 + x
 ln( x + x) dx = x ln x 2 + x − 
2
67. y = dx
x2 + x
2x + 1
= x ln x 2 + x −  dx
x +1
1
= x ln x 2 + x −  2 dx +  x + 1 dx
= x ln x 2 + x − 2 x + ln x + 1 + C

dv = dx  v = x
2x + 1
u = ln ( x 2 + x)  du = 2 dx
x + x

68. y =  1 − cos θ dθ 70.


1 x
 0 ( x − 2)( x − 4) dx
1
= 2 ln x − 4 − ln x − 2  0
sin θ
=  1 + cos θ
dθ = 2 ln 3 − 2 ln 4 + ln 2
9
= −  (1 + cos θ )
−1 2
(−sin θ ) dθ = ln ≈ 0.118
8
= −2 1 + cos θ + C 4
4 ln x 1 2 1 2
u = 1 + cos θ , du = −sin θ dθ 71. 1 dx =  (ln x )  = (ln 4) ≈ 0.961
x 2 1 2
5
dx =  15 ( x 2 − 4) 
5 32 52
x( x 2 − 4)
2
69. 2 2
= 1
5 72.
2
0
 e3 x  1
xe3 x dx =  (3 x − 1) = (5e6 + 1) ≈ 224.238
 9 0 9

 (x − 4) sin x dx = ( x2 − 4)( − cos x) − 2 x( − sin x) + 2 cos x + C


2
73.
π π
2 ( x − 4) sin x dx = ( 4 − x 2 ) cos x + 2 x sin x + 2 cos x
2
2

= ( 4 − π 2 )( −1) − 2 − [4 sin 2 + 2 cos 2]


= π 2 − 4 sin 2 − 2 cos 2 − 6 ≈ 1.0647

Alternate signs u and its derivatives v′ and its antiderivatives


2
+ x − 4 sin x
– 2x − cos x
+ 2 − sin x
– 0 cos x

© 2018 Cengage Learning. All Rights Reserved. May not be scanned, copied or duplicated, or posted to a publicly accessible website, in whole or in part.
Review Exercises for Chapter 8 845

5 x  2 x − 16 
5 78. By symmetry, y = 0.
74. 0 4+ x
dx = 
 3
4 + x
0 A = π + 4π = 5π
16 14 1(π ) + 4( 4π )
= −2(3) + ( 2) = x =
3 3 π + 4π
17π
32 = = 3.4
75. A = 0 x 3 − 2 x dx 5π
3−u ( x , y ) = (3.4, 0)
Let 3 − 2 x = u , − 2 dx = du , x =
2 y

0  3 − u  1 2 1  3
A = 3  u  − du 
 2   2 
2
1
(3.4, 0)
1
(3u ) du
3 x
0
12 32
= −u 1 3 4 5
4 −1
−2
3
1 3 2 2  −3
= 2u − u 5 2 
4  5 0
16
1 3 2 2  16 1 4  32
= 2(3 ) − (35 2 ) 79. 0 dx = lim  x 3 4  =
4  5 
4
x b → 0+  3 b 3
1 2 
= 6 3 − 9 3 7
4 
2 b
5  80. 0 x − 2
dx = lim 7 ln x − 2  0
b→2 −
3 = −∞ Diverges
= 3 ≈ 1.0392
5
b
∞  x3 
4 1 81. 1 x 2 ln x dx = lim  ( −1 + 3 ln x) = ∞
76. A = 0 25 − x 2
dx b →∞ 9 1

 1 x−5 1 1 1
4 Diverges
= − ln  = − ln = ln 9 ≈ 0.220
 10 x + 5 0 10 9 10 ∞ e −1 x b
82. 0 dx = lim [e −1 x ]a = 1 − 0 = 1
x2 a → 0+
1 b→∞
77. By symmetry, x = 0, A = π.
2
1
1 83. Let u = ln x, du = dx, dv = x −2 dx, v = − x −1.
y =    ( 1 − x 2 ) dx =  x − x 3  =
21 1 2 1 1  4 x
π  2  −1 π 3  −1 3π ln x −ln x 1 −ln x 1
 x2
dx =
x
+  x2 dx =
x
− +C
x
( x , y ) =  0, 
4
b
 3π  ∞ ln x  −ln x 1
y 1 x 2
dx = lim 
b → ∞  x
− 
x 1
 −ln b 1
2
= lim  −  − ( −1)
b → ∞ b b
= 0 +1 =1
(0, 34π( ∞ 1 b
1 1 x
−1 4
x 84. 4
dx = lim dx
−1 1 x b→∞

b
4 
= lim  x3 4 
b→∞3 1
4 4
= lim  b3 4 −  = ∞
b→∞3 3
Diverges

© 2018 Cengage Learning. All Rights Reserved. May not be scanned, copied or duplicated, or posted to a publicly accessible website, in whole or in part.
846 Chapter 8 Integration Techniques and Improper Integrals

∞ 1 3 1 ∞ 1
85. 2 x 2
x − 4
dx = 2 x 2
x − 4
dx + 3 x x2 − 4
dx

3 c
1  x  1  x 
= lim  arcsec  + lim  arcsec 
b → 2+  2  2  b c → ∞  2  2  3
1 3 1 1π  1 3
= arcsec  − (0) +   − arcsec 
2 2 2 2 2  2 2
π
=
4

1
86. Let u = x , du = dx  dx = 2u du.
2 x

2 2 4 u  x
 x ( x + 4)
dx =  u(u 2 + 4)
2u du =  u2 + 4
du = 2 arctan   + C = 2 arctan 
2
+C
 2 
1 c
∞ 2   x    x   1  π  1
0 x ( x + 4)
dx = lim 2 arctan 
b → 0+ 
 + lim 2 arctan 
 2  b c →∞ 
 =  2 arctan − 0  + 2  − 2 arctan = π
 2 1  2  2 2

t ∞ 2
t0  500,000 −0.05t  0 88. V = π  ( xe − x ) dx
87. 0 500,000e −0.05t dt = 
 −0.05
e 
0
0

−500,000 ( −0.05t0 = π  x 2e −2 x dx
= e − 1) 0
0.05 b
 π e −2 x
= 10,000,000(1 − e −0.05t0 ) = lim − (2 x 2 + 2 x + 1) = π
b →∞ 4 0 4
(a) t0 = 20: $6,321,205.59
(b) t0 → ∞ : $10,000,000

1 ∞
−( x −12.9)2 2(0.95)2
89. (a) P(13 ≤ x < ∞) =
0.95 2π 13 e dx ≈ 0.4581

1 ∞
−( x −12.9)2 2(0.95)2
(b) P(15 ≤ x < ∞) =
0.95 2π 15 e dx ≈ 0.0135

Problem Solving for Chapter 8


1
1  x3   1 4
1. (a)  (1 − x 2 ) dx =  x −  = 21 −  =
−1  3 −1  3 3
1
1
2 2
1  2 x3 x5   2 1  16
 −1 (1 − x ) dx =  −1 (1 − 2 x + x ) dx = x − 3 + 5  −1 = 21 − 3 + 5  = 15
2 4

(b) Let x = sin u, dx = cos u du, 1 − x 2 = 1 − sin 2 u = cos2 u.


1 π 2
2 n n
 −1 (1 − x ) dx =  −π 2 (cos u ) cos u du
2

π 2
 −π 2 cos
2 n +1
= u du

2 4 6 ( 2n ) 
= 2 ⋅ ⋅  (Wallis’s Formula)
3 5 7 ( 2n + 1) 
 2 2 ⋅ 4 2 ⋅ 6 2  ( 2n )
2

= 2 
 2 ⋅ 3 ⋅ 4 ⋅ 5  ( 2n)( 2n + 1) 
2( 22 n )( n!)
2 2
22 n +1 ( n!)
= =
( 2n + 1)! ( 2n + 1)!

© 2018 Cengage Learning. All Rights Reserved. May not be scanned, copied or duplicated, or posted to a publicly accessible website, in whole or in part.
Problem Solving for Chapter 8 847

1
= lim [ x ln − x]b
1
2. (a)  0 ln x dx b → 0+

= ( −1) − lim (b ln b − b) = −1
b → 0+

ln b 1b
Note: lim b ln b = lim −1
= lim 2
= 0
b → 0+ b → 0+ b b → 0 −1 b
+

1 1
 0 (ln x) dx = lim  x(ln x) − 2 x ln x + 2 x b
2 2
+ b→0

(
= 2 − lim b (ln b) − 2b ln b + 2b = 2
b → 0+
2
)
n
(b) Note first that lim b(ln b) = 0 (Mathematical induction).
b → 0+

n +1 n +1 n
Also,  (ln x) dx = x(ln x) − ( n + 1)  (ln x) dx.
1 n
 0 (ln x)
n
Assume dx = ( −1) n!.

1 1 1
dx = lim  x(ln x)  b − ( n + 1)  (ln x) dx = 0 − ( n + 1)( −1) n! = ( −1) ( n + 1)!.
n +1 n +1 n n +1
 0 (ln x)
n
Then,
b → 0+ 0

3. (a) R < I < T < L


4−0 1  1  1
(b) S ( 4) =  f (0) + 4 f (1) + 2 f ( 2) + 4 f (3) + f ( 4) ≈ 4 + 4( 2) + 2(1) + 4  +  ≈ 5.417
3( 4)  3  2  4

4. (a) 0.2

0 4
0

Area ≈ 0.2986
(b) Let x = 3 tan θ , dx = 3 sec2 θ dθ , x 2 + 9 = 9 sec2 θ .

x2 9 tan 2 θ
 ( x2  (9 sec2 θ )3 2 (3 sec θ dθ )
2
32
dx =
+ 9)
tan 2 θ
=  sec θ

sin 2 θ
=  cos θ dθ

1 − cos 2 θ
=  cos θ

= ln sec θ + tan θ − sin θ + C


2
4 x tan −1( 4 3)
Area = 0 ( x + 9)
2 32
dx = ln sec θ + tan θ − sin θ  0

4
  x2 + 9 x x 
= ln  +  − 
x2 + 9
  3 3 x 2 + 9  0 x
5 4 4 4
= ln  +  − = ln 3 − θ
3 3 5 5 3

© 2018 Cengage Learning. All Rights Reserved. May not be scanned, copied or duplicated, or posted to a publicly accessible website, in whole or in part.
848 Chapter 8 Integration Techniques and Improper Integrals

(c) x = 3 sinh u, dx = 3 cosh u du, x 2 + 9 = 9 sinh 2 u + 9 = 9 cosh 2 u


4 x2 sinh −1( 4 3) 9 sinh 2 u sinh −1( 4 3)
A =  0 ( x2 dx = 0 (3 cosh u du ) = 0 tanh 2 u du
(9 cosh 2 u )
32 32
+ 9)
sinh −1( 4 3) tsinh −1( 4 3)
= 0 (1 − sech 2 u ) du = [u − tanh u]0

 4   4  4 16   4 16 
= sinh −1   − tanh  sinh −1    = ln  + + 1  − tanh ln  + + 1 
 3   3  3 9   3 9 
4 5   4 5 
= ln  +  − tanh  ln  +   = ln 3 − tanh (ln 3)
3 3  3 3 
3 − (1 3) 4
= ln 3 − = ln 3 −
3 + (1 3) 5

x 1 − u2 −2 x
5. u = tan , cos x = , 6. y = ln (1 − x 2 ), y′ =
2 1 + u2 1 − x2
1 − u2 3 + u2 2 4x2
2 + cos x = 2 + 2
= 1 + ( y′) = 1 +
1+u 1 + u2 (1 − x 2 )2
2 du
dx = 1 − 2 x2 + x4 + 4x2
1 + u2 =
(1 − x 2 )2
π 2 1 11
+ u 2  2 
0 2 + cos x
dx =  0  3 + u 2   du
 1 + u 2  1 + x2 
2

=  
1 2  1 − x2 
=  du
0 3 + u2 12
1 + ( y′) dx
2
1
Arc length = 0
 1  u 
= 2 arctan   1 2 1
+ x2 
 3  3  0 = 0
  dx
 1 − x2 
2  1 
= arctan   12 2 
3  3 =   −1 +  dx
0  1 − x2 
2 π  π 3 12
=   = ≈ 0.6046 1 1 
36 9 =   −1 + +  dx
0  x + 1 1 − x
= [− x + ln (1 + x) − ln (1 − x)]0
12

 1 3 1
=  − + ln − ln 
 2 2 2
1
= − + ln 3 − ln 2 + ln 2
2
1
= ln 3 − ≈ 0.5986
2

© 2018 Cengage Learning. All Rights Reserved. May not be scanned, copied or duplicated, or posted to a publicly accessible website, in whole or in part.
Problem Solving for Chapter 8 849

7. Let u = cx, du = c dx.


b
− c2 x2
cb 2 du 1 cb 2
0 e dx = 0 e−u
c
=  e − u du
c 0
∞ 2 x2 1 ∞ − x2
As b → ∞ , cb → ∞. So, 0 e−c dx =
c0
e dx.

x = 0 by symmetry.

2
∞ (e ) dx − c2 x2

Mx 0 2
y = = ∞
m − c2 x2
2 e dx
0
∞ 2 2
 1  0
e −2c x dx
=  ∞ 22
 2
 e dx
−c x
0

1 ∞ 2

 1  2c  0
e − x dx
= 
 2  1 ∞ e − x2 dx
c0
1 2
= =
2 2 4
 2
So, ( x , y ) =  0, .
 4 

8. f ′( a)(b − a) −
b
 a f ′′(t )(t − b) dt { b
= f ′( a)(b − a) −  f ′(t )(t − b) a −  a f ′(t ) dt}
b

b
= f ′( a)(b − a) + f ′( a)( a − b) +  f (t ) a = f (b) − f ( a)

dv = f ′′(t ) dt  v = f ′(t )
u = t −b  du = dt

9. (a) Let y = f −1( x), f ( y) = x, dx = f ′( y) dy.

f  yf ′( y) dy
−1 (
x) dx =
u = y, du = dy 
= yf ( y ) −  f ( y ) dy  
dv = f ′( y ) dy , v = f ( y )
= xf −1 ( x ) −  f ( y ) dy

(b) f −1 ( x) = arcsin x = y, f ( x) = sin x

 arcsin x dx = x arcsin x −  sin y dy = x arcsin x + cos y + C = x arcsin x + 1 − x2 + C

3 x+2

9 − (x + 2) 2

(c) f ( x) = e x , f −1 ( x) = ln x = y x = 1 ⇔ y = 0; x = e ⇔ y = 1
e 1 1
1 ln x dx = [ x ln x]1 −  0 e dy = e − [e y ]0 = e − (e − 1) = 1
e y

© 2018 Cengage Learning. All Rights Reserved. May not be scanned, copied or duplicated, or posted to a publicly accessible website, in whole or in part.
850 Chapter 8 Integration Techniques and Improper Integrals

10. x 4 + 1 = ( x 2 + ax + b)( x 2 + cx + d )
= x 4 + ( a + c) x3 + ( ac + b + d ) x 2 + ( ad + bc) x + bd

a = −c, b = d = 1, a = 2
x4 + 1 = ( x2 + 2 x + 1)( x 2 − 2 x + 1)
1 1 1 Ax + B 1 Cx + D
0 4
x +1
dx = 0 2
x + 2x + 1
dx + 0 2
x − 2x + 1
dx

1 2 1 2
1
+ x 1
− + x
2 4 2 4
= 0 x2 + 2x + 1
dx − 0 x2 + 2x + 1
dx

2 1 2 1
arctan ( 2 x + 1) + arctan ( 2 x − 1) 0 + ln ( x + 2 x + 1) − ln ( x 2 − 2 x + 1) 0
2
=
4 8
2 2 2 π π 2
= arctan ( 2 + 1) + arctan ( 2 − 1) + ln ( 2 + 2 ) − ln ( 2 − 2 ) − −  − [0]
4 8 4  4 4 8
≈ 0.5554 + 0.3116
≈ 0.8670

N ( x) P1 P2 Pn
11. = + ++
(
D x ) x − c1 x − c2 x − cn
N ( x) = P1 ( x − c2 )( x − c3 )  ( x − cn ) + P2 ( x − c1 )( x − c3 )  ( x − cn ) +  + Pn ( x − c1 )( x − c2 )  ( x − cn −1 )

Let x = c1 : N (c1 ) = P1 (c1 − c2 )(c1 − c3 )  (c1 − cn )


N (c1 )
P1 =
(c1 − c2 )(c1 − c3 )  (c1 − cn )
Let x = c2 : N (c2 ) = P2 (c2 − c1 )(c2 − c3 )  (c2 − cn )
N (c2 )
P2 =
(c2 − c1 )(c2 − c3 )  (c2 − cn )
 
Let x = cn : N (cn ) = Pn (cn − c1 )(cn − c2 )  (cn − cn −1 )
N (cn )
Pn =
(cn − c1 )(cn − c2 )  (cn − cn −1 )
If D( x) = ( x − c1 )( x − c2 )( x − c3 )  ( x − cn ), then by the Product Rule
D′( x) = ( x − c2 )( x − c3 )  ( x − cn ) + ( x − c1 )( x − c3 )  ( x − cn ) +  + ( x − c1 )( x − c2 )( x − c3 )  ( x − cn −1 )
and
D′(c1 ) = (c1 − c2 )(c1 − c3 )  (c1 − cn )
D′(c2 ) = (c2 − c1 )(c2 − c3 )  (c2 − cn )

D′(cn ) = (cn − c1 )(cn − c2 )  (cn − cn −1 ).
So, Pk = N (ck ) D′(ck ) for k = 1, 2, , n.

© 2018 Cengage Learning. All Rights Reserved. May not be scanned, copied or duplicated, or posted to a publicly accessible website, in whole or in part.
Problem Solving for Chapter 8 851

x3 − 3x 2 + 1 P P2 P3 P4
12. = 1 + + +  c1 = 0, c2 = 1, c3 = − 4, c4 = 3
x − 13x 2 + 12 x
4
x x −1 x + 4 x −3
N ( x) = x 3 − 3 x 2 + 1
D′( x) = 4 x3 − 26 x + 12
N (0) 1
P1 = =
D′(0) 12
N (1) −1 1
P2 = = =
(
D′ 1 ) −10 10
N ( −4) −111 111
P3 = = =
D′( −4) −140 140
N (3) 1
P4 = =
D′(3) 42
x3 − 3x 2 + 1 1 12 1 10 111 140 1 42
So, = + + + .
x − 13x 2 + 12 x
4
x x −1 x + 4 x −3

π
13. (a) Let x = − u , dx = du.
2
π 
sin  − u 
π 2 sin x 0 2  π 2 cos u
I = 0 cos x + sin x
dx = π 2 π  π 
( −du ) = 0 sin u + cos u
du
cos − u  + sin  − u 
2  2 
So,
π 2 sin x π 2 cos x π 2 π π
2I = 0 cos x + sin x
dx + 0 sin x + cos x
dx = 0 1 dx =
2
 I =
4
.

π 
sin n  − u 
0 2  π 2 cos n u
(b) I = π 2 nπ  nπ 
( −du ) = 0 sin u + cos n u
n
du
cos  − u  + sin  − u 
2  2 
π 2 π π
So, 2 I = 0 1 dx =
2
 I =
4
.

1
14. Consider  ln x dx.
1 1 1 eu
 ln x dx  ue u
u
Let u = ln x, du = dx, x = eu . Then = du = du.
x

1 eu
If  ln x
dx were elementary, then  u
du would be too, which is false.

1
So,  ln x dx is not elementary.

© 2018 Cengage Learning. All Rights Reserved. May not be scanned, copied or duplicated, or posted to a publicly accessible website, in whole or in part.
852 Chapter 8 Integration Techniques and Improper Integrals

 20,000 
15. s(t ) =  −19.6t + 4000 ln
20,000 − 200t 
dt = − 9.8t 2 + 4000  ln 20,000 − ln ( 20,000 − 200t ) dt

 − 200t 
= − 9.8t 2 + 4000t ln 20,000 − 4000 t ln ( 20,000 − 200t ) −  dt
 20,000 − 200t 
20,000  20,000 
= − 9.8t 2 + 4000t ln + 4000 1 −  dt
20,000 − 200t  20,000 − 200 t 
20,000
= − 9.8t 2 + 4000t ln + 4000t + 400,000 ln ( 20,000 − 200t ) + C
20,000 − 200t
s(0) = 400,000 ln 20,000 + C = 0
C = − 400,000 ln 20,000
 20,000  20,000 − 200t
s(t ) = − 9.8t 2 + 4000t 1 + ln  + 400,000 ln
 20,000 − 200t  20,000
When t = 90, s(90) ≈ 188,516.6 m.

16. By parts,
b b
f ( x) g ′′( x) dx = [ f ( x) g ′( x)]a −
b
a a f ′( x) g ′( x ) dx u = f ( x), dv = g ′′( x) dx
b
= −  f ′( x) g ′( x) dx
a
b
= [− f ′( x) g ( x)]a +
b
 a g ( x) f ′′( x) dx u = f ′( x), dv = g ′( x ) dx
b
= a f ′′( x) g ( x) dx.

17. Let u = ( x − a)( x − b), du = [( x − a) + ( x − b)] dx, dv = f ′′( x) dx, v = f ′( x).


b b
 a ( x − a)( x − b) dx = [( x − a)( x − b) f ′( x)]a −  a [( x − a) + ( x − b)] f ′( x) dx
b

b u = 2x − a − b
= −  ( 2 x − a − b) f ′( x) dx  
a
 dv = f ′( x) dx 
b b
= [−( 2 x − a − b) f ( x )]a +
b
 a 2 f ( x) dx = 2  f ( x) dx
a

∞ 1 1 1  ∞ 1 ∞ 1 1 2 
18. 2  x5 + x10 + x15  dx < 2 x5 − 1
dx < 2  x 5 + x10 + x15  dx
b b
 1 1 1  ∞ 1  1 1 1 
lim − 4 − 9 −
b →∞ 
 4x 9x 14 x14  2
< 2 x5 − 1
dx < lim − 4 − 9 − 14 
b →∞  4 x 9x 7 x 2
∞ 2
0.015846 < 2 x5 − 1
dx < 0.015851

1 arcsin(c) 2 π 2 2 2c 2π − 8c + π
19. V = 0 π (c − sin x) dx +  arcsin(c) π (sin x − c) dx = π = f (c)
2 4
4cπ − 8 2
f ′(c) = π = 0  c =
4 π
1 π2
For c = 0, V = ≈ 2.4674.
2 4
1 π
For c = 1, V = (3π − 8) ≈ 1.1190.
2 4
2 1 π2 − 8
For c = , V = ≈ 0.4674.
π 2 4
(a) Maximum: c = 0
2
(b) Minimum: c =
π

© 2018 Cengage Learning. All Rights Reserved. May not be scanned, copied or duplicated, or posted to a publicly accessible website, in whole or in part.

You might also like